Jump to content

Wikipedia:Reference desk/Science

From Wikipedia, the free encyclopedia

This is an old revision of this page, as edited by Trevor Loughlin (talk | contribs) at 10:58, 3 October 2009 (→‎Speed of light between casmir plates?). The present address (URL) is a permanent link to this revision, which may differ significantly from the current revision.

Welcome to the science section
of the Wikipedia reference desk.
Select a section:
Want a faster answer?

Main page: Help searching Wikipedia

   

How can I get my question answered?

  • Select the section of the desk that best fits the general topic of your question (see the navigation column to the right).
  • Post your question to only one section, providing a short header that gives the topic of your question.
  • Type '~~~~' (that is, four tilde characters) at the end – this signs and dates your contribution so we know who wrote what and when.
  • Don't post personal contact information – it will be removed. Any answers will be provided here.
  • Please be as specific as possible, and include all relevant context – the usefulness of answers may depend on the context.
  • Note:
    • We don't answer (and may remove) questions that require medical diagnosis or legal advice.
    • We don't answer requests for opinions, predictions or debate.
    • We don't do your homework for you, though we'll help you past the stuck point.
    • We don't conduct original research or provide a free source of ideas, but we'll help you find information you need.



How do I answer a question?

Main page: Wikipedia:Reference desk/Guidelines

  • The best answers address the question directly, and back up facts with wikilinks and links to sources. Do not edit others' comments and do not give any medical or legal advice.
See also:



September 26

Blood thinners and broken bones

A guy at the school I attend recently died. From what I understand of the case, he had broken his leg a couple weeks ago. Then just the other day, he collapsed and died due to a blood clot going to his brain. I've heard of this from other places as well where someone breaks a bone and then dies of a stroke due to a blood clot breaking free or whatever they do. So, this got me thinking, would blood thinning medication help this? Also, how rare is this?

Note: I am not requesting medical advice! I am asking out of medical curiosity only. I do not have any broken bones. I have never even had a broken bone, torn a ligament, or sprained an ankle. To my knowledge, I do not know anyone who currently has a broken bone. I did not know the first person that I mentioned and the second person died well over five years ago. I am not going to self medicate any future illness or injury based on your responses. I am not currently, or recently, under any doctor's care due to a bone or blood issue. I do not suffer from any mental illness that would make me lie about my current well being. I am of reasonably sound mind (as much as many of us, at least) and am asking the preceding question of my own free will. I have not died. I have read the reference desk header and am aware of Wikipedia's policy pertaining to not giving medical advice, so please do not advise me of that policy. As I said, I am simply asking out of my own curiosity. Thank you for any useful response that you can give that would satisfy my curiosity, Dismas|(talk) 00:12, 26 September 2009 (UTC)[reply]

LOL. Blood thinners might help, but there's also a risk involved, as they can cause internal bleeding. StuRat (talk) 03:54, 26 September 2009 (UTC)[reply]

Slightly off-topic, but this term blood thinner is one of my mild peeves. I think a lot of people hear the term and imagine, not unreasonably, that these pharmaceuticals actually make your blood, you know, thinner. Less viscous.

I don't think they do anything of the sort. What anticoagulants actually do is, they interfere with the clotting mechanism at some stage (clotting is an amazingly complicated cascade of events). Since under normal circumstances your blood doesn't clot, there's no reason I can see that interfering with that mechanism should make the blood less viscous. --Trovatore (talk) 05:20, 26 September 2009 (UTC)[reply]

How certain are you that he died of a blood clot on the brain? It is more likely that after a broken long bone death is caused by a fat embolism, [1]in the brain. If this were the case then anticoagulant therapy would be of little help. Richard Avery (talk) 09:29, 26 September 2009 (UTC)[reply]
Doesn't that depend on how bad the fracture was? Clots from immobilised limbs do happen but AFAIK they don't give routine Warfarin unless there are other genetic or lifestyle risk factors. --BozMo talk 10:20, 26 September 2009 (UTC)[reply]

What you are describing, Dismas, appears to be something called a DVT and the resultant pulmonary embolism. The basics are these: Virhow's Triad states that any mix of hypercoagulability (a state that makes the blood more likely to clot), venous stasis (blood sitting in a vein not really moving) and vascular damage (injury to the inside wall of the veins, in this case) increases the risk of the formation of a blood clot, or thrombus. A broken leg certainly causes stasis, as the movement of blood through the lower extremity is very much dependent on the use of the calf muscles to pump the blood. This injury is also consistent with the possibility of vascular damage. The problem of hypercoagulability is also likely present. Several things can make us hypercoagulable. Surgery anywhere in the body exposes something called tissue factor which can increase coagulation. Fracture itself is known to do the same through another independent mechanism unknown to me, and several genetic traits can increase one's risk of making a blood clot even if they have been silent for a lifetime ( factor V leiden, prothrombin 20210, etc). The problem with forming such a blood clot is that they tend to travel. In most people, the venous system is not directly connected to the arterial system in a way that would allow a large clot to pass from a vein to an artery. And so a clot anywhere in the venous system passes through larger and larger veins until it gets to the heart. Here, it would be pumped into the lungs, where it may cause a potentially deadly condition known as a pulmonary embolus. However there is more. A certain non-trivial percentage of the population has a hole between the two sides of their heart called a patent foramen ovale, or PFO. This hole is normally closed, as it has a flap over it that stays shut as long as the pressure on the right (venous) side of the heart is less than that on the left (arterial) side. However, a pulmonary embolism, if big enough, can reverse that pressure differential, opening the hole. The danger here is that if one clot has passed through the heart to go to the lungs and cause a PE that opened a PFO, another clot may come also, and pass through the PFO into the arterial circulation. A PE is an emergent medical problem, but a thromboembolism in the arterial circulation is even worse. A clot here can easily travel to the brain, and cerainly will cause damage pretty much anywhere it lands. If the patient you describe really died from a clot to the brain, this is the likely mechanism. Tuckerekcut (talk) 15:15, 26 September 2009 (UTC)[reply]

Thank you everyone for your responses. And no, I'm not certain that it was a blood clot in the brain that killed him. That's just the word around campus. Thanks again, Dismas|(talk) 09:36, 27 September 2009 (UTC)[reply]

Could number of binary stars tell us how many planets there are?

I recently read in WP article Binary Stars that a massive 1/3 of ALL stars in the Milky Way (and thus presumably in other galaxies) are in binary or multi-star systems. It then occured to me that solar systems like our own could easily have been a binary star system if Jupiter had been larger than it is. (Jupiter is sometimes called a "failed star"). If the physical processes that cause binary / multi-star systems and planetary systems are essentially the same, then we might be able to make good deductions as to how copious is the number of solar systems. If 1/3 of all stars are binaries or multi, then that suggests that there are a hell of a lot of solar systems. Especially when you factor in that binary star systems would be a lot rarer than sun to planet systems. I would guess that just about EVERY sun had a solar system. Ironically, I would guess that the suns WITHOUT solar systems would be more likely to be binary systems, as the complex gravitational effects to do with the "3 body problem" would make for a much more chaotic system. Planetary perturbations would disturb orbits leading to satellites being ejected from the system or cracking up, and making life (which needs very mild, stable conditions) just about impossible. Anyone think this idea has anything going for it? Myles325a (talk) 04:54, 26 September 2009 (UTC)[reply]

This question has been discussed in literature; If I remember correctly, Iosif Shklovsky in "Vselennaya, Zhizn', Razum" (aka Intelligent Life in the Universe) discusses this question, although at the purely theoretical level (the book is 40 years old...). Our article on Drake equation, however, does not discuss this at all. Now for your question proper. Let's assume the binary system is of a detached type (most binary systems probably are, and anyway that's the least volatile type; so a habitable planet can actually stay habitable for a time sufficient to develop life). A habitable planet may orbit either one star of the two (if the stars are sufficiently far apart), or both stars (if the stars are sufficiently close together); in both cases the orbit is nearly elliptical, and may persist indefinitely. Now, you suggest a possibility of deduction of fP from prevalence of binary systems; well, I am not convinced. It may turn out that there are relatively more gas giants in binary systems because of the initial conditions (more hydrogen??? higher angular momentum???), or relatively fewer gas giants in binary systems (stability issues??? Protoplanetary disk dynamics???). We don't know even that for sure. Gas giants and rocky planets are quite different from each-other, too. So, to the best of my knowledge, at present it is impossible to say if there are more or fewer rocky planets in binary systems. It is also impossible to say if a binary system is more conductive to the development of life. I would guess that the cometary bombardment should be more intense in a binary system, which may be a good thing or a bad thing. So, to summarize: you ask very good questions, but the answers are unknown so far. --Dr Dima (talk) 06:05, 26 September 2009 (UTC)[reply]
There is an entire chapter, "Multiple Star Systems", in Intelligent Life in the Universe. Among the theories postulated are in-situ co-formation of the stars; a single star which takes on an unstable, non-spherical form and fissures into two stars; gravitational capture of a star as it drifts near another; and it seems that there's a U.S./Soviet rift on the scientific validity of each of these theories. Naturally, each formation mechanism for binary stars also has implications to planetary formation, but even in our Solar System, the process of planetary formation is not completely understood. As Dr Dima has pointed out, you might really enjoy Shklovskii's book - the questions you raise are not yet solidly answered by even the great cosmologists. Nimur (talk) 14:32, 26 September 2009 (UTC)[reply]

Op myles325a back here. Thanks for those solid and interesting answers. I was happy to see that my ideas were not simply way out of line. On an associated topic, a similar, simple piece of logical speculation might go as follows. If we on planet Earth know about pulsars when the beam of light that emanates from them intersects with the Earth (the "lighthouse effect"), then could we not make a good guess as to how many pulsars there are in the Milky Way by extrapolating from this? If the orientation of such beams is purely random, then simple math should give us a good a priori approximation of how many pulsar beams do NOT intersect with the Earth, and thus how many there are overall. (Incidentally, I have left a complaint on the Neutron Star talk table to the effect that the article is ambiguous as to whether ALL neuton stars are pulsars and we only see the ones whose beam shines over the Earth, or whether pulsars are only some limited sub set of neutron stars. If the latter, the article gives no idea of what proportion of neutron stars might be pulsars, or why some would be pulsars and some not.) But these are other questions. Myles325a (talk) 23:31, 27 September 2009 (UTC)[reply]

Efects of a electromagnetic pulse on living organisms

It's a well known fact that emp's will toast any electronic devices in range, and that on the other hand, they are harmless to any living being. But why? I know why electronic devices are affected, but I have never heard any reason why living beings aren't affected other than a blunt "we don't have wires".

Can anyone direct me to any scientific articles that explain in detail why living organisms are immune to emp's? Why the nervous system (which works on electricity) doesn't act as a bunch of wires or why aren't muscles (which also work on electricity) affected by the surges of an emp? —Preceding unsigned comment added by User:79.113.188.135 (talkcontribs)

An electromagnetic pulse affects electrical and electronic devices by inducing currents and voltage surges in wires and other metal parts. Nerves are not wires and nerve impulses are electrochemical, not electromagnetic, so people and animals are not directly affected by an EMP - although electrocution could be an indirect danger for anyone touching an ungrounded metal object. This item at Yahoo! Answers gives more details. Gandalf61 (talk) 11:22, 26 September 2009 (UTC)[reply]

Any chance of a link to a in depth scientific article please? —Preceding unsigned comment added by User:79.113.188.135 (talkcontribs)

The Wikipedia articles that I linked to above explain how the nervous system works, and how it does not "work on electricity" in the way that a light bulb or a toaster does. An article such as action potential has references and reading lists if you want to explore that topic further. Gandalf61 (talk) 13:26, 26 September 2009 (UTC)[reply]
The nervous system works by electrochemical processes, and is extremely responsive to applied electricity, even if it is not a simple electric circuit like a flashlight. 1 milliampere is easily felt through the skin. Quite low voltages can be felt by some sensory nerves. Strong magnetic fields can affect the functioning of the brain. The activity of the brain and nerves can be recorded by surface electrodes, and muscle activity can be controlled by external pulses. Edison (talk) 19:37, 26 September 2009 (UTC)[reply]
The OP's assertion that electromagnetic pulses "toast" electronic devices is a common misconception. Even a sufficiently powerful pulse must somehow couple into the device's circuitry and induce enough current or voltage to damage something. This is never a "well known fact", it is a case-by-case analysis that must be analyzed for any particular definition of "electromagnetic pulse". It may help to read about electromagnetic pulse - it will help you understand that this phrase actually refers to a huge class of phenomena. Some of them can damage electronics temporarily - or permanently. Some can damage humans - temporarily or permanently. A dose of gamma radiation would count as an EMP and could have long-term health consequences to a living creature. A flashbang or firework releases a bright flash of light - an EMP - which can temporarily stun a human. An FM radio transmitter broadcasts EMPs every minute of the day - and your car stereo doesn't get "fried" each time it tunes in to a station. The term "EMP" has been greatly overused - it's such a vague term that it's hard to make any blanket statement about it at all. Nimur (talk) 14:40, 26 September 2009 (UTC)[reply]
Standing in front of a radar transmitter was a popular way to keep warm until it was found that the electromagnetic radiation made men sterile. Cuddlyable3 (talk) 23:39, 26 September 2009 (UTC)[reply]
Perhaps one way to look at the difference between animals and electronics is to consider the difference in impedance (which, in this context, is basically a fancy word for resistance). Animals are bags of brine, which is a good conductor, so their internal tissues tend to have resistances on the order of a few ohms. It would take a strong external magnetic field to induce a harmful voltage in them. Modern electronic circuits, on the other hand, tend to have high impedances on the order of millions or even billions of ohms. This means that a small external magnetic field can induce a high voltage in a circuit and destroy it.
Perhaps you meant to say that the insulating substrate of modern electronics has very high impedance. I question it being more than tens or hundreds of megohms from one part of a printed circuit to another. Within the circuit, there are resistors and semiconductors, which do not have the thousand megohm resistance you claim. Within the human body, once the skin is punctured, the resistance would go down to hundreds of ohms. Resistance of dry skin would be much higher. Edison (talk) 14:27, 27 September 2009 (UTC)[reply]
One technique that the military uses to harden its critical electronic systems against EMP is to go back to old-fashioned low-impedance circuitry, like electromagnetic relays and some types of vacuum tubes. These systems, like living organisms, would probably survive the EMP if they survived the heat and blast of the explosion itself. --Heron (talk) 10:39, 27 September 2009 (UTC)[reply]

Isotope´s half-life measurement units

In a table of isotopes of certain element... for example this one:

there´s a cloumn for the half-life of each isotope. In that section, the units used like "µs", "ns", "min" are micro seconds, nano seconds and minutes accordingly (if I´m not wrong), but what does the "a" means? - Damërung . -- 13:28, 26 September 2009 (UTC)[reply]

I was going to guess Attosecond, but that is normally as. SGGH ping! 13:32, 26 September 2009 (UTC)[reply]
Very likely it is annum or "year". I just verified with this source. The numbers match, but the abbreviation used there is "yr." The usage of a is fairly archaic. We might consider changing the article. Nimur (talk) 14:50, 26 September 2009 (UTC)[reply]
Editing done (a to yr). B00P (talk) 19:02, 26 September 2009 (UTC)[reply]
Whoa there, it's not archaic. Although the year is not a "unit accepted for use with the SI" in the SI standard, which could have specified the symbol to be used, the official US Guide for the Use of the International System of Units says this:
Although there is no universally accepted symbol for the year, Ref. [4: ISO 80000-3] suggests the symbol a.
And this is echoed by Wikipedia's article year, right at the top. Similarly, Russ Rowlett's excellent web site about units of measure says here that a is "the official abbreviation for the year in scientific writing". (It seems that "official" is an exaggeration and I'll write to him about that.) --Anonymous, 19:50 UTC, September 26, 2009.

In space no one can hear you...

I recnelty through the f=ma equation around a couple of times to work out the acceleration given to a stationary naked person in deep space (somehow alive) if that person passed wind. For a one second "expulsion" from a 90kg person, which some guesses as to the velocity of such an eruption, I hazarded a rought guess of about 0.5m/s/s or thereabouts. Bearing in mind this was a mess about in a pub, how (im)plausible is that figure? SGGH ping! 13:30, 26 September 2009 (UTC)[reply]

This has been done well before you were born,,, Try again to make sure of yuor findings!

(Edit Conflict) Presumably your m = metres, not miles! 0.5m/s/s equates to 1.1 miles per hour per second, which intuitively seems implausibly high. What were your estimates for the mass and velocity of your, um, expelled reaction mass? My own hasty guesstimation gives an answer on the order of 0.00005ms-2, assuming a fart-mass of about 1 gram (probably over generous) and a fart-velocity of about 10mph. 87.81.230.195 (talk) 14:26, 26 September 2009 (UTC)[reply]
I in fact had a far highest guess at the velocity, and a slightly higher mass. Shame, no future in my propulsion theories! SGGH ping! 14:39, 26 September 2009 (UTC)[reply]
In all seriousness, the physics here is trivial. But, your answers are wildly inaccurate because you don't know anything about the expelled gas. The only unknowns are its mass and velocity (ergo, the momentum), of the flatus gas. You might be surprised and elated to learn that quantitative studies of flatus gas exist. Test subjects were connected, via rectal tube, to a gas impermeable bag: "Flatus was collected via a rectal tube (Davol, Cranston, Rhode Island, USA) connected to a gas impermeable bag (Quintron, Milwaukee, Wisconsin, USA). Preliminary studies showed that the anus makes a gas tight seal with a rectal tube, provided that the tube is patent. Each passage was collected in a separate bag, and volume was determined by aspiration into a calibrated syringe." The average gas volume was 107 mL, the expulsion velocity is difficult to estimate; but you can calculate the mass very precisely. This study also links to numerous other quantitative flatus studies, and you can probably estimate a velocity from the pressure gradient and cross-sectional area of the orifice. Surely, when you posted this question on the science desk, you expected a little science? Nimur (talk) 15:11, 26 September 2009 (UTC)[reply]
Holy shit! They ran a tube from some guy's ass in Cranston, Rhode Island to a bag in Milwaukee, Wisconsin? Damn, that's quite an elaborate way to collect fart gas. --Jayron32 00:34, 27 September 2009 (UTC)[reply]
Those are the suppliers and manufacturers for the equipment. The author of that journal article provided the suppliers' contact information in case you want to replicate his work. Nimur (talk) 18:59, 27 September 2009 (UTC) [reply]
Can't you work out the speed using the temperature? It won't move that fast under pressure, but this is space. — DanielLC 05:40, 27 September 2009 (UTC)[reply]
This is rocket science.Cuddlyable3 (talk) 23:34, 26 September 2009 (UTC)[reply]
Atmospheric pressure is really quite strong. I'd assume that the pressure differential that produces a fart under normal conditions is rather small compared to atmospheric pressure. I.e., the pressure in the rectum of someone who's about ready to fart as of when they got thrown out of an air lock is going to be just a little bit over 1 atm. To calculate the force then produced when that gas is expelled into a vacuum, we just need to multiply that pressure by the area of the opening through which it passes. That area is going to vary from person to person, of course, and also from fart to fart for a given person. But if you use an estimated area of 0.5 cm2, that leads to a force of roughly 5N, which produces an accelleration of roughly 0.05 m/s2. Red Act (talk) 20:11, 27 September 2009 (UTC)[reply]
This might be one of the few times in life where you'd be better off to shart, since it would increase the reaction mass. --Sean 15:23, 28 September 2009 (UTC)[reply]

Question about Science V Religon

Hi,

I am Protestant and somewhat of a scientist akso. I am not against god but I want to prove him, have you any clues??

As if I would get clues! I Believe in what is preeched to me! However god gave me a free mind and I question the answer's given to me!

It is important to understand that the Word of god in god's own book is true. Now and then someone finds a bit is irrational or just not true. Then we change infallible to allegorical. How do you know you have a free mind?Cuddlyable3 (talk) 14:29, 26 September 2009 (UTC)[reply]
Many theologians and scientists (some of whom are both) agree on definitions of "God(s)" and "prove" which mean that it is impossible to either prove or disprove the existence of God(s) by scientific reasoning or evidence. You might like to read the article on Non-overlapping magisteria. 87.81.230.195 (talk) 14:34, 26 September 2009 (UTC)[reply]
If you want to use the scientific method - you need to start by defining god, and then you can set about logically deducing some hypothetical consequences that would arise from its existence. Then, you need to set up an experiment and test for these consequences. You also need a control experiment to test the null hypothesis. Finally, you need to analyze the results to determine whether it is likely that god exists. If you feel sufficiently confident, you can boast that you have "proved" something, but most scientists are a little bit more humble than that - we prefer to say we have "shown" or "demonstrated", but we rarely "prove" anything. If you prefer not to use the scientific method, then all bets are off - you can say you've proved or disproved god even without empirical evidence! Nimur (talk) 15:00, 26 September 2009 (UTC)[reply]
If you think that the scientific method will get anywhere on this you could always start with seeing how it gets on with a similar hypothesis, namely "you". Decide what you think it means to say that "you" exist rather than are "just" a collection of chemical reactions. Don't worry, you do exist (or at least conscious beings capable of observing and deciding are assumed in the empirical method so you won't conclude you do not exist unless you make an error). Once you have mastered the way that the scientific method might or might not deal with the existence of a being then you could try on someone else. Finally once you have worked out whether they exist do by all means try God. Many people attempt to do the difficult "God" case without mastering the elementary cases and therefore end up with errors (e.g. "not needed to explain all phenomena" is an invalid argument since it gives an error looking at people). --BozMo talk 15:18, 26 September 2009 (UTC)[reply]
I agree that you need to start with a solid definition for this term "god" - I don't imagine you'd have too much trouble coming up with something acceptable: "God is defined as an omniscient being with unlimited powers"...or something like that. Then you write down a hypothesis: "There exists a God (defined as:...whatever...)."...woohoo! We have a hypothesis! So far, no problem!
However, the next step in the scientific process is to devise an experiment - or make a prediction for some future event that can be measured experimentally - that would prove your hypothesis. This can't be something like "Look at the amazing nature of the universe - God is self-evident." - that's not gonna fly. It has to be some kind of concrete measurement that has not yet been made that would turn out to be either true (if your hypothesis is true) or false (if it's not).
The second part of that is essential...you can't come up with an experiment that will always come out true - it MUST be "falsifiable". Falsifiability is a crucial part of this step of the process - and if you are serious about this then it's essential that you read and understand our article on that if you'd like your results to have scientific merit.
But this is where you're going to run into trouble. No such experiment has ever been devised that would provide this proof/disproof possibility. The god of common mythology steadfastly refuses to prove his/her/it's existence.
Let's suppose that you predicted that God would make a pink piano-playing Aardvark spontaneously appear in the middle of the Nobel-Prize for Physics award ceremony this year. If this actually happened after you predicted it...then this would widely be regarded as some sort of proof of your "God exists" hypothesis. By our earlier definition, a god would certainly have the power to do that. But the problem is that you can't write: "My hypothesis is proven if a pink piano-playing Aardvark spontaneously appears in the middle of the Nobel prize for Physics ceremony." - because you haven't said what happens if the Aardvark doesn't appear in the specified manner. I'm sure you'd agree that the non-appearance of the aardvark is not a disproof of your hypothesis. Maybe God just didn't feel like making a pink Aardvark? So this experiment can't disprove ("falsify") your hypothesis. So that's not a valid experiment and no serious scientist will support it as an acceptable experiment to back the hypothesis that God exists.
So if we suppose that this particular god does not wish to have his/her/it's existence proven - and if its/her/his powers are literally infinite - then it would be reasonable to presume that he/she/it could adjust the results of any experiment you might devise to produce ambiguous results - or even to manipulate our minds to make us forget the results - or change the course of time such that the experiment was never performed. That means that there is no experiment you could possibly come up with to prove that God DOESN'T exist because God always has the power to make the experiment come out any way he wants - including the "false" way. So no concievable experiment can be performed that could prove god's existance that would pass this "unfalsifiability" test.
The only way to 'trap' a god into being proven would be to limit his/her/it's powers in some manner - find some experiment that the god couldn't tamper with in any way. But that violates our original definition of what a god is - gods are (by definition) infinitely powerful. This means that the only way to devise a solid, falsifiable experiment is to deny the very thing you're attempting to prove.
Hence, you (along with a LOT of other people over the ages) are doomed to failure. There cannot ever be proof of existence of a being with limitless powers unless he/she/it chooses to have his/her/it's existence proven.
Sorry - no experiment to prove the existence of God is possible. SteveBaker (talk) 15:57, 26 September 2009 (UTC)[reply]
Steve, Funnily you are obviously right in outcome given that your logic is all over the place. Can you conceive of an experiment to demonstrate your own existence? --BozMo talk 18:05, 26 September 2009 (UTC)[reply]
Doubting everything is an absurdist position. You can jump out of an airplane and see what I mean. Rational people can agree on some things while testing more controversial subjects. Imagine Reason (talk) 19:19, 26 September 2009 (UTC)[reply]
I'm inclined to agree with SteveBaker that no experiment to prove (or disprove) the existence of God is possible. For a very simple reason: God will not allow it. You come to God through faith, not through proof. In fact, if it were possible to prove it (or disprove it) there would be no debate about it, now would there? Or very little. You can easily demonstrate that the earth is round. There are still a few who may cling to the notion that the earth is flat, but they have little or no influence in society. You cannot prove religion true or false. Hence the debate continues endlessly. →Baseball Bugs What's up, Doc? carrots 19:37, 26 September 2009 (UTC)[reply]
That's ridiculous! You are saying that no proof is possible by assuming the answer that the proof is attempting to resolve! You are not thinking about falsifiability - which lies at the heart of the scientific method. SteveBaker (talk) 20:42, 26 September 2009 (UTC)[reply]
I'm agreeing with you and you're calling it ridiculous? Hard to know where to go from here. :\ →Baseball Bugs What's up, Doc? carrots 00:51, 27 September 2009 (UTC)[reply]
You've reached the same conclusion - but for entirely the wrong reason - that's not the same thing as agreeing with me. SteveBaker (talk) 01:12, 27 September 2009 (UTC)[reply]
Baseball Bugs you claim as facts "God will not allow it. You come to God through faith.." which is the prejudgement that SteveBaker resists. Cuddlyable3 (talk) 13:22, 27 September 2009 (UTC)[reply]
I assume we're trying to have a rational discussion here. If you're throwing logic out the window when God is involved, then I think we can say that science and religion cannot mix. Imagine Reason (talk) 00:40, 27 September 2009 (UTC)[reply]
I have to disagree with Steve's description of the scientific method. Scientists don't try to prove their theories, they try to disprove them. Proving them is completely impossible - you can show it was right that time, you can't show it will always be right. You come up with a hypothesis, you then come up with an experiment that could prove it wrong and try it out. If, after lots of such experiments, you still haven't managed to disprove you, you assume it it true. --Tango (talk) 22:45, 26 September 2009 (UTC)[reply]
Technically, that's true - but you only have to put the word "NOT" into your hypothesis and you can go off and prove your original idea rather than having to disprove it. So instead of having a hard time proving the hypothesis that there is a god - you can have an equally hard time DISproving the hypothesis that there is NOT a god. The result is the same...although it sounds much better your way! SteveBaker (talk) 01:38, 27 September 2009 (UTC)[reply]
I think I !agree with you... ;) I think a distinction needs to be drawn between a theory as a whole and it's various parts and their implications. Logically, that distinction just boils down to applying basic rules for combining ANDs, ORs and NOTs to get from one to the other, so is pretty moot, but conceptually the distinction is more significant. The thing you try and disprove isn't the entire negation, but rather one part of it or an implication of one part of it, which is usually a far simpler statement, which makes it easier to test. I wouldn't know where to start trying to prove the general theory of relativity, but I know exactly how to try and disprove the hypothesis that when I hold a pencil above the ground and then let go of it it will fall down. --Tango (talk) 07:01, 27 September 2009 (UTC)[reply]
On the scientific method Tango I think you need to read someone other than Popper, Thomas Samuel Kuhn for example. Falsifiability is only partly right, although it is part of the story. But I am a bit stuck on why you (Steve) think the existence of God is an empirically testable thing when apparently you concede that your own existence could not be established against the null hypothesis that you are just a chemical phenomenon. A guy called Hans Frei wrote a famous book in 1974 which pointed out that the perversion of Christianity from human meaning to (blatantly wrong) pseudoscience was post enlightenment and mainly nineteenth century. So some enthusiastic early scientists savaged religious belief into bad science. That does not mean the religious belief was not correct in its own right. Since at least 1963 (Honest to God) mainstream Christians have agreed that in a sense God does not exist (nothing in the universe which can be prodded with a pencil is God). Discussion on testability and Popper's view of science belongs in the fourth form where it is taught. --BozMo talk 07:17, 27 September 2009 (UTC)[reply]
The comment makes unfounded assumptions regarding my reading, my thoughts and my education, none of which is true. --Tango (talk) 07:38, 27 September 2009 (UTC)[reply]
Fair enough, apologies I mixed up some of Steve's comments with yours (but it is a fair risk in this kind of mele). --BozMo talk 07:41, 27 September 2009 (UTC)[reply]
I don't think Steve thinks the existence of god(s) is testable any more than I do... --Tango (talk) 08:33, 27 September 2009 (UTC)[reply]
Yeah - I went to a lot of trouble to prove that the existence of an omnipotent god is impossible to test. I have never claimed that my own existence could not be proved or disproved. To the contrary - I think it's a relatively simple experiment with perfectly acceptable true and false outcomes. Assuming you define "me" as a human, on earth - then the experimenter merely needs to enumerate all humans on earth - examine each one - and find if one of them meets the criteria laid down in the definition of "SteveBaker". There is nothing unfalsifiable there. Sure, there are experimental details to iron out - and it's a tough experiment - but it's definitely do-able in principle. SteveBaker (talk) 15:26, 27 September 2009 (UTC)[reply]
I don't know if you are being deliberately obtuse for irony/fun or really don't get the question. I cannot tell with you. Do you think that Human Beings are in any way capable of acting in a way not in line with known physical laws, or do you think they are entirely determined? If you thought they have a soul which disobeys quantum chemistry you would be a fruitcake which I do not believe for a second. But if you think they are completely determined why on earth do you persist in believing in the myth of "humans "rather than just view the earth as a set of simultaneous partial differential equations? Is not "humanity" and every part of that description an unneeded hypothesis to understand the world? (This is intended as a Reduction ad absurdum of course; the absurdity being that "explanation not needed" is relevant to the existence of beings) --BozMo talk 18:50, 27 September 2009 (UTC)[reply]
Humans are part of the universe, I don't see a contradiction there. I expect human behaviour can be modelled mathematically (that there are laws of physics that everything obeys is the fundamental assumption of science, and has held up pretty well so far), although we don't yet have the appropriate models to do it with any precision and, even if we did, measuring the initial conditions would be very difficult without killing the human you are interested in (and probably even then). --Tango (talk) 19:51, 27 September 2009 (UTC)[reply]
I'll break up your somewhat confusing post into chunks for the sake of making a coherent reply:
  • Do you think that Human Beings are in any way capable of acting in a way not in line with known physical laws, - No, there is no evidence that any part of a human being is anything other than 'normal matter' - so physical laws apply.
  • ...or do you think they are entirely determined? - That's not an 'either/or' question. The laws of physics do not imply determinism. Quantum theory and Chaos theory both imply a fundamental randomness in all matter that means that there is no determinism. But that applies to a lump of rock just as much as it does to a human being. However, I'm pretty sure that's not what you're trying to get at here. You are probably asking about "free will" - and that's an interesting question. It's my opinion that there is no free will - BUT that our brains are wired such as to give our conscious mind the illusion that there is free will. That wiring is so firmly emplaced that it is impossible to cease to exercise "free will" even if we know at an intellectual level that this is simply a consequence of interactions of atoms and forces that are 'mechanical' in nature.
  • If you thought they have a soul which disobeys quantum chemistry you would be a fruitcake which I do not believe for a second. - Indeed, disobeying quantum theory is not likely.
  • But if you think they are completely determined why on earth do you persist in believing in the myth of "humans "rather than just view the earth as a set of simultaneous partial differential equations? - Again, you are supposing these are opposing views, when they are not. If I believed that humans were completely determined (which is a fair approximation at the macro-scale) that would not prevent me from viewing the universe at a range of convenient scales. At the scale of atoms, talking about human beings is very tough...but at the scale of objects larger than a fraction of a millimeter, using the word "Human Being" to summarize an insane amount of math/physics/chemistry is a handy convenience. We can simultaneously hold mental models of the world at various scales - and at some scales "Human Being" is a convenient short-hand for a complex phenomenon.
  • Is not "humanity" and every part of that description an unneeded hypothesis to understand the world? - Well, in theory, you could describe 'the world' without this degree of abstraction - some very large pile of quantum-theoretic equations is a sufficient description. It's just not very convenient. This is no different than that we use words like "City" and "Nation" as a way to summarize the details of large numbers of humans.
  • (This is intended as a Reduction ad absurdum of course; the absurdity being that "explanation not needed" is relevant to the existence of beings) - I don't see the absurdity. I think you are trying to make an argument from a set of assumptions of how people have to think - that simply does not apply to most people. You can take the phrase "Human Being" and make it inapplicable. If I take a sample of your blood - is that blood "Human"? If I take a hydrogen atom from your big toe - is that atom "Human"? How could you tell? These questions are only tricky if you insist on embodying words with solid meanings. Words like "human" are merely a convenient shorthand for a complex concept. Something is only "human" if we decide that that set of symbols on the page apply to it. That consideration tells you a great deal about the applicability of those symbols - but it tells you nothing whatever about what a human being is.
SteveBaker (talk) 20:13, 27 September 2009 (UTC)[reply]
This helps me to understand your position. thanks. So we are heading toward "Human being" being a useful interpretation of a complex phenomenon which is fine because as a scientist you are not an uncompromising reductionist, you regard parallel descriptions as allowed and insist only on consistency, not on primacy of one description, yes? --BozMo talk 20:40, 27 September 2009 (UTC)[reply]
I wouldn't say it was an interpretation of the phenomenon, it's just a name for it. It is essential in day-to-day science to accept certain things as given without worrying about how they work, for example chemists rarely worry about the nature of the nucleus of an atom because it isn't important for what they do. Similarly, psychologists don't usually care about the nature of neurons, etc., because it isn't important for what they do, and neurologists don't care about the finer points of biochemistry, and so on. I wouldn't say those are parallel descriptions, they are just useful approximations to the actual description. --Tango (talk) 23:42, 27 September 2009 (UTC)[reply]
Way back in one of my earliest responses to the OP, I mentioned the null hypothesis - and testing it with a control experiment - that's an important part of the scientific method. This is exactly what you guys are talking about when you're discussing the negations and logical inversions of the original hypothesis. A formal method for such analysis does exist. Nimur (talk) 07:08, 27 September 2009 (UTC)[reply]
Absolutely. When we want to prove a hypothesis we often actually do so by disproving the null hypothesis. That is, indeed, exactly what we are talking about. We aren't really discussing the scientific method, on which I am quite sure we agree, but rather the best way to describe it. --Tango (talk) 07:38, 27 September 2009 (UTC)[reply]

This discussion seems to postulate, and be limited to, the God of Abraham. To really do it right, the experiment needs to go beyond monotheism and also look into large polytheistic religions, the most obvious of which is Hinduism. There's no evidence that any one religion has the "right" definition of God, or the "whole picture" of God. Each religion tries to form a concept of what God is, in terms they can sort-of understand. There's a lot more similarity between Hinduism and Catholicism, for example, than either religion would likely be willing to admit. →Baseball Bugs What's up, Doc? carrots 16:02, 26 September 2009 (UTC)[reply]

No - you're quite wrong. It doesn't matter - it doesn't change the argument one iota. If your definition of a "god" allows for infinite powers - then there is no possible unfalsifiable experiment and science cannot prove or disprove the existence of any "god" by that definition. It doesn't matter which religion or whether the god has six arms or lots of tentacles or is inexplicably fond of ham and pineapple pizza (mhhbb). The rules of scientific debate are the same.
Of course if you have a definition for some kind of "god" that has some solid limitations on the god's power - then perhaps there is something that can be done. But there aren't many religions where that truly is the case.
If you had (for example) a "river god" that was constrained to living in a river - and by whose divine presence alone caused that river to (say) have life-extending properties - the fountain of youth or whatever. Then you could perform a statistical study of people who drank from every river on the planet - find the one with the life-giving properties - perform extensive chemical analysis of the water - discover no other possible means for the water to provide their unexpected vigor - and thereby deduce the existence of a river god living there. This experiment has the countervailing proposition that if the water from NONE of rivers on Earth appear to have any special life-giving properties - then there cannot possibly be a river god. That's a valid experiment and would solidly prove or disprove the hypothesis.
But when you have a god who has totally unlimited powers - he/she might choose NOT to live in the river - or NOT to provide life-giving properties to the water - or alter the minds of the researchers to make them think the results came out false. Then you're back to unfalsifiability again - and you don't have a solid scientific hypothesis because it has zero predictive power. SteveBaker (talk) 16:27, 26 September 2009 (UTC)[reply]

(I moved this post to the end of my response: Please do not intersperse your comments between paragraphs of other peoples' posts - it's very rude.) SteveBaker (talk) 20:38, 26 September 2009 (UTC)[reply]

I can. Omniscience and omnipotence cannot coexist, because a god that doesn't know what he'll do is not omniscient, but one who know everything he'll ever do cannot change the future and is thus not omnipotent. Imagine Reason (talk) 19:19, 26 September 2009 (UTC)[reply]
That's nonsense. If you are literally omnipotent - you can do literally anything - being everywhere at once - and even everywhere in time at once is just a part of having literally unlimited abilities. It is rare indeed to find a religion that allows for hard limitations in the abilities of their gods. Your argument is akin to "Can god create a rock so heavy that he can't lift it?" - there are indeed paradoxes inherent in postulating literally infinite capabilities. The ability to perfectly see the future and the ability to change the future are also incompatible paradoxes - but if you believe in all this, then paradoxes go with the territory. If you wish to claim that the capabilities of god(s) have to be limited in some manner in order to avoid paradoxes - then you have some serious religious issues to deal with! SteveBaker (talk) 20:38, 26 September 2009 (UTC)[reply]
You don't solve paradoxes by claiming without basis that something--e.g., your god--is not limited by it. Imagine Reason (talk) 17:25, 28 September 2009 (UTC)[reply]
This is why religion and science don't mix. To someone who believes in a personal God, an atheist can make logical arguments all day and it won't make any difference, because those arguments run counter to the believer's own experience and faith. You can logically argue that God is not needed for the existence of the universe, but that does not prove the non-existence of God (or gods). And it comes back to the core question: Who or what is God? And that's the unanswerable question. The best you can do with such an experiment is to say, "IF this is what God is, THEN this-or-that hypothesis about God may be true (or false)." →Baseball Bugs What's up, Doc? carrots 16:41, 26 September 2009 (UTC)[reply]
Our OP is specifically asking about ways to prove the existence of god using scientific principles. Such debate is equally valid for atheists and theists because if you are doing this right, you have to start out with an open mind and let the results of your experiment determine what you believe. This thread is not about whether god or gods exist or not - it's about whether you can use scientific principles to prove that. Logical arguments are at the core of this question because without such arguments, no scientific proof can even be considered. Sadly, (as I hope I have shown) it is conclusively impossible to come up with such an experiment. SteveBaker (talk) 20:38, 26 September 2009 (UTC)[reply]
And not all conceptions of god involve omnipotence. Suppose somebody worships an idol, or all instances of a particular species of animal - in that case, it is easy to prove that god exists. Clearly, you only need to observe the thing which they define as their god. A perfectly acceptable religion might be one which worships a pet cat, which can absolutely be shown to exist. Do you want to say their religion is wrong? You can call it a stupid religion, but that would be nothing but insulting to its believers. Actually, it's a much more straightforward religion than most. And, it's easy to prove that its god does actually exist. "Do as the cat does. Eat, sleep, purr." I don't know why gods or religions necessarily have to be abstract or supernatural concepts - that's really only an aspect of judeo-christian or buddhism/hindu/dharmic religions and their particular choice of definition for "divine". Nimur (talk) 18:16, 26 September 2009 (UTC)[reply]
Not all religions claim omnipotence - but most do. The Catholic church for example, in their "Catechism" state this quite clearly: "We believe that his might is universal, for God who created everything also rules everything and can do everything."[2]...so the Catholic god (at least) is unfalsifiable. If our OP is thinking of some other god with dramatically smaller powers - then perhaps there are experiments that could be devised. Odin - in Norse mythology is a god - with extensive powers - yet at Ragnarok he will have to fight Fenrir and will die. Evidently, he can't just snap his fingers and have Fenrir disappear - this god evidently has limits - and perhaps we could think of a falsifiable experiment to prove (or disprove) the existence of Odin...but definitely not the Catholic god. That's not because the Catholic god is in any way more "real" - to the contrary - scientifically speaking, unfalsifiable things are the subject of Occam's razor and are generally considered to be less "real" than things like string theory that are theoretically falsifiable. SteveBaker (talk) 20:38, 26 September 2009 (UTC)[reply]
Once you start discussing the gods of particular religions it becomes more complicated - what, precisely, is "the Catholic god"? Is it just a being with the powers described in the Bible (in which case, you are right) or is it a being with those powers that has done the things described in the Bible? In the latter case, we can find falsifiable claims. For example, the Bible says (indirectly, but quite clearly) that God created about 6000 years ago (I know most Catholics choose to ignore that bit, their inconsistency is their problem), that is verifiably false. Unless we assume God is intentionally trying to trick us (which is within his power and not is kind of consistent with his behaviour described in the Bible - he does like to test people's faith, but such trickery isn't described in the Bible [which would kind of defeat the object, I guess]), then we have a falsifiable (and false) statement about the Catholic god. If you allow omnipotence and lies of omission in divinely inspired texts, then it is completely unfalsifiable, but get rid of either of them and we're ok. --Tango (talk) 01:28, 27 September 2009 (UTC)[reply]
I picked the catholics for a very specific reason. Of all religions, they are the most careful to document their terms - to define everything very carefully. The document I linked to that says that the Catholic God is omnipotent is right there on the Vatican web site - it's easy to find, it's written in clear english, it's unambiguous. What, precisely, is "the Catholic god"? It's defined by whatever the church and the pope says it is...you can just read the catechisms - and then you know. It's not the powers defined in the bible - it's clear and unambiguous. Of course some individual might choose to ignore or disbelieve parts of the catechism - but then they are denying a part of the churches teachings and they aren't a proper catholic (as defined by the church themselves). It's not so easy in other religions - but still - almost none of them are prepared to accept limitations of any kind on the powers of their gods. That's probably because there has been a kind of religious "arms race" between gods over human history. Each early religion needed to show that their "rain god" could kick the butt of the next tribe's "mountain god" - so the list of claimed powers grows over time. Nobody would want to admit that their god could not under any circumstances save your sick child or prevent your crops from failing. So the religions that survived this process are the ones which claim complete omnipotence for their deity. They rail at questions like "Could your god create a rock so heavy that he couldn't lift it?" - giving all sorts of wishy-washy answers.
Your example of the creation of the earth (6000 years ago) is NOT falsifiable. We do radio-carbon dating of objects and PROVE that they are more than 6000 years old - and the nut jobs say "Well, God wanted to challenge your faith - so he deliberately created objects with the right amount of radio-carbon in them to make your experiments come out that way."...thereby destroying falsifiability. That's the problem at the heart of this. Unless you also limit your god's powers such that he can't create objects with controlled amounts of radiocarbon in them (and so that he can't influence the results of a mass-spectrometer - and so he can't alter the minds of the experimenter - and so he can't cause their results to be published with numerous misprints)...you can't falsify that claim.
Claiming that "God wouldn't seek to trick us" is a valid option - but it has to be written into the hypothesis: "A god is a being that has infinite powers who absolutely guarantees that he will never use them in the following ways: ....<long list>". That imposes limits on the infinite power of god - and most religions simply will not accept that. I often argue with the more rabid religious nuts that they have no way to know that the god they believe in isn't cheating on them. Perhaps he's lying to us all the time? How do we KNOW that "the devil" isn't actually the good guy? Some of the things that god traditionally demands that we don't do - but the devil supposedly wants us to do - are actually rather cool. If we can't covet our neighbors oxen - how will we have aspirations to improve ourselves? SteveBaker (talk) 15:26, 27 September 2009 (UTC)[reply]
Another example of what you're describing would be The Church of Baseball. And it has the advantage that it's not as likely to die as your pet cat. →Baseball Bugs What's up, Doc? carrots 18:43, 26 September 2009 (UTC)[reply]
AHA! The dimly-lit bulb finally comes on. You worship your cat. But it dies. You can't worship your cat anymore, right? Yes. You can. You can say its spirit continues to live and continues to watch over you... and that it will return someday. And there, in a nutshell, is the core premise of a number of religions. →Baseball Bugs What's up, Doc? carrots 19:32, 26 September 2009 (UTC)[reply]
Just to raise another common theological point: if you could prove God existed, what would be the point of faith? It's only faith because the evidence isn't really very concrete. No one needs faith in, say, electricity. To say you believe in electricity is not very much of a demonstration of personal commitment. --Mr.98 (talk) 20:14, 26 September 2009 (UTC)[reply]
If you want to "demonstrate personal commitment" then go out and do good works. If you believe in electricity then go make the world a better place with electricity. That's the point of faith. Faith divorced from works is meaningless. You can find that sentiment in the Bible, but it's no more a religious sentiment than "don't commit adultery". Faith, like morality, is a secular concept that's been co-opted by religion. If you think it matters whether God exists then you're missing the point. What matters is that he ain't solving our problems for us. That much is empirically obvious. -- BenRG (talk) 22:15, 26 September 2009 (UTC)[reply]
That's the "old man on a cloud" theory, i.e. some kind of direct interventionist. God works through people. So does Satan. That's the concept, anyway. Maybe you haven't heard the story about the guy on a rooftop during a flood? →Baseball Bugs What's up, Doc? carrots 22:36, 26 September 2009 (UTC)[reply]

Please attend closer to the OP's posting. It is not relevant to discuss Catholicism, Polytheism, Hinduism, a river god, idol worship, pet cats, church of baseball (have I missed anyone?) when the OP's explicit standpoint is "I am Protestant". Whatever lies behind that declaration must provide all the evidence the OP will probably ever find. We can't know whether that declaration is the result of a personal epiphany (apparently not), social conformity or an upbringing that the OP is now questioning. An answer to the OP that is appropriately couched in terms relevant to Protestantism is in the quoted words of apostle Thomas: Unless I see in his hands the print of the nails and stick my finger into the print of the nails and stick my hand into his side, I will certainly not believe[the resurrection]. Protestants frown on such objectivity when they quote "Blessed are those who do not see and yet believe." - John 20:24-29. Therein lies the OP's quandary. I think the OP wants that blessing promised by christianity and that the Ref. Desk. have no clue how to prove that it exists for him. Cuddlyable3 (talk) 22:49, 26 September 2009 (UTC)[reply]

The OP never defined what "protestant" means, or how he interprets it. Needless to say, that word can run the gamut from atheism to biblical literalism - many people describe many different beliefs under the single umbrella term of "protestantism". I think it has been relevant to discuss how different interpretations of god will yield a different degree of falsifiability. The easiest are the biblical literalists - they make factual claims that can be shown to be contradictory. So, for their definition of god, it is very easy to disprove much of their faith (whether they want to accept this fact or not). On the other hand, as the interpretations become allegorical and vague, it becomes harder and harder to prove or disprove anything. Nimur (talk) 01:39, 27 September 2009 (UTC)[reply]
Protestantism with a capital "P" means Lutheranism and though much fragmented by schisms since Martin Luther (1483-1546), it consistently relies on the Bible as source and can not encompass atheism. Cuddlyable3 (talk) 13:13, 27 September 2009 (UTC)[reply]
That is how you define it. By virtue of the fact that protestants reject external doctrine, they are free to make up their own definitions for their theological ideas. I know many self-identifying protestant atheists. Do you want to tell them they are not allowed to be protestants because they aren't Lutheran? Nimur (talk) 15:32, 27 September 2009 (UTC)[reply]
Well, I wouldn't get that far - I would tell them they aren't protestants (it's not a matter of being allowed or not, it's just a fact) because they aren't Christian. Protestantism is a branch of Christianity, which involves believing in God and Christ in some form. --Tango (talk) 19:59, 27 September 2009 (UTC)[reply]
Nimur, the way I define Protestantism is the way that history and Wikipedia define it. Is it possible that your "self-identifying protestant atheist" friends are as confused as people who say they don't believe in God and do believe in Jesus ? The foregoing two wikilinks may enlighten them.Cuddlyable3 (talk) 15:20, 28 September 2009 (UTC)[reply]
Then the answer is easy: No, you can't prove God exists. And you can't prove God does not exist. Does that about cover it? →Baseball Bugs What's up, Doc? carrots 00:49, 27 September 2009 (UTC)[reply]
Protestantism says that god chooses not to use all of his powers - preferring to work by coercion of people - but the religion doesn't go so far as to say that he isn't omnipotent or even that he won't ever use these powers. The church proclaims truth of the bible - so all of the various miracles described there are held to have literally happened. Ergo, you're still talking about an omnipotent being - and falsifiability is a problem. Hence, our OP cannot devise a suitable experiment to test that hypothesis. SteveBaker (talk) 01:38, 27 September 2009 (UTC)[reply]
Yes, Protestantism encourages the concept of free will, with God supposedly taking a laissez faire approach to a certain extent (though not to the pretty much totally hands-off concept of Deism). As far as God's powers, I would say Protestants go even farther than Catholics, who portray the various saints as being kind of an extension of the Church's earthly hierarchy, and Catholics pray to "specialist" saints (vaguely analogous to Hinduism's polytheism) apparently on the grounds that God is "too busy". Protestantism, however, maintains that God is omniscient and omnipresent, therefore there is no need for this "saint" stuff. But since God apparently restrains His powers to allow free will, that does kind of throw a spanner into the experimental thesis, yes? Because it means that sometimes God intervenes and sometimes He doesn't, so how could you tell? P.S. I have the answer to that question, if anyone's curious. :) →Baseball Bugs What's up, Doc? carrots 04:00, 27 September 2009 (UTC)[reply]
Depends on the sort of Protestantism. One of the most dominant strains, Calvinism, has in theory no room at all for the concept of free will — even our sins are God's determined will, though that doesn't keep them from being our fault. There is nothing logically wrong with this; it just doesn't seem fair. But for the Calvinist, I suppose it's our concept of "fair" that's defective here.
Of course in practice few of the faithful really worry about this. It's extremely difficult to tell the difference between a Methodist and a Presbyterian, even though in theory the former should believe in free will and the latter should not. --Trovatore (talk) 21:06, 27 September 2009 (UTC)[reply]
To Baseball Bugs: I am curious. How could I tell? Cuddlyable3 (talk) 13:28, 27 September 2009 (UTC)[reply]
If God intervenes on whim, then there's no way to prove or disprove certain religious tenets, such as the so-called "power of prayer" - because if person A prays for something and doesn't get it, while person B prays for something and does get it, does that demonstrate that apparent help from God is merely random chance? Or does it merely demonstrate that God is whimsical? And if God is whimsical, then you can't apply science, because you can't predict what He will do. →Baseball Bugs What's up, Doc? carrots 06:17, 29 September 2009 (UTC)[reply]
There would be no point in worshipping something like that. Zain Ebrahim (talk) 07:31, 29 September 2009 (UTC)[reply]
Religion is more than just being "about God" or gods. It's also a cohesive force in the community, something that's shared. The usual answer to the apparent whimsy of God is that "He works in mysterious ways" or "sometimes the answer is No". Both of these apparently contradict the "ask and ye shall receive" promise. Maybe the problem is that it's "the wrong question." Because if you ask for spiritual strength, and are open to it, then the answer is always "yes". →Baseball Bugs What's up, Doc? carrots 13:15, 29 September 2009 (UTC)[reply]
God is not a vending machine. If he was required to grant any specific request, it would be a challenge to his omnipotence as he would not have the power to deny the request. Googlemeister (talk) 18:33, 29 September 2009 (UTC)[reply]

This link may be relevant

Voltage

Hello. How is there a positive voltage between one end of exposed copper strip and another end of copper covered in paper towel soaked with zinc sulfate solution? If Cu and Zn2+ were to oxidize and reduce respectively, the redox reaction is not spontaneous. Thanks in advance. --Mayfare (talk) 14:52, 26 September 2009 (UTC)[reply]

Is this to do with confusion between electric potential - which exists in the absence of any current, and voltage measured in a complete circuit - what you describe isn't a completed electrical circuit. ? .83.100.251.196 (talk) 17:30, 26 September 2009 (UTC)[reply]
Hang on a minute.. there would be a voltage difference if zinc wire was in copper sulphate solution at one end.
There is (practically) no reaction between Zn2+ and Cu. So no voltage. ? (If it's your teacher again that told you this - is it possible that you misheard?)83.100.251.196 (talk) 17:32, 26 September 2009 (UTC)[reply]
The article Battery (electricity) will help.Cuddlyable3 (talk) 22:56, 26 September 2009 (UTC)[reply]

New AIDS Vaccine

Reports indicate the new vaccine reduces the chances of getting AIDS by 30 percent. If the clinical trials are typical, then the researchers hired a bunch of healthy people and had a control group that didn't get the experimental vaccine. They attempted to infect both groups with HIV and measured results accordingly. My question is: who in their right mind would volunteer for such a study? —Preceding unsigned comment added by 76.229.89.46 (talk)

Perhaps prisoners who are under a life sentence and feel they have nothing to lose? →Baseball Bugs What's up, Doc? carrots 15:52, 26 September 2009 (UTC)[reply]
If you have a large enough group - you don't need to deliberately attempt to infect them (which would, of course, be unethical). If you picked a large number of people in an already "at risk" group (intravenous drug users, gay men, people living in rural Africa, etc) then enough of them would get an HIV infection "normally" to make the experiment valid without exposing them to any more risk than they were already prepared to accept. However, you do need a much larger group that way because you need a large enough sample to be able to assume that the exposure of both the vaccinated and control groups is about the same. But at least you're not doing anything unethical in the process. SteveBaker (talk) 16:11, 26 September 2009 (UTC)[reply]
I'm not suggesting forcing prisoners to do it. However, what's needed here would be an answer to the question of how any vaccine is tested ethically. The answer to that general question would likely supply the answer to this specific one. →Baseball Bugs What's up, Doc? carrots 16:16, 26 September 2009 (UTC)[reply]

It is a different scenario than other vaccines, where there is no huge difference between risky and safe behavior. How can the testing work at all? The scientists give people a shot and tell them: "now, just go around doing whatever you know, but don't bother too much about protection, so we can know whether this vaccine is working and not. After that, we can publish our work in Science or Nature and even get a Wikipedia article." And, what does happen if the test persons start to ask what this whole AIDS thing is all about and take precaution against it?--Quest09 (talk) 16:34, 26 September 2009 (UTC)[reply]

If you tell both groups the same thing, it doesn't matter much what you say, you just look for statistical differences between the groups' results. As you note "do not practice safer sex" is unethical. But in a large at-risk population, there are enough who wouldn't without substantial advice to do so (if at all), and the rates of condom use (and mis/non-use) in both groups is the same, so that would not affect the difference of HIV/AIDS rates between the groups. DMacks (talk) 16:48, 26 September 2009 (UTC)[reply]
Let's be very clear about this: they did not not not deliberately try to infect anybody. In fact they went out of their way to make sure that all the subjects were thoroughly informed on how to avoid getting HIV. To even further reduce the chance of a conflict of interest, the advice was given by an independent medical team who were not directly involved with the scientists conducting the study. The assertions in this section are pretty damn close to BLP violations, even considering this is a non-mainspace page. Looie496 (talk) 17:23, 26 September 2009 (UTC)[reply]
Furthermore, if you think about events such as the HIV trial in Libya, you might realize that loose talk like this is not harmless, it can have deadly consequences for the people involved. Looie496 (talk) 17:39, 26 September 2009 (UTC)[reply]
As yet another followup, I want to clarify that the last comment was aimed at everybody who has written in this section, not specifically at DMacks. Looie496 (talk) 18:22, 26 September 2009 (UTC)[reply]
Gee - thanks! How the heck does discussing scientific approaches to doing vaccine trials have anything whatever to do with WP:BLP? Please go back and re-read my post - then apologize for such a vile and unwarranted accusation. SteveBaker (talk) 20:11, 26 September 2009 (UTC)[reply]
I think Looie496's point is not so much that WP:BLP policy is violated here but that the ethics of any HIV/AIDS trials are liable to be perceived differently in societies where researchers have to work, and that is a reason to be circumspect to avoid giving ammunition to viewpoints hostile to their work. Looie496 aimed his caution to everybody but I don't see any accusation of loose talk made specifically to SteveBaker's post. SteveBaker could rightly say that such an accusation would be unwarranted if it had been made.Cuddlyable3 (talk) 23:24, 26 September 2009 (UTC)[reply]
Actually I do apologize to Steve and to DMacks, neither of whom made incorrect statements about how the experiment was done. The BLP issue in this thread is that the original question implicitly accused the people who carried out the experiment of doing something that would have been totally unethical, and other comments assumed a lesser level of unethical behavior. There are far too many people in this world who will jump to the conclusion that scientists are evil given the slightest reason; we have to be careful not to feed into that when there is no basis for it. Looie496 (talk) 02:34, 27 September 2009 (UTC)[reply]
  • Actually, I didn't accused any one of being unethical. I was - under the supposition that there must be a difference - asking how an AIDS vaccine could be tested ethically. Furthermore, I have to point out that if someone else jumps to conclusion due to my words that is enterily their mistake and problem. This fact cannot restrict my liberty. --Quest09 (talk) 17:25, 27 September 2009 (UTC)[reply]
I'm pretty sure Looie496 was referring to the OP i.e. 76.229.89.46 who did say "They attempted to infect both groups with HIV and measured results accordingly" which is an extremely offensive accusation of the people involved even if it was out of pure ignorance rather then malice. I presume this is not you. BLP does cover all pages and accusing a specific even if unnamed group of researchers of intentionally infecting people with HIV (or for that matter doing anything unethical) is a problem BLP wise. By editing wikipedia, you do agree to obey wikipedia policies no matter how they 'restrict your liberty', if you don't accept that then I respectfully suggest you don't edit wikipedia. In general terms it's best to not suggest what people are doing if you don't know. Your answer does appear to have inadvertently presumed that the people would behave unethically. E.g. "And, what does happen if the test persons start to ask what this whole AIDS thing is all about and take precaution against it". Et al. This is rather offensive since it suggests the researchers are simply testing these poor ignorant people and hoping they don't ask hard questions. Modern research does not work like that. Nor is this something unique to HIV. If you are testing a drug to lower cholesterol or reduce the risk of a MI you either inform your patients yourself or use patient who are already well aware of dietary and other lifestyle modifications they should be doing. You don't collect a bunch of people and keep them in the dark about other ways to try and improve their conditions and test the drug on these poor saps. Nil Einne (talk) 18:52, 28 September 2009 (UTC)[reply]
  • My answer just presumes that they went through a different way than the testing of other vaccines. It is not offensive in any way and I was not breaking any Wikipedia policies. I just said that it is not my problem if other people would jump to conclusions. There is not Wikipedia policy against that, is there? —Preceding unsigned comment added by Quest09 (talkcontribs) 15:13, 29 September 2009 (UTC)[reply]

What I want to know is where did the idea that deliberate infections are used to test vaccines come from? As far as I'm aware few vaccines are tested by deliberate infection of the subjects with the virulent agent being tested against after vaccination. Even Edward Jenner#Smallpox who's methods are often considered not entirely ethical nowadays used variolation to test his vaccines, which while a deliberate infection of sorts was an existing practice intended to provide immunity via a less severe form of the disease. (In fact as section d mentions, it wasn't necessarily the best methodology since it's possible it wouldn't have protected against natural infections.) AFAIK, even influenza vaccines often don't use deliberate infections, for example the way this [3] recommends deliberate infections as a testing methodology makes it sound like it is uncommon presently and this [4] says "Malaria is unusual in that the efficacy of vaccines can be tested in small numbers of individuals by deliberate infection with Plasmodium falciparum". In any case as others above have mentioned and this [5] mentions, deliberate infection is not used with highly dangerous pathogens. Edit: We do have a vaccine trial but it doesn't have much useful information Nil Einne (talk) 08:55, 27 September 2009 (UTC)[reply]

Mining in volcanically active areas

How does mining look like in volcanically active areas? I had taken a look at articles like mining and mining in Japan, but didn't learn anything about the effects of tectonic and volcanic activity.--Mátyás (talk) 18:32, 26 September 2009 (UTC)[reply]

It's drilling, not mining, but check out Sidoarjo mud flow for what can go wrong. --Sean 15:32, 28 September 2009 (UTC)[reply]
There are a few minor differences (for example, groundwater can be hot instead of cold), but mostly it's just like mining anywhere else. --Carnildo (talk) 00:15, 30 September 2009 (UTC)[reply]

Food poisoning from protein-less foods

How might it occur? The only scenario I can imagine is harmful bacteria eating up nonharmful ones. Imagine Reason (talk) 19:15, 26 September 2009 (UTC)[reply]

How many foods are actually protein-less? Are there not proteins in most things we eat, many of which humans can't digest?--Leon (talk) 19:19, 26 September 2009 (UTC)[reply]
I don't think there are any whole foods with no protein at all. Fruit tends to have a lower fraction of protein on a per-calorie basis than pretty much any other category of whole food, but even fruit has some protein. You can, however, have proteinless refined food products, such as sugar or oil. I don't see how you could get food poisoning from sugar. Oil goes rancid, but I'm not sure eating rancid oil counts as food poisoning. Red Act (talk) 19:43, 26 September 2009 (UTC)[reply]
Well, though it is not possible to suffer from food poisoning per se from lack of proteins (food poisoning is caused by pathogens), it may be possible to experience adverse effects from lack of protein in the diet. There are 22 standard amino acids which compose almost all, if not all, polypeptides. Humans can biosynthesize fourteen of these; the other eight (called essential amino acids), must be obtained by from food. This is done by breaking down existing proteins in food and converting them into amino acids. Intelligentsium 00:31, 27 September 2009 (UTC)[reply]
Yeah, but malnutrition is rather different from food poisoning. Red Act (talk) 01:37, 27 September 2009 (UTC)[reply]
Food poisoning doesn't really have any direct relationship to protein at all. Most proteins are broken down in the stomach and never make it into the bloodstream in appreciable quantities; the few that survive the stomach (such as keratin) are indigestible. Many venoms -- injected into the bloodstream by a bite or sting -- are protein-based, but toxins absorbed through the digestive system are chemically different. Looie496 (talk) 17:59, 27 September 2009 (UTC)[reply]
I think the OP was thinking about food poisoning having an indirect relationship to protein, i.e., thinking that the pathogens that produce food poisoning would need a source of protein in order to survive and multiply. However, I don't think that's the case (although biology is not my strong suit). For example, I had been thinking you probably couldn't get food poisoning from sugar. But left to sit long enough, mold will grow in an opened bottle of soda, in which the only available nutrients are sugar and water. So it does seem like you could get food poisoning from drinking a (proteinless) old opened bottle of soda, that you didn't notice that there was mold growing in. Red Act (talk) 18:43, 27 September 2009 (UTC)[reply]
But without know what species of mold could grow in those conditions, you will not know if those molds would be toxic to people. Actually, being toxic is not the same thing as food poisoning is it? Googlemeister (talk) 14:01, 28 September 2009 (UTC)[reply]


September 27

Question about the article "Psychological immune system"

According to this article, each persons view of the world, or reality, actually consists of a set of lies and biases, and that in order to keep our ego healthy, we continue to view reality through biased eyes whenever we are confronted with adverse situations, that could potentially cause our ego harm. Is my interpretation correct?

What really caught me by surprise is that, according to the article, each one of us has a biased view of reality, and that our brains subconsciously deceive us in negative situations by transforming reality into a psychologically more comfortable state. —Preceding unsigned comment added by 201.21.180.57 (talkcontribs) at 23:44, 26 September, 2009

Have you ever heard of denial, or perhaps repression? This is all this is. However, this theory is accepted only in some views of psychology, most notably the Freudian and neo-Freudian views. You may be interested in defence mechanisms, which gives more information on what you discuss. Intelligentsium 00:19, 27 September 2009 (UTC)[reply]
Our individual world view is sometimes called our "reality construct". More bluntly, in his book, The Way of the Weasel, Scott Adams concluded that we are not only weasels with each other, but with ourselves as well.Baseball Bugs What's up, Doc? carrots 00:30, 27 September 2009 (UTC)[reply]

This same question was asked at the Humanities desk. →Baseball Bugs What's up, Doc? carrots 00:31, 27 September 2009 (UTC)[reply]

Diagrams of ancient earth

I was wondering if there was any one site that had alot of pictures of the earth like: http://en.wikipedia.org/wiki/File:Laurasia-Gondwana.svg At maybe 100 MA intervals (or at important times such as formation of supercontinents). Thanks~ 66.133.196.152 (talk) 01:33, 27 September 2009 (UTC)[reply]

You might find the Continents template helpful. I found plenty of maps for each of the major historical supercontinents, although it appears we don't have a single (easy-to-find) article that has a chronological line-up of maps. However, many of our articles reference the same sources, including paleomaps at Northern Arizona University; this PaleoMap project, and its accompanying animations; and lots of others. It seems that our Plate tectonics article is the center-point; we also have continent, with a great history section; but again, I can't seem to find a nice historical chronology with maps (even though we have maps on each separate article for each historical era - you can follow those links to each main article for maps and overview). Nimur (talk) 06:48, 27 September 2009 (UTC)[reply]

Thanks! Those sites are what I had in mind. 66.133.196.152 (talk) 12:53, 27 September 2009 (UTC)[reply]

Volume of mercury

Yes, this is a homework question. I've worked through the problem, got an answer, looked up the answer in the back of the book, and one of us is wrong.

The question is (I'm paraphrasing but getting all the figures straight): You have a beaker that has a capacity of 1000 cm3. There are currently 990 cm3 of mercury in the beaker. The temp is 20*C. At what temp (ignoring vaporization) does the beaker become filled to capacity.

The formula that I'm using is: deltaV = Vo * B * (deltaT)

With my values plugged in I have: 10 = 990 * 1.8x10^-4 * (Th - 20)

This gives me a high temp of 76.12*C where the beaker will be full. The book gives an answer 79*C. Who's wrong? Thanks, Dismas|(talk) 01:48, 27 September 2009 (UTC)[reply]

I get a different answer because the value I have for expansion of mercury is 182e-6C-1 (a little more exact). So, I get TF=75.5C. To get 79C, you have to use an expansion of 171e-1C-1. -- kainaw 03:41, 27 September 2009 (UTC)[reply]
Did you consider the fact that the beaker also expands? Dauto (talk) 03:48, 27 September 2009 (UTC)[reply]
No, I hadn't. Hrm... Dismas|(talk) 03:57, 27 September 2009 (UTC)[reply]
The beaker will not expand THAT MUCH in the course of 3C, the difference in temp between his answer and the books answer. It cannot be that. The coefficient of expansion of glass is miniscule compared to that of mercury, much below the significant figure range; I wouldn't even consider it a factor here. --Jayron32 04:02, 27 September 2009 (UTC)[reply]
jayron, think again... Dauto (talk) 04:28, 27 September 2009 (UTC)[reply]
Pyrex is used specifically because its thermal properties are negligible - preventing cracking, chipping, etc., even when rapidly changing its temperature. This supplier quotes 32.5x10-7/°C; this supplier confirms that value. Expansion of the glass beaker is negligible. Nimur (talk) 06:24, 27 September 2009 (UTC)[reply]
If the question had intended us to consider the expansion of the beaker, it would surely have specified what the beaker was made of because without knowing that, we cannot answer the question. We should definitely ignore that factor. SteveBaker (talk) 14:51, 27 September 2009 (UTC)[reply]
I tried integrating using the relation dv = B*v*dt, but that didn't give much difference. It, in fact, reduced to 75.58°C. I don't think you need to consider the expansion of anything other than mercury here. AFAIK, I think 75.58 must be the right answer, unless, as suggested above, you choose a different value for B. Rkr1991 (Wanna chat?) 06:14, 27 September 2009 (UTC)[reply]
This site from University of Denver quotes a quadratic rule for the thermal expansion of mercury:
"V(t) = V(1 + 1.82 x 10-4t + 7.8 x 10-9t2), where t is in °C, and V is the volume at 0°C."
It looks like the linear approximation may not be sufficiently accurate; this would seem to explain the discrepancy. Thermal expansion isn't really linear anyway - it's surprising that a linear approximation is ever good enough to work. Compounding the issue is that we're considering a reasonably large temperate range - fifty or sixty degrees celsius - so a more sophisticated model is probably needed. Nimur (talk) 06:33, 27 September 2009 (UTC)[reply]
I really don't think those are needed for this text book problem. This appears to be a school-level problem, which certainly wouldn't utilize all these sophistications, unless explicitly taught in class, which I don't think is the case. So I would say our only option is to conclude that the textbook has made a mistake, unless someone can come up with something radically new... Rkr1991 (Wanna chat?) 09:08, 27 September 2009 (UTC)[reply]
That amount of nonlinearity doesn't make much of a difference. Using that quadratic equation for the volume, the solution is 75.46°C. So it still just looks like the book is wrong. Red Act (talk) 09:44, 27 September 2009 (UTC)[reply]
The beaker has a capacity for mercury that is greater than its nominal capacity1000cm3 and the reason is that the mercury forms a convex meniscus before it spills over.Cuddlyable3 (talk) 12:43, 27 September 2009 (UTC)[reply]
What exactly do you mean by "Nominal Capacity" ? Since the capacity of the beaker in question is said to be 1000cm3, we must assume it is so for mercury. Rkr1991 (Wanna chat?) 12:54, 27 September 2009 (UTC)[reply]
My book would not be taking into account a meniscus. We're just learning a bit about thermal expansion. The formula that I provided is the one that we're to use. I'm just going to chalk it up to publisher's error. Thank you all for trying to help! Dismas|(talk) 13:01, 27 September 2009 (UTC)[reply]
Again, if the question had intended us to take into account the meniscus, it would have had to say what the dimensions of the beaker is - a tall-thin beaker would hold less mercury than a short-wide beaker. (Strictly: It would also be necessary to tell us what the beaker was made of since the extent of the meniscus depends on that.) So if we are intended to take the meniscus into account, the question would be unanswerable. SteveBaker (talk) 14:51, 27 September 2009 (UTC)[reply]
Please write to the principle author via the publisher. Probably his grad student made an error while slaving through the textbook questions. Sometimes such errors have appeared in several editions of a book. I have gotten back nice letters from such authors when I pointed out errors. Some schools have errata sheets they pass out which have corrections for booboos in textbooks. You might save other students the puzzlement you have experienced, but there is also benefit in learning to check your work and look for other solution methods to get the book answer, or back solving to see the possible origin of the erroneous answer. Does the erroneous coefficient 171e-1C-1 per Kainaw, correspond to some other element or common compound which would appear in the next line in the CRC Handbook of Chemistry and Physics, that the hypothetical underpaid overworked grad assistant might have looked at when preparing the answer guide? Edison (talk) 14:13, 27 September 2009 (UTC)[reply]
This question should be saved for posterity as an example of the right way to ask a homework question here. --Sean 15:38, 28 September 2009 (UTC)[reply]

what disease is this

hi.... wat kind of disease is this fits?

Your question is hard to understand. Are you talking about epilepsy, maybe? --Anonymous, 05:22 UTC, September 27, 2009.
A "fit" is a slang term for "a seizure or convulsion, especially one caused by epilepsy".[6] So "fits" doesn't always refer to epilepsy, and is really more of a word for a symptom, rather than a disease. Red Act (talk) 07:41, 27 September 2009 (UTC)[reply]
It is precisely a word for a symptom, not a disease. Epilepsy is kind of a symptom too - there is often a specific underlying cause (brain damage of some kind, I guess). In fact, there is probably always a specific underlying cause, just sometimes we don't know what it is. --Tango (talk) 09:37, 27 September 2009 (UTC)[reply]
Another possibility -- apparently Fifth disease is sometimes misnamed as "Fits disease". Looie496 (talk) 16:07, 28 September 2009 (UTC)[reply]

Body building

  • Question originally posted on the Humanities reference desk by user:Kanthasamy; moved here by me. --Anonymous, 05:30 UTC, September 27, 2009.

why the arms get unshaped

What do you mean? →Baseball Bugs What's up, Doc? carrots 05:52, 27 September 2009 (UTC)[reply]
See Bodybuilding Rkr1991 (Wanna chat?) 06:15, 27 September 2009 (UTC)[reply]

Tox-sick

Which is the best way to remove the toxins from our body i.e. liver,lungs,heart etc

What kind of toxins? Letting the body's own methods get on with it is usually best (there is certainly no point spending lots of money on "detox" products). For some toxins there are antidotes. If the toxin has been ingested and is still in the stomach it can help to either induce vomiting or consume something like charcoal to absorb the toxin so it can pass straight through you (don't try either of those without consulting a doctor, though). --Tango (talk) 10:40, 27 September 2009 (UTC)[reply]
Depending on what they're trying to remove, our detoxification article says doctors may use techniques such as dialysis or chelation therapy.
If you're asking about detox diets, though, you should remember that "Body cleansing and detoxification have been referred to as an elaborate hoax used by con artists to cure non-existent illnesses. Most doctors contend that the 'toxins' in question do not even exist." AlmostReadytoFly (talk) 11:44, 27 September 2009 (UTC)[reply]
"The toxins" implies the OP is believing the quackery and not referring to a specific toxin for which there is an antidote. Chelation therapy is recommended by some sites, without mentioning the risks (and the 30+ people who have died from it [7]). --Mark PEA (talk) 11:59, 27 September 2009 (UTC)[reply]
The OP doesn't say whether the question concerns long-term accumulated toxins or short-term poisoning. For critical poisoning one should know in advance what poisons one may have to handle (can children reach your household chemicals?) and know the local telephone number for poison information or hospital emergency. Ordinary emergency call operators have no chemical training. Nor do I think the Ref. Desk. should prescribe what medical treatment is "best". Cuddlyable3 (talk) 12:33, 27 September 2009 (UTC)[reply]

I just got it. "Tox - sick" - sounds like "toxic", that's clever. Cuddlyable3 (talk) 12:35, 27 September 2009 (UTC)[reply]

We need to be really clear on one point: The manufacturers of many, MANY 'quack' medicines and diets and stoopid pads you stick on the soles of your feet(!!) are being deliberately vague about "removing toxins". It's a great advertising bullet - but it's quite utterly meaningless. In fact, these things do nothing to remove poisonous substances from your body. They have zero effect (if you're lucky) or even add mildly toxic substances (if you're not lucky) because they tend to be based on herbal ingredients that escape proper medical testing thanks to a ridiculous legal loophole.
So if that vague description of "toxins" is what you're talking about - then forget it - stop worrying - you don't need to do anything.
However, if you're talking about a very specific toxin - such as the slow accumulation of lead in children in areas where lead-based paint or lead-based plumbing is common - then this is an important medical matter, and one that we, as a library reference desk, are not allowed to address. If you think you are getting slowly poisoned by something specific - then you need to take this seriously and go see a doctor as soon as possible - and possibly make a call to the EPA (or your local equivalent) to get the toxin removed from your environment.
SteveBaker (talk) 14:45, 27 September 2009 (UTC)[reply]
The ultimate challenge question, which succinctly summarizes Steve's commentary, is to demand from the manufacturer to clearly state which toxin(s) they remove. If they cannot provide a list, it is probably because it is illegal for them to claim something which has not been tested and shown to be true (or, it may recategorize them as a medicine, and force an entire new level of testing rigor). This is the reason why so many phony products hide in the generic "de-tox" verbage. But if they don't say what toxins they are removing, they are probably removing nothing. Nimur (talk) 15:39, 27 September 2009 (UTC)[reply]
Maybe "money in the wallets of the gullible and scientifically-illiterate" is a toxin? DMacks (talk) 18:37, 27 September 2009 (UTC)[reply]
Could be. AlmostReadytoFly (talk) 21:19, 27 September 2009 (UTC)[reply]

Internet lines

Were specific communication lines built for the purposes of the internet or does the internet just utilise existing communication lines? Clover345 (talk) 18:05, 27 September 2009 (UTC)[reply]

Both. And some originally-built-for-Internet are also used for other modes. DMacks (talk) 18:35, 27 September 2009 (UTC)[reply]
(edit conflict) Both. You might want to read History of the internet. In the very very early days, computers were connected over special-purpose lines - e.g. a special data cable ran from Palo Alto to Menlo Park, between the Stanford University campus and the Stanford Research Institute. The next phase was to use a telephone line, with a suitable modem on each end, and a routing computer at each end. These early routers were called Interface Message Processors, and they were built at four or five sites (UCLA, Utah, Stanford, and SRI in Menlo Park - if I recall properly). Over the next few years, telephone became the mainstay of the internet. As the 1970s passed on to the 1980s, the telecommunications industry started to go digital - actually switching over to a packet-switching network that made up the core internet backbone lines. This makes it difficult to distinguish whether a particular "wire" was used for "data traffic" or "voice telephone" or cable television traffic - in fact, it was a shared network connection as early as the late 1970s and early 1980s. This is a huge concept - a packet switched network is shared by data, voice, television, telephone, and computer traffic. So, you lay down a single network connection, and use it for all telecommunication. By the late 1990s, internet data became the dominant economic force driving the increased network connectivity - and many network theorists and economists attribute the dot com bubble to the over-investment in excess network capacity - "dark fiber". Today, we are still reeling from that surplus - hundreds of billions of dollars worldwide of cabling and routing equipment purchased in the 1990s - which sits unused unless we waste it away on freely available, bandwidth-heavy activities. For better or worse, it is stimulating many other economic side-effects - for example, free long-distance voice-calls to almost any number in North America are now standard with any mobile telephone plan - because the traffic can be routed for virtually no cost through the backbones that crisscross our continent. So many of these exist that competition has driven the rates down to near-zero (in North America). Nimur (talk) 18:40, 27 September 2009 (UTC)[reply]
Nimur I disagree that YouTube that you linked to is a waste. Cuddlyable3 (talk) 23:27, 27 September 2009 (UTC)[reply]
In any case, it wouldn't exist if network connectivity cost as much as it did in 1985. We've seen an exponential falloff - you don't have to pay $400 to download the reference I just linked. But somebody had to pay for that network connection. Whoever did, lost a lot of money. Nimur (talk) 23:36, 27 September 2009 (UTC)[reply]
There is a confusion here about costs of hardware and service that hardly justifies a perception that YouTube is a waste. Nimur is correct that connection costs in 1985 would have made YouTube a non-starter. YouTube was founded in 2005 using a combination of the latest video/audio data compression techniques and the Internet connections that were already established as commercially viable. YouTube is an added-value service and no one has lost money because of its activity (notwithstanding some copyright complaints). The fact that the YouTube founders were able to sell their enterprise for $1.65 billion is their serendipitous payback for innovation. We may bemoan the unfairness of that sum not being shared with network providers but unless you can show that YouTube has a major negative impact on operation of other Internet services, I don't think you should disparage it as a waste. The bandwidth relation of YouTube's Flash Video to non-video Internet services e.g websites, e-mail, search motors, Wikipedia, is analagous to the relation between analog TV- and radio- broadcasting. Cuddlyable3 (talk) 23:02, 28 September 2009 (UTC)[reply]
Here's a relevant news article - "US may need as much as $350 bln to extend broadband". Again, this serves to demonstrate that the installation of network capacity is a real cost, borne by somebody. However, the productivity, commercial marketability, and internet startup companies, which require the internet connectivity, rarely pay in to the system as much as they extract from it after it is built. I think this is the opposite of the "tragedy of the commons" - it is the "benefit" of the commons. Some "benevolent investor" (typically a Tier 1 network provider) pays a huge sum of money to develop a network - and everybody except them profits off of it. Nimur (talk) 20:54, 29 September 2009 (UTC)[reply]

Strange unit of weight, "OR" = 0.322 grams, what is it?

Resolved

Oh, ye wise friends -


I have in my possession an odd weight, made of brass - you know, the typical cylinder with a knob on top, used for weighing on balance scales way back then...

The odd thing is the stamping, and the mass of this weight.

It weighs (as closely as I can determine on my digital scale) 1610 grams, i.e. 56.8 ounces avoirdupois.

The marking on the weight is - and here I am completely baffled, even an extensive Google search doesn't help:

"OR 5000"


Thus, one OR would be 0.322 grams...


What is an "OR" unit - does it have something to do with weighing gold, since "or" is French for gold?

Any enlightenment would be gratefully received.

Greetings, --Janke | Talk 20:45, 27 September 2009 (UTC)[reply]

The troy ounce, according to our article, is 31.1035 grams, and is used for weights of precious metals (please don't tell me about weight vs mass; in this context the word "weight" means precisely mass, in the sense of "quantity of matter", and always has). Maybe your scale is a bit off, or maybe there's something sticking to your brass weight? --Trovatore (talk) 20:50, 27 September 2009 (UTC)[reply]
Nope - 50 x 31.1034768 grams (one troy oz) would make it 1555 grams, not 1610. My digital scale is more exact than that, a 1 kg weight reads exactly 1000 grams. From its appearance, I'd say this weight is from the 1800s, and I think, in addition to the "OR" (French for gold), it may have a background in banking (the person who gave it to me has...) --Janke | Talk 20:53, 27 September 2009 (UTC)[reply]


French word order is the same as English for number-and-unit combinations: "10 minutes", for example, is written identically in both languages. It also doesn't make sense that "OR 5000" would mean "gold 5000". Having said that, I have no idea what OR does mean. Remember that the language might not be French: if "or" is a word in English and French, it could easily be a word in several other languages. And of course it could also be an abbreviation.

There doesn't seem to be any unit of measure on Russ Rowlett's web site that's about 322 milligrams, nor one with the 1.61 kg mass of the whole object, nor one whose name starts with the letters "or". There was an old French unit of weight/mass called the once, but that was similar in size to our ounce, about 30.59 grams. --Anonymous, 04:08 UTC, September 28, 2009.

Using this website, I cannot find ANY standard unit of measurement with 322 mg = 1 ANYTHING. The only thing even in the right order of magnitude is a carat (mass), and that was still pretty far off (1 metric carat = 200 mg, 1 British carat = 259 mg, and 1 "Board of Trade" carat = ~205 mg.) Nothing else in any standard weight system anywhere I can find comes even close to that unit. It's likely not an actual measure of unit, rather, it may be a mark that means something entirely different. --Jayron32 04:30, 28 September 2009 (UTC)[reply]
Maybe it's the weight of gold equal to the value of 5000 of some particular currency. A lot of currencies were on the gold standard, and 322 mg of gold seems like it's in the range of plausible values. Randomly looking through some of the pages on some European denominations I didn't find any valued at that amount but for instance the French franc was pegged at 290.32 mg of gold. A weight like that would make sense for a bank to have, since they would need to be able to measure out the correct amount of gold to exchange for currency. Rckrone (talk) 05:29, 28 September 2009 (UTC)[reply]
  • Looking closer, I found one more stamping on the weight; "TARE FRANCS" - does that help? Google gives one (yes, only one!) hit, in a Latin text... Sure looks like Rckrone is on the right track, but the unit (if there is a unit involved) doesn't fit anything we know... --Janke | Talk 06:28, 28 September 2009 (UTC)[reply]
    • Hey, I'm on to something: 0.29032 grams of pure gold diluted to 90% purity (with, for instance copper or silver) would be very close to 0.322 grams. A Google search turned up this: "The Swiss 20 Franc Gold coin was minted in Bern and consists of 90% gold. . . . the gold Swiss 20 Franc coins are the most famous and were issued in Switzerland from 1897 to 1935." Can we consider this an answer to the original question? Any definite info, anyone? --Janke | Talk 06:59, 28 September 2009 (UTC)[reply]
      • One more hit: "Swiss 20 Francs coin, which contains 90% (.900 fine) gold. These coins, in as-minted, uncirculated condition, weigh 6.45 grams and contain .1867 Troy ounces of actual gold weight" - 6.45 divided by 20 gives 0.3225 - I think that's it! Thanks, Rckrone - you got me onto the right track... Now, I only wish I had those 5000 Swiss francs instead of the weight that only represents them... ;-) --Janke | Talk 07:06, 28 September 2009 (UTC)[reply]
Wouldn't that weight only represent 1 SF? Googlemeister (talk) 19:22, 28 September 2009 (UTC)[reply]
Good find!
Googlemeister: I don't really see how sig figs play into this. Are you talking about the discrepancy between 5000*(0.29032 g)/.9 ~ 1613g and the measured weight of 1610g? Rckrone (talk) 00:28, 29 September 2009 (UTC)[reply]
Rck, I'm sure Goog's abbreviation "SF" meant "Swiss franc" (more commonly Sfr. in traditional abbreviation or CHF in modern currency codes). Goog, the idea is that the 1610 g weight equals the weight of 5000 Swiss francs in gold coins of the relevant period. Makes sense to me too. --Anonymous, 04:18 UTC, September 29, 2009.
  • Indeed! Some sweet talk resulted in my obtaining a few more weights - in addition to the "TARE FRANCS OR 5000" weight, I now have brass knobs representing 200, 300, 500, 1000 and 2000 Swiss Gold Francs of the period 1897 to 1935! The discrepancy in the exact weight, 1612.5 grams and my 1610 g weighing result is most probably due to the tolerance of my digital kitchen scale... Now, I only need to know about the two smallest weights that I also got (they appear to be a different series altogether). They're marked 1M and 2M and weigh a little over 5 and 10 grams, respectively. Any ideas? --Janke | Talk 13:57, 29 September 2009 (UTC)[reply]
Oh I see, the weight is not 0.322 grams, the weight is 5000x larger. Googlemeister (talk) 14:18, 29 September 2009 (UTC)[reply]
One guess is the weight of some .900 fine silver Franc coin. The silver-gold exchange rate was fixed at 15.5 in France and probably elsewhere (Silver as an investment mentions this), and 15.5*0.32258g = 5g exactly (in fact this explains where the weird rate 9/31 g of gold = 1 Franc came from). This link [8] mentions such a coin in 19th century France. I don't know about a Swiss version, or what "M" is. Rckrone (talk) 19:40, 30 September 2009 (UTC)[reply]
The Swiss franc article has some information. They mention some of the silver coins used lower purity silver than 90%, which might explain why the weights are over 5g. Rckrone (talk) 20:10, 30 September 2009 (UTC)[reply]
Although the French franc article says the weights of the coins weren't changed from 5g when they went from 90% silver to 83.5% silver, and they're supposed to have the same value as the Swiss franc coins, so that explanation doesn't really make sense. Rckrone (talk) 21:17, 30 September 2009 (UTC)[reply]

is mars shit still sterilized

yo is mars shit still sterilized, now that there is 99% pure water on mars obviously anything we send there would bring lots of microbes that have no problems living there. now, i personally dont care if there used to be microbes before we put any there on ours mars shit but i bet a lot of scientists would, so is mars shit still sterilized so that the scientists can be sure that the life they discover is not earth microbes but mars microbes. also, is it true you could just pitch a tent on mars, and, as long as you had oxygen to breathe, you could be there in a tshirt and not explode from the zero pressure or freeze or burn up. are there any plans to camp out on mars.

Yes, Mars landers and still sterilised, as far as I know. The atmospheric pressure on Mars is so low that it would be considered a pretty good quality vacuum in a lab on Earth - you certainly couldn't survive without a spacesuit. --Tango (talk) 22:19, 27 September 2009 (UTC)[reply]
I can't think of a better sterilization method than an Earth-to-Mars transit: several months exposed to cosmic radiation, solar wind, intense cold, intense heat, the vacuum of space, and a total absence of nutrients and water, followed by a harsh atmospheric entry. There are extremophiles, but few can sustain that many different types of extreme conditions. Nimur (talk) 22:29, 27 September 2009 (UTC)[reply]
It might be different with shuttles -- what with pressurised cabins and all. John Riemann Soong (talk) 22:39, 27 September 2009 (UTC)[reply]
Have you looked at the Mars article? It gives the surface temperature range on Mars as minus 87 °C to minus 5 °C, and that is like Antarctica on Earth. The atmospheric pressure on Mars is less than 1 kPa compared to our 100 kPa. Camping in a tent is not survivable, not even with your own oxygen and tshirt. Cuddlyable3 (talk) 22:55, 27 September 2009 (UTC)[reply]
Armstrong Limit. You would move around without a suit at least as long as in 2001:A Space Odessey. Sagittarian Milky Way (talk) 23:53, 27 September 2009 (UTC)[reply]
Also, mars is mostly CO2 atmosphere. our microbes are more adapted to nitrogen-oxygen, so that would also help to kill them off. Googlemeister (talk) 13:45, 28 September 2009 (UTC)[reply]
Earth microbes could and likely do survive interplanetary flights. Not everything which goes to Mars is autoclaved. Edison (talk) 19:16, 28 September 2009 (UTC)[reply]

A discovery of excrement on Mars would revolutionise our knowledge of extra-terrestrial life. Cuddlyable3 (talk) 23:07, 28 September 2009 (UTC)[reply]

Making acohlo from water

How you making ackohol from plane water? —Preceding unsigned comment added by 79.75.61.101 (talkcontribs)

It isn't possible to make alcohol from nothing but water. All forms of alcohol contain carbon, which water does not contain. Red Act (talk) 23:08, 27 September 2009 (UTC)[reply]
Maybe he was asking about an in-flight beverage? Nimur (talk) 23:11, 27 September 2009 (UTC) [reply]
Converting "water" into "wine" is a popular magic trick.[9] But the magic trick doesn't really turn pure water into wine. It's just a magic trick. Red Act (talk) 23:18, 27 September 2009 (UTC)[reply]
The New Testament John 2:7-9 reports that Jesus made wine from water but the method is not disclosed. Cuddlyable3 (talk) 23:19, 27 September 2009 (UTC)[reply]
Of course, that's just a mythological story about something that didn't really happen. Red Act (talk) 23:42, 27 September 2009 (UTC)[reply]
Dear Jesus, these unbelievers Rev 21:8 are people that I don't even know. Jesus could have used the water to irrigate grape vines.Cuddlyable3 (talk) 12:14, 28 September 2009 (UTC)[reply]
Ok can I use air and water to cet the carbon?
  • Not easily. For a start, there is very little carbon in air - carbon dioxide makes up a fraction of a percent of the atmosphere. Almost all alcohol does, in sense, come from a mixture of air and water, though - plants turn the CO2 in air and water into sugar using photosynthesis (which requires sunlight), which is fermented into alcohol by yeast, or other micro-organisms. There is more involved in that process than just air and water, though. --Tango (talk) 23:37, 27 September 2009 (UTC)[reply]
  • (ec) Yes, you could, although that would be a painful process. The major source of carbon in air is CO2, which is at 380 ppm or so - or 0.04%. The well-understood way to make alcohol from water and air is to grow some sugary plants and then ferment the sugars into alcohol. You need some extra ingredients, but they really act as catalysts and are not used up (except for the sunlight). --Stephan Schulz (talk) 23:40, 27 September 2009 (UTC)[reply]

You could react ethylene with aqueous sulfuric acid .... you could acquire ethylene from that gas fruits emit when they are ripening. John Riemann Soong (talk) 23:51, 27 September 2009 (UTC)[reply]

And where do you get the sulfuric acid? --Tango (talk) 23:56, 27 September 2009 (UTC)[reply]
Fisher Scientific. Nimur (talk) 00:07, 28 September 2009 (UTC)[reply]

Are there any industrial imitations of the Calvin Cycle? Couldn't we toss the right enzymes in a solution of suitable pH and feed carbon dioxide into it? Maybe extract ethanol or acetic acid as products? John Riemann Soong (talk) 00:38, 28 September 2009 (UTC)[reply]

Its just far less complicated to let fresh fruit juice sit around and ferment. Heck, if pruno exists, it proves you can pretty much make alcohol from anything edible... --Jayron32 01:09, 28 September 2009 (UTC)[reply]
Well we're talking a large scale processes here. Economy of scale, etc. Hasn't anyone looked into the applications of a large pool of RuBisCO (& allies)? Seeing that CO2 is a major industrial output, we may have an environmentally-friendly solution (solution harhar) here. John Riemann Soong (talk) 05:14, 28 September 2009 (UTC)[reply]
Fusing two hydrogen (protium, to be exact) nuclei results in a deuterium nucleus and a beta particle. Fusing two deuterium nuclei results in a tritium nucleus and a free neutron. Fusing a deuterium nucleus and a tritium nucleus results in a helium-4 nucleus and a free neutron. Fusing three helium 4 nuclei results in a carbon-12 nucleus. This, along with the water, will give you hydrogen, oxygen, and carbon. Forming that into ethanol is comparatively trivial. Did I make any mistakes? — DanielLC 02:21, 28 September 2009 (UTC)[reply]


September 28

What species is this?

Unidentified spider

So many brown spiders and they all look alike. Does anyone know what species this is? It was taken in Arkansas, near water, if that helps. --ErgoSumtalktrib 03:11, 28 September 2009 (UTC)[reply]

Possibly a nursery-web spider -- family Pisauridae -- but I cannot see the eye arrangement in this picture so I'm really not sure. I can only tell that she's carrying an egg-sack. Hope this helps. --Dr Dima (talk) 16:47, 28 September 2009 (UTC)[reply]
second image
Here is a second picture if that helps. This is all I have of the spider. --ErgoSumtalktrib 23:56, 1 October 2009 (UTC)[reply]

carbon monoxide and the Calvin Cycle

Are plants able to carbon-fixate carbon monoxide? I assume it's a bit easier since carbon monoxide has this huge polar effect. It would be opposite of carbon dioxide though (the carbon is negative). Would it be kind of like how plants process cyanide? John Riemann Soong (talk) 05:19, 28 September 2009 (UTC)[reply]

[10] p168 section 6.9.3 Not yet found.
CO also affects nitrogen fixation in plants (like CO affects O fixation in humans) [11] page 17 Nitrogenase is the enzyme affected.
There's bacteria that use CO as an energy source in the sea - by oxidation to CO2. And other processes eg [12] 83.100.251.196 (talk) 11:41, 28 September 2009 (UTC)[reply]

does hyperconjugation stabilise only cations?

Here's what I understand about hyperconjugation...by orienting the hydrogen atoms' antibonding orbitals towards the positive charge of a carbocation, that's like the same thing as removing electron density from an antibonding orbital. Does this mean that carbanions (and lone pairs, negative charges, etc.) cannot be stabilised by hyperconjugation? Or would it in fact be stabilised by "anti-hyperconjugation" -- where the eclipsed or gauche conformer is more stable than the anti? John Riemann Soong (talk) 06:00, 28 September 2009 (UTC)[reply]

Did you mean "orienting the hydrogen atoms' antibonding orbitals towards the positive charge" - the orbitals must have electrons in - in CH bonds these are the bonding orbitals. eg [13] Hyperconjugation
Can carbanions be stabilised by hyperconjugation? - at first no, then yes "antibonding orbital-hyperconjugation stabilisation of carbanions" would be a good term. Yes - I think this does happen - for example elements lower down the periodic table Sulphur , Silicon [14] also search "silicon stabilised adjacent negative charge" etc etc stabilise an adjacent negative charge on carbon - I think this is due to the process you mention.
For first row elements I think the antibonding orbitals do not give any significant stabilisation.83.100.251.196 (talk) 11:51, 28 September 2009 (UTC) (There might be exceptions to this rule in the case of some per-flouro compouns - eg [15] p274 chap 7.1 - this is a special and extreme case - usually ignore)[reply]
The term "hyperconjugation carbanions" [16] turns up a lot with respect to P, Si etc.83.100.251.196 (talk) 12:03, 28 September 2009 (UTC)[reply]
Well if the pi orbital is empty (like in a carbocation), C-H bonding orbitals want to overlap with the (bonding?) p-orbital, but if it is filled, then the C-H antibonding orbitals want to overlap with the filled bonding p-orbital or the empty antibonding orbital? (What about the case of nonbonding p orbitals?). John Riemann Soong (talk) 16:49, 28 September 2009 (UTC)[reply]
Overlap of filled and unfilled orbitals results in net stabilisation, if both the orbitals are half filled (as in CH3· + CH3· ) then that forms a bond too. But if both orbitals are fully filled then the molecular orbital formed by overlap of them would have the antibonding, and bonding orbitals filled - so no net bonding or stabilisation eg as in He2 .
So it's Carbanion + unfilled orbitals = stabilisation when considering hyperconjugation.83.100.251.196 (talk) 18:40, 28 September 2009 (UTC)[reply]
Remember that in pi systems (resonance - not hyperconjugation) the inital p(i) orbitals are only half filled83.100.251.196 (talk) 18:42, 28 September 2009 (UTC)[reply]
Wait, overlap of a filled bonding and an empty antibonding orbital results in net stabilisation? John Riemann Soong (talk) 18:58, 28 September 2009 (UTC)[reply]
if you mean "antibonding" as in ["antibonding" orbital for the C-H bond] in hyperconjugation - yes - but the term "antibonding" has different meanings in different contexts - ie an orbtial may be antibonding in interaction with one atom - but bonding with another.
(edit conflict)I'll start again..
An anti-bonding orbtial can mean two things:
a. When filled with electrons it decreases the bond order -eg pi* orbitals in ethene.
b. When interacting with another orbital it produces an antibonding orbital (see a) - in case b this arises from the degeneracy of the orbitals - eg for H interacting with another H s orbital there is a s orbital that is bonding, and other that is antibonding. Take the other degenerate orbital on the second H and the situation is reversed.
For 'normal' (ie electron balanced) molecules (or parts of molecules) the antibonding orbital is always empty, the bonding orbital is always filled. I'll avoid these terms below:
Carbocation is stabilised by hyperconjugation interaction with a filled (or partially filed) orbital of the right symmetry.
Carboanion is stabilised by hyperconjugation with an empty (or partially empty) orbital of the right symmetry.
83.100.251.196 (talk) 19:06, 28 September 2009 (UTC)[reply]
Hmm, is this kinda related to proton sharing (in say an enone). Let me see if I have this right: a carbanion overlaps with a C-H antibonding orbital, weakening the first C-H bond but forming a second partial C-H bond? So it's sort of like a hydrocarbon intramolecular form of hydrogen bonding, and it results in net stabilisation because of delocalisation of charge?
I'm also wondering whether hyperconjugation generally destabilises carbocations. As I recall, the stability is methyl > pri > sec > tert. One professor (not mine though!) suggests this is because of induction, but I think the inductive differences are too weak and I think it's because of C-H repulsion with the carbanion orbitals. John Riemann Soong (talk) 19:25, 28 September 2009 (UTC)[reply]
First part yes, second part you wrote carbocation at firsst - carbanions are quite difficult if you are thinking of compounds like MeLi, BuLi, t-BuLi - since they are clusters (see methyllithium for some description) - the actual Me- might not ever exist.. 83.100.251.196 (talk) 19:45, 28 September 2009 (UTC)[reply]
ok I'm going to have two more goes at examples - this seems like a good example - consider NCCH3 (acrylonitrile) - then deprotonate it once..
Interaction of the first electron in the p orbital on C (ie the negative charge) with one of the cyanide (CN) pi bonding orbitals produces pi system stabilisation - this is a major effect. (here I assume the CH2 is sp2, and the negative charge is in the p orbital)
But additionally interaction of the same p orbital with the CN pi antibonding orbital does this:
NC-CH2-  >>>  -N=C=CH2
This is a minor stabilisation - here it works because N is more electronegative than C, and thus stabilises the negative charge. - So this can be expected to contribute to stabilisation - this is similar to hyperconjugation (in some ways)
Note that this doesn't work when using allyl carboanion, since the resonance structure is the same as the starting structure - ie interaction with the antibonding orbital give no stabilisation, but interaction with the bonding orbital does.83.100.251.196 (talk) 19:45, 28 September 2009 (UTC)[reply]
Here's the second part - consider C2--CF3 ... now consider on of the C-F antibonding (unfilled) orbitals being donated into by the negative charge on C.. If this is brought to it's logical conclusion you get this reaction:
CH2--CF3 >>> CH2=CF2 + F-
The idea of one of the bonds being weaken by some types of htperconjugation is absolutely right - in the case above you can see that F- is a stable anion - of course in the case of hyperconjugation it doesn't go all the way - you could still view the hyperconjugation as a resonance hybrid between the two (with the balance very much on the left).83.100.251.196 (talk) 19:45, 28 September 2009 (UTC)[reply]
Hmm, that's curious -- the induction effect parallels the hyperconjugation effect. Is it possible to separate the two effects? I mean, one is carried out via bonds and the other through space, right? John Riemann Soong (talk) 19:53, 28 September 2009 (UTC)[reply]
If you are doing a full molecular orbital treatment including the difference in nuclear charge between nuclei then you don't need to consider anything else - the method should explain every property..
Before chemists had MO methods they noticed that molecular properties could be explained by combining several different techniques - electron counting (pairing), valence pair repulsion, resonance structures, inductive effects, long range steric effects.. etc - if you can sensibly combine all these you should get a very good understanding of molecules, AND get the same results as those obtained from a supercomputer guessing a MO structures (and cheaper too).
But it's important to note that both are describing the same thing - the second set are empirical - based on a lot of observations of different molecules. The MO methods are more mathematical/computational - and require less empirical knowledge (or non at all). but don't mix the two up in explanations - a molecules reactivity or structure can be explained through either. Only if your MO calculation is very simple (ignoring a lot of factors) do you need to use empirical methods to 'fine tune' the answer.
To fully explain the inductive effect in MO calculations I think you would need to include nuclear charge and hyperconjugation.83.100.251.196 (talk) 20:16, 28 September 2009 (UTC)[reply]
Were MO calculations expensive back then? They seem pretty simple now -- I just load up a program on the library computer (2.6 GHz) and hit "calculate"! As long as I'm not doing something like a majorly large protein it gets done within 30 seconds to 10 minutes. (That was when I was doing beta-carotene derivatives or something.)
So is in fact hyperconjugation one of the "root causes" of the inductive effect? I don't quite get your answer. Both induction and hyperconjugation aren't MO explanations, aren't they? John Riemann Soong (talk) 20:38, 28 September 2009 (UTC)[reply]
(Yes - back then 2.6GHz was science fiction) It depends on how good your MO program is - simple ones just calculate direct interactions (some early ones used a lot of heavy approximations), you could check how good it is by using the fluorocyclobutane anion in the example linked above - the CF3 fluorines should be split in energy as described in the article (they'd still be split in energy/bond length in the protonated neutral form too - MO should do this too). I've no idea how good they are generally nowadays.
Inductive effect is mostly dipolar/electrostatic (or dipole conducted through conjugated system) - hyperconjugation should be a smaller effect on top of that.
MO calculations are explanations unto themselves - hyperconjugation is easier to explain in terms of molecular orbitals, the inductive effect is more an empirical non MO explanation. Non of the methods listed above 'explains' MO calculations as such - but the results of MO calculations can be compared to the 'hand waving' efforts of these empirical methods - in general there should be a match in the conclusions (usually is).
I suppose it depends on what you mean by separate - could be to attempt to measure the extent of each - specially prepared molecules (with the substituents held in place by rings) could be prepared to compare with non constrained (linear or branching), and/or molecules that were constrained so as to not have good inductive or hyperconjugative effects. Deviations from expected hammett derived activities would be another way that has been tried.83.100.251.196 (talk) 20:23, 28 September 2009 (UTC)[reply]

Neutron bomb question

Our article on Neutron bomb states that neutrons released from a nuclear explosion will not travel very far in air. But yet, that contradicts a statement in this rather well written analysis on Fourth Generation Nuclear Weapons, namely pure fusion bombs. http://arxiv.org/PS_cache/physics/pdf/0510/0510071v5.pdf it states in the target coupling section,

"Heat the volume of a material. Penetrating high energy radiations (neutrons, pions,15 or highenergy gammarays) will easily cross a low density intervening medium such as air and deposit their energy deep into any high density material. As a result, a substantial (i.e., centimeter to meter thick)layer of a bomb irradiated material can be brought to a temperature sufficiently high for it to melt, vaporize, or even explode."

So isn't it true that neutrons can penetrate easily through air? It seems logical, they are neutral after all, and are high energy (14 Mev in the case of a pure fusion bomb), so I would think they can penetrate through air easily. ScienceApe (talk) 06:29, 28 September 2009 (UTC)[reply]

Our article on neutron radiation agrees with you - it says "Because neutrons are uncharged, they are more penetrating than alpha radiation or beta radiation. In some cases they are more penetrating than gamma radiation, which is impeded in materials of high atomic number". The passage that you refer to in the neutron bomb article is confusing and may be plain wrong. Possibly it is confusing the fast neutrons produced by nuclear reactions with slower thermal neutrons, which are absorbed much more easily. Gandalf61 (talk) 09:02, 28 September 2009 (UTC)[reply]
There are a few different things going on here that you're each taking out of context.
In the context of neutron bomb radiation effects, "very far" means on the scale of, say, many meters. A 10kt fission weapon will put out very high energy neutrons for only some 800 yards or so. 800 yards is only "small" in the context of nuclear weapon explosions! Thus the article says:
Neutron bombs have low explosive yields compared with other nuclear weapons. This is because neutrons are absorbed by air, so a high-yield neutron bomb is not able to radiate neutrons beyond its blast range and so would have no destructive advantage over a normal hydrogen bomb.
AFIAK this is generally accurate except that the neutrons are not "absorbed" but "scattered". It's not a question of penetration in the sense used by "neutron radiation", it's about the fact that neutrons scatter off of light elements (oxygen, hydrogen) very easily, and by doing so, lose their energy (and thus become less of an effects problem). So if you set off a "regular sized" nuclear bomb, the range of neutron effects is going to be a lot smaller than the blast and other radiation effects, and thus not be of much use. In a neutron bomb, you use a low yield weapon, modified to increase radiation output, so that the radiation goes well beyond the blast and heat effects. Again, the distances/times involved are only "small" when compared to the other effects of a nuclear explosion, which are huge!
The article linked to above refers to radiation transport (which is what the paper refers to), inside an actual hydrogen bomb—it's talking about a couple of yards at most, usually just a few feet with modern warheads. Neutrons have no problem penetrating air at that distance. That's not what the neutron bomb paragraph is referring to at all.
There is a lot of discussion on this in Chapter VIII of Glasstone and Dolan's The Effects of Nuclear Weapons, 1977 edn. But it's important not to take these statements out of context and to start applying them from one domain to another. Neutrons are highly penetrating on table-top experiment. When you're talking about over the course of a mile through air, they scatter out comparatively quickly. --Mr.98 (talk) 14:58, 28 September 2009 (UTC)[reply]

Source transformation..........

This is something which I was wondering about........ I know that in principle, a 1A current source in parallel with a 1 ohm resistance is the same as a 1V voltage source in series with a 1 ohm resistance. My question is, how do you differentiate between the two ? Say you have 2 black boxes, totally identical from the outside. Say you can't open them and look inside. All you have is two terminals poking out of each box. One of them contains an ideal Voltage source in series with a resistance, and the other an ideal current source with a parallel resistance. How do you say which is which ? Is there any means at all......? Rkr1991 (Wanna chat?) 07:16, 28 September 2009 (UTC)[reply]

Yes, of course there is! You just connect the terminals across a one ohm resistance and measure the current or voltage. The series arrangement will have 0.5 volts and 0.5 amps. The parallel arrangement (There is no such thing as a 1A current source, so I've assumed one volt EMF) will have 1 volt and one amp (assuming no voltage drop in the source). Your hypothetical parallel arrangement would never be sold as a product because there would be a constant one amp internal drain on the source, though I suppose such an arrangement might be used in some voltage stabilising circuits. apology below Dbfirs 08:06, 28 September 2009 (UTC)[reply]
Read Current source. Agreed, there's no such thing as an ideal current source, but assuming there is, how do you differentiate it then is my question. Of course you can't replace that by a voltage source again, like how you explained. I am asking how to differentiate like how i asked. Rkr1991 (Wanna chat?) 08:39, 28 September 2009 (UTC)[reply]
No, you can't differentiate them at all. When Norton and Thévenin called them equivalent circuits, they really meant it. What Dbfirs is forgetting is that the current from the 1 A source will divide equally between the internal 1 ohm resistor and the external 1 ohm resistor, resulting in 0.5 A in each. Thus the current and voltage in the external resistor will be the same in both cases. --Heron (talk) 09:51, 28 September 2009 (UTC)[reply]
OK, but they are equivalent only if we assume Ohm's law right ? Isn't it possible to bring some sort of a situation where Ohm's law is no longer a good enough approximation, and hence we would be able to differentiate ? Or would they, even under such conditions, produce exactly the same outputs ? Rkr1991 (Wanna chat?) 10:43, 28 September 2009 (UTC)[reply]
Yes you can tell the difference. The box containing the current source is dissipating 1W internally which can be felt as warmth. The other box is cold. Previous answers are below the standard that the OP has a right to expect. Cuddlyable3 (talk) 12:07, 28 September 2009 (UTC)[reply]
Great answer. Thanks, everyone. Rkr1991 (Wanna chat?) 13:36, 28 September 2009 (UTC)[reply]

More methods:
#1 Place the two boxes in identical parallel universes. Measure the time to reach heat death in each universe. The universe with the voltage source dies second.
#2 The black paint on the box with the current source dries quicker.
Cuddlyable3 (talk) 15:34, 28 September 2009 (UTC)[reply]

Apologies to Rkr1991 for misunderstanding the question. I should have read the article first, even if the device doesn't exist. Dbfirs 18:00, 28 September 2009 (UTC)[reply]
There is no difference, even in terms of power dissipated. Edison (talk) 19:12, 28 September 2009 (UTC)[reply]
... except internally? Dbfirs 21:37, 28 September 2009 (UTC)[reply]
Edison please explain why you think I am mistaken. Cuddlyable3 (talk) 21:46, 28 September 2009 (UTC)[reply]
Yes, AFAIK, Cuddlyable's answer seems perfectly reasonable.Rkr1991 (Wanna chat?) 07:44, 29 September 2009 (UTC)[reply]

genius

do geniuses live long?

What makes you think they would ? Rkr1991 (Wanna chat?) 07:43, 28 September 2009 (UTC)[reply]
Some do, some don't (just like the rest of us). Has anyone done any research? There does seem to be a slight bias towards dying young amongst geniuses. Do some burn out, or is this just media hype? Dbfirs 08:19, 28 September 2009 (UTC)[reply]
To answer your question: "Further evidence for a positive relation of intelligence and suicide mortality is provided by the observation of excess suicide prevalence in the Terman Genetic Study of Genius sample, relative to the general population." from Voracek, M. 2004. National intelligence and suicide rate: an ecological study of 85 countries. Personality and Individual Differences, 37, 543-553.--droptone (talk) 12:47, 28 September 2009 (UTC)[reply]
IQ is associated with mortality:
It has recently been discovered that people with lower IQs tend to die younger than people with higher IQs. This association is found whether psychometric intelligence is measured in childhood (Deary, Whiteman, Starr, Whalley, & Fox, 2004; Osler et al., 2003; Whalley & Deary, 2001), early adulthood (O’Toole, 1990; O’Toole & Stankov, 1992), middle age (Pavlik et al., 2003), or old age (Korten et al., 1999). For example, Whalley and Deary (2001) traced 80% of people born in 1921 who took part in the Scottish Mental Survey of 1932 in Aberdeen city (n52,230 from 2,792). They used medical and public databases to find whether people were still alive at age 76 in 1997, 65 years after the IQ test. People with a standard-deviation (15-point) disadvantage in IQ score relative to others at age 11 were only 79% as likely to live to age 76. Further linkage studies of the Scottish Mental Survey 1932 found that lower childhood IQ is associated with earlier death from a variety of diseases, including cardiovascular disease (Hart, Taylor et al., 2003) and stomach and lung cancers (Deary, Whalley, & Starr, 2003). (from Deary, I.J. and Der, G. 2005. Reaction time explains IQ's association with death. Psychological Science, 16, 64-69)
Do you specifically want to know if this trend is continues as the IQ gets increasingly high?--droptone (talk) 12:38, 28 September 2009 (UTC)[reply]
And to answer your question specifically, here is a quote from Martin, L.T. and Kubzansky, L.D. 2005. Childhood Cognitive Performance and Risk of Mortality: A Prospective Cohort Study of Gifted Individuals. American Journal of Epidemiology, 162, 887-890: A 15-point advantage in childhood IQ was significantly associated with a decreased risk of mortality (hazard ratio = 0.68, 95% confidence interval: 0.49, 0.93) for IQ scores up to 163; beyond that, the risk of death plateaued.--droptone (talk) 12:47, 28 September 2009 (UTC)[reply]
I think it depends on the type of genius. Lots of artistic geniuses seem to die young, perhaps because artistic genius is strongly associated with risky or self-destructive behavior. "Intellectual" geniuses don't seem to follow this pattern, though. (Disclaimer: this is just my own impressions; never seen any data on the question.) Looie496 (talk) 15:57, 28 September 2009 (UTC)[reply]
Mozart, George Gershwin, and Irving Thalberg come to mind as artistic geniuses who died young, although it was disease and not lifestyle that got them, as far as I know. On the other hand, Aaron Copland and Irving Berlin both lived to be about 100. →Baseball Bugs What's up, Doc? carrots 06:12, 29 September 2009 (UTC)[reply]
They lived pretty long compared to maths genius Évariste Galois.... --Pykk (talk) 18:28, 29 September 2009 (UTC)[reply]
If geniuses died very young - we wouldn't know they were geniuses. That must tend to skew the numbers upwards. SteveBaker (talk) 21:50, 28 September 2009 (UTC)[reply]
This is interesting and all but then the question becomes why do people with higher IQ live longer? My guess is it is simply because they come from more wealth, on average, since IQ is not just about how quickly you can learn but also about what you know. More wealth means better education and they are also able to focus on education as opposed to someone who is very poor and has to work early to help the family. So they know more than those exact same people would know if they were very poor. And, a wealthier person is probably less likely to do a very dangerous job, such as being a miner. And, diet is going to be different as well which could have a large effect on life. For those who know more than me, do you think this sounds right or not? Thanks. StatisticsMan (talk) 15:19, 4 October 2009 (UTC)[reply]

Innovations in the last 50 years.

I believe there havent been any 'breakthrough' innovations in the past 50 years, in the field of science, or otherwise. Something on the scale of the invention of steam engine, electricity, computer etc.

You have to define breakthrough. Many scientists would disagree with you, though: Sanger sequencing (1975) made it possible to analyze, with modifications over the years, entire genomes; and restriction enzymes (1970) made molecular cloning possible, and with it most modern understanding of molecular biology. Someguy1221 (talk) 08:25, 28 September 2009 (UTC)[reply]
The Integrated circuit is now just out of your 50-year limit, but developments in data transmission have been phenomenal in the last twenty years. Does this count as "breakthrough"? Dbfirs 08:28, 28 September 2009 (UTC)[reply]
Electricity is not an invention, it's a natural phenomenon. For breakthroughs, consider the Internet, and the WWW that runs on top of it. GPS. If you allow for scientific breakthroughs, dark matter and dark energy are recent and fundamental. The superposition calculus is cool. Most of genetic engineering and DNA releated techniques post-date 1959. The first heart transplant was in 1967, and the moon landing in 1969. Of course, most newer inventions rely on older ones - but in that sense, there has been no breakthrough since some ape hit another with a rock for the first time. --Stephan Schulz (talk) 08:32, 28 September 2009 (UTC)[reply]

Yes Stephan, Internet is a good example. Landing on the moon was not something innovative. My point is that we might have become more intelligent in using old inventions and modifying them, but we have lost creativity with regard to new ideas.

Well the personal computer really falls into that range, and that was certainly game changing. How about cell phones and portable music players? Polymerase chain reaction is a good one too. ~ Amory (usertalkcontribs) 11:08, 28 September 2009 (UTC)[reply]

Has the OP delivered a declaration or a challenge? I can't see a question. The time from making an invention to it being recognized as having been a 'breakthrough' can be long so it is premature to conclude about recent inventions. Like Stephan Schulz comments about electricity, all inventions are discoveries of principles that were already there, just waiting to be invented.Cuddlyable3 (talk) 11:52, 28 September 2009 (UTC)[reply]

Transistor seems to be 55 years old in its present form. Pocket calculator appears to date from 1970, and believe me, it was truly groundbreaking for those of us who couldn't get the hang of either mental arithmetic or slide rules! --TammyMoet (talk) 11:58, 28 September 2009 (UTC)[reply]
Other great inventions of the past 50 years. Nuclear magnetic resonance is a bit old (1930's and 1940's), but its cousin Magnetic resonance imaging became practical in 1973 or so, and it is undoubtedly one of the most important modern medical diagnostic tools. Also, while Sputnik falls just barely outside this range (1957), the first practical Communications satellite was put into orbit in 1962 (see Telstar), and communications sattelites are hugely important to the modern world. If we ever move from the carbon economy to the hydrogen economy, then Fuel cells would be a huge innovation. While the principle was devised in the 1830's, the first modern fuel cells used in transportation came about in 1959-1960, so that falls just inside the 50 year window. --Jayron32 14:03, 28 September 2009 (UTC)[reply]
If you are only talking about the sorts of things that reconfigure entire societies and economies, the personal computer (something that is quite different from the previous ideas of the "computer") and the internet (and the developments in telecommunications that have been simultaneous with it) probably rank the highest. They are the backbone of our information society and have radically changed how the world works in the past few decades, with no signs of letting up. Additionally, the development of psychoactive medications has been a major game-changer for a lot of people, and signals a very different era in regards to making sense of our own minds that existed previously. And of course there are the advanced genetic techniques which are only now just beginning to have a real impact, but promise a lot more in the future.
Note that there is no reason to suspect there have been "fewer" creative things at all. Compared to the other time periods you are indirectly mentioning, we invent and move forward at a blinding pace. --Mr.98 (talk) 15:33, 28 September 2009 (UTC)[reply]

In many cases there have been decades between an innovation and the time it was recognized as a "breakthrough" -- electricity and the steam engine are examples. Fusion power generation, which was developed within the past 50 years, is currently in a state comparable to the earliest electrical generators. Gene sequencing (already mentioned above) has already shown itself as a major breakthrough. Superconductivity is still moving toward widespread usefulness. In fact I think an argument could be made that major breakthroughs have been happening faster than ever. Looie496 (talk) 15:51, 28 September 2009 (UTC)[reply]

There are some pretty huge inventions in the last 50 years listed in Timeline of historic inventions, some of the most important of which have not been mentioned above. I think perhaps perhaps hugely important inventions occur so frequently now, that we aren't as impressed by each of them as we would otherwise be. It's like we've grown to expect huge inventions. Red Act (talk) 17:42, 28 September 2009 (UTC)[reply]

I'm pretty sure we've had a similar question before and we came up with a similar answer, it depends on your definitions however there have been some very major changes even in the past 20 years which have substantially changed the way many people in developing countries live and work and many people would argue things are changing far faster now then before. Nil Einne (talk) 18:44, 28 September 2009 (UTC)[reply]

I would call manned spaceflight a pretty dramatic development of the past 50 years. Robotic exploration of the other planets is pretty dramatic as well. The Cat scan and MRI for medical imaging have reduced the need for exploratory surgery and are a quantum leap ahead of older Xray capabilities. Endoscopic surgery for tumors or defects of the base of the brain i a similar leap forward, compared to older brain surgery techniques. In the power industry, SCADA and computerized relaying were a dramatic breakthrough, compared to older remote control systems and electromechanical relaying, respectively. In the military/spying/assassination field, remote control airplanes which can circle invisibly for tens of hours and then hit a target with Hellfire missiles give dramatic new capabilities. Edison (talk) 19:06, 28 September 2009 (UTC)[reply]
I think the changes over the past 50 years are more than simple individual inventions - we've largely transitioned from the industrial age to the information age. As people describe changes in "ages" like that - we've only really had: stone age, iron age, the agricultural age, industrial age and now, the information age...that last change having happened almost entirely in the past 50 years. Not many generations of humans have lived through such a dramatic change. Many of us are able to make a living producing nothing but bits and bytes - with no physical/tangiable 'thing' coming out of the process. Google makes 20 billion dollars a year in a business sector that has only existed for 10 years! We are seriously able to say that all of human knowledge is available to any human who can afford a few hundred dollars to get online. We can communicate around the planet so easily that we've forgotten what it was like to have to wait for news to arrive. The idea that for $1 you could have literally any piece of music you could think of available to listen to within maybe 30 seconds of feeling the desire to do that...that's just stunning. I can talk to my car - tell it to communicate with my cellphone to call my wife on her cell phone - which in turn talks to her car to tell it that the phone is ringing. The car obligingly turns down the radio so she can hear. She tells her car to answer the phone...then we can talk - while on the move - from almost anywhere in the world to almost anywhere in the world.
Admittedly we've had many of the individual inventions for more than 50 years - but the integration of them into one massively interconnected whole is something that was inconceivable in the 1950's. All of this would have been like science fiction to people from the 1950's.
21:47, 28 September 2009 (UTC)
It also depends on the field you are looking at. In the field of music and music production the last 50 years has been essentially one long sustained period of innovation. Pfly (talk) 05:15, 29 September 2009 (UTC)[reply]

Betelgeuse Supernova

The article on the star Betelgeuse suggests that it may go supernova within a time to be observable by human civilization and possibly within a lifespan of humans alive today. If Betelgeuse were to do that, what would happen to the Earth? Would there be a danger from the radiation put off by a massive star exploding within a few hundred light years? Would it be dangerous to watch the explosion itself occur? What would it look like? 63.245.144.68 (talk) 11:09, 28 September 2009 (UTC)[reply]

Our article states rather clearly: "Since its rotational axis is not toward the Earth and also because of its 430 light year distance, Betelgeuse's supernova will not cause a gamma ray burst in the direction of Earth large enough to damage its ecosystems." What more are you looking for? -- kainaw 12:45, 28 September 2009 (UTC)[reply]
lol my phisics teacher was talking about this. He said that its a very slim chance.
The crazy thing is that it might have gone supernova when humans were running around the planet on wooden sailing ships, and we still would not know that it happened! Googlemeister (talk) 13:31, 28 September 2009 (UTC)[reply]
I'd say that was more paradoxical than crazy.
A paradox requires some kind of contradiction. There is no contradiction involved in the speed of light being finite. --Tango (talk) 15:37, 28 September 2009 (UTC)[reply]
Humans never ran around the planet on wooden sailing ships though one sailed around it on one. Cuddlyable3 (talk) 15:51, 28 September 2009 (UTC)[reply]
au contraire Googlemeister (talk) 19:09, 28 September 2009 (UTC) [reply]
From memory of movies (not a good source I know) it was uncommon to run around on wooden sailing ships, particularly if another ship is shooting at you or about to board you (or vice versa). Besides sounds like good exercise and they didn't have treadmills in those days Nil Einne (talk) 18:39, 28 September 2009 (UTC)[reply]
See Near-Earth supernova. SUch an event would likely only affect the Earth if within about 100 light years, which makes Betelgeuse an order of magnitude too far away to have any substantial effect should it go kaboom. Such events HAVE occured, though not on a human time scale, supernovas that affect the earth only happen about once every 1,000,000,000 which means there have only been 4-5 since the solar system formed. There have been supernova which have been observed in human history, see Crab Nebula, though that one is an order of magnitude FARTHER away than Betelgeuse. --Jayron32 13:54, 28 September 2009 (UTC)[reply]
So, if this becomes a visible phenomenon during our lifetimes (say, in the next 50 years), shouldn't we be using the past tense in terms of whether the supernova might have occurred about 400 years ago? I realize that many of you know this, I just think it's interesting as a grammatical and conceptual construct - we're hypothesizing that something may have happened about 400 years ago so that we can see it soon, and yet there will be no way for us to know whether it happened (during Shakespeare's lifetime) except when it becomes visible. --Scray (talk) 15:53, 28 September 2009 (UTC)[reply]
Potentially Hazardous Objects are rather more of a risk.--Shantavira|feed me 16:08, 28 September 2009 (UTC)[reply]
In my experience, it is more common to use the present tense to described events for which the light is just reaching us. It is only when writing for people with little knowledge of astronomy that anyone makes any mention of the fact that it actually happened many years ago. You may also be interested in Relativity of simultaneity, not strictly relevant to your question, but a related topic. --Tango (talk) 17:13, 28 September 2009 (UTC)[reply]
Just Imagine, "Supernova -2584371A" :) Sagittarian Milky Way (talk) 19:07, 28 September 2009 (UTC)[reply]

Surveying a pyramid

A Gravimeter can be both portable and very sensitive. What would be the resolution achievable by an array of gravimeters used to survey Khufu's pyramid for cavities? Cuddlyable3 (talk) 15:46, 28 September 2009 (UTC)[reply]

What really causes the resting cell membrane potential (in animal cells)?

I'm taking Anatomy and Physiology and our course book "Human Anatomy and Physiology" eighth edition by E. Marieb and K. Hoehn published by Benjamin Cummings states that the resting cell membrane potential is caused by the diffusion of Ka+.

However it seems to me that the KA+ ions are passively responding to the forces applied by the concentration gradient and electrical charge difference so that Ka+ is at equilibrium. It seems to me that what CAUSES both the concentration gradients and the voltage differential is NA+ being pumped out of the cell by the Sodium-potasium pump. Since the pump pushes out 3 NA+ ions and only brings in 2 KA+ ions, it would seem to me that the voltage difference is due to this "active" work and not the passive response of the KA+ ions to these forces.

I've looked at several on-line sources including the wikipedia article on cell membrane potential and most of them seem to attribute the voltage differential to KA+ diffusion.

Can anyone provide additional information so that I may understand why the voltage differential is attributed to KA+ diffusion and not the active Na+ pump?

Curious-BioPhysStudent (talk) 16:04, 28 September 2009 (UTC) Curious-BioPhysStudent[reply]

  • Mmm... Maybe you are considering two different kinds of potential; the negatives charges exceed in the inner side of the membrane, due to the presence of undiffusable, charged proteic anions in the cell. This causes the K+, wich can enter easily the cell, to be brought passively in the inner side, and the Ca2+ to be brought in the outer one. That's what happens in erythrocytes, wich have membrane a potential of about -10 mV. --87.1.124.116 (talk) 17:58, 28 September 2009 (UTC)[reply]
The OP is correct that nearly all the work of setting up the membrane potential is done by the sodium-potassium exchange pump, which is powered by ATP. However, the precise voltage that results depends on many other factors, including the permeability of the membrane to Na+, K+, Ca++, and Cl- ions.

Turning a car 90 degrees

Hi. Assume you have two cars, which both have front wheels that turn to the same maximum angle, but fixed rear wheels. The cars have different spacing between the front pairs of wheels and the back pair of wheels (because the cars are different lengths), and also different spacing between the wheels in each pair (because the cars are wider). Is there a formula to calculate how quickly the back end will turn 90 degress given the angle of the front wheels, the space between a pair of wheels and the space between the front and back pair of wheels (assuming that factors such as downforce etc are equal). I am trying to picture how to figure out how different sized cars would compare in terms of turning 90 degrees into a parking space, but can't find any online sources. Would appreciate any help you could give. Thanks Jimmy Laser (talk) 16:26, 28 September 2009 (UTC)[reply]

Our article, Turning radius, might help. I think the turning radius can be estimated from the wheel base, the angle the tires are at with respect to the ground, vehicle length, and so forth; but other experimental details can also affect the result. Nimur (talk) 16:59, 28 September 2009 (UTC)[reply]
Thanks, I had looked at that article and also the links it has, but I cannot see anywhere that directly discusses a calculation. I am also not sure if turning circle is exactly the same because the real issue is how soon does the backend finish the 90 degree turn (it probably is the same, but I am just having trouble picturing it). I imagine that the experimental details do make a big difference, but am trying to understand just the hypothetical impact of varying the distances between tires. I imagine it is just a case of working out the forces on the backend of the car, but just don't know how to figure this out. Thanks for your help so far. Jimmy Laser (talk) 17:46, 28 September 2009 (UTC)[reply]
Ideally the turning radius should only depend on the maximum angle of the wheels and the length of the car. To find the center of curvature (the point around which the turning car rotates) draw a line along the axis of rotation of the rear wheels, and a line along the axis of rotation of each of the front wheels. Where the lines intersect are your centers of curvature. You'll notice that you get a different point for each front wheel, but the effective center of curvature is probably going to be somewhere in between (probably right in the middle if you're driving slowly). The wider the car, the larger the discrepancy between the two front wheels, but otherwise width shouldn't play a significant factor in turning radius. The turning radius will be proportional to the length of the car. Rckrone (talk) 17:50, 28 September 2009 (UTC)[reply]
Apparently car steering corrects for the discrepancy between the centers of curvature of the two front tires (see Ackermann steering geometry), so the angles of the two wheels won't be the same when turning like I assumed above. Rckrone (talk) 18:07, 28 September 2009 (UTC)[reply]
Don't forget the differential, which controls the angular speed of each wheel, too. Nimur (talk) 04:19, 30 September 2009 (UTC)[reply]

Potatoes no good?

Why are potatoes not counted as one of your 5 vegetables a day?--CruelSea (talk) 17:22, 28 September 2009 (UTC)[reply]

I would suspect because while they contain some minerals and vitamins (vitamin C in particular), potatoes are mostly starch and water, i.e. carbohydrates. Much of the vitamin content is also lost in typical preparations. That said, a typical Secundo in Italy can have boiled potatoes or even fries as a side just as well as steamed broccoli. --Stephan Schulz (talk) 17:44, 28 September 2009 (UTC)[reply]
Potatoes, like maize and some others, are nutrutionally much closer to breads, and so usually get counted among those, even if part of a side dish. Culinarily, most cuisines would treat Potatoes like a "starch", much like pasta or rice, rather than a vegetable. See vegetable for further information. --Jayron32 18:38, 28 September 2009 (UTC)[reply]
from a biochemical basis, "vegetables" is a loose term. it includes things where we eat the leaves and other green parts, which supply one group of nutrients, things where we eat the fruits, like tomatoes and peppers and so on, which are nutritionally closer to their biolgical class, fruits and berries, and things like roots which are a mixed bag with sometimes leafy type nutrients (sweet potatoes, carrots, etc. for instance) and sometimes not so much. then there's rice, which is really a grain, of course. Gzuckier (talk) 20:12, 28 September 2009 (UTC)[reply]
However, making a salad from the leaves of potato plants would NOT be recommended. →Baseball Bugs What's up, Doc? carrots 06:08, 29 September 2009 (UTC)[reply]
Agree, not only is it toxic, but it would probably taste terrible! Googlemeister (talk) 15:17, 29 September 2009 (UTC)[reply]
Especially if you throw in some rhubarb leaves and crushed acorns. Top that off with some chocolates for dessert - maybe some crunchy frog or anthrax ripple. →Baseball Bugs What's up, Doc? carrots 17:50, 29 September 2009 (UTC)[reply]

Breed of dog with the most powerful sense of smell & Police dog breed

What is the breed of dog with the most powerful sense of smell? 87.1.124.116 (talk) 17:39, 28 September 2009 (UTC)[reply]

What are the most important features for a breed of dog used by policemen for discovering drugs or corpses? 87.1.124.116 (talk) 17:39, 28 September 2009 (UTC)[reply]

The Scent hound article says those dogs are generally considered to have the greatest sense of smell; you'll probably have to dig through the breed links in that article for specific breeds. The article Police dog has a list of breeds used for that profession (and the list itself has its own article, linked to in the Police Dog article). Detection dog has a few links to explore. Comet Tuttle (talk) 18:08, 28 September 2009 (UTC)[reply]
The Black and Tan coon hound probably has the most sensitive ability to detect and follow cold scent. Other factors may dictate the use of bloodhounds for tracking people. Edison (talk) 19:01, 28 September 2009 (UTC)[reply]
(ec)Have read the article Police dog? It mentions Labrador Retriever, Springer Spaniel, Bloodhound and Beagle as breeds able to sniff out bombs and drugs. Beagles are used in airports because they do not worry most passengers. Desireable qualities in a police dog are intelligence, loyalty, and a willingness to be trained and worked. Dog breeds that have extraordinary sensitive noses are Scent hounds one of which is the Bloodhound. Cuddlyable3 (talk) 21:10, 28 September 2009 (UTC)[reply]

Wow, that helps. Thank you, friends. I have a last question: wich is the canid in general with the most powerful sense of smell? 87.10.128.11 (talk) 01:40, 29 September 2009 (UTC)[reply]

emulsifiers

i need a comprehensive list of E numbers and their sourceMihir1825 (talk) 18:36, 28 September 2009 (UTC)[reply]

i have tried best to collect present info from this encyclopedia, but it is not sufficient. i need to know thoroughly the source of emulsifiers being veg or non veg.

I have removed the copied list of text. Interested readers can find the list at E number. There's not really a need to duplicate the whole thing here. If your concern is over keeping "kosher" or "halal", there is this resource: [17] which lists the additives acording to religious dietary traditions, but it does not explicitly list the original source of the additives. --Jayron32 18:41, 28 September 2009 (UTC)[reply]
Perhaps the OP is vegan? Nil Einne (talk) 19:28, 28 September 2009 (UTC)[reply]
Furthermore, for people with strict dietary practices, like vegans OR people with religious restrictions, may find that sticking to Whole foods may be easier than trying to track down the source of every additive. --Jayron32 20:20, 28 September 2009 (UTC)[reply]
By the way the E in E-number doesn't stand for emulsifier.83.100.251.196 (talk) 20:54, 28 September 2009 (UTC)[reply]
Here http://www.veggieglobal.com/nutrition/non-vegetarian-food-additives.htm or http://www.vegsoc.org/info/enumbers.html 83.100.251.196 (talk) 20:57, 28 September 2009 (UTC)[reply]

Human Mind Altering Parasites, Bugs, etc

Are there other human mind-altering parasites or bugs besides Toxoplasma gondii? I am looking for something like Dicrocoelium dendriticum that affects ants. --Reticuli88 (talk) 18:51, 28 September 2009 (UTC)[reply]

Rabies? Nil Einne (talk) 19:24, 28 September 2009 (UTC)[reply]
High fevers can cause delusions and can be induced by a great number of pathogens such as that which causes Dengue Fever or malaria. PS that Toxoplasma gondii is one of the more interesting species I have heard of. Making mice attarcted to cats! Googlemeister (talk) 19:35, 28 September 2009 (UTC)[reply]
depends on your definitions and where you draw the lines. No doubt that somebody with a bad cold behaves and thinks differently than when he's in good shape, for instance. Gzuckier (talk) 20:05, 28 September 2009 (UTC)[reply]
A Drug dealer is a human mind altering parasite. Cuddlyable3 (talk) 21:40, 28 September 2009 (UTC)[reply]
Religion. Imagine Reason (talk) 14:48, 30 September 2009 (UTC)[reply]

Mushrooms

Are mushrooms considered more plant than animal?--Reticuli88 (talk) 18:54, 28 September 2009 (UTC)[reply]

Mushrooms are fungus which is neither plant nor animal. Therefore, comparing them to either a plant or animals is going to be kind of like trying to determine if an airplane or a car is more like a boat. It is mostly stationary like a plant and has cell walls, but the material of the cell walls is not the same as plants. Googlemeister (talk) 19:06, 28 September 2009 (UTC)[reply]
Personally, I'd consider them more plant than animal, but scientifically I believe they have their own kingdom (Fungi), which is closer to animals than plants on the evolutionary cladogram.129.65.196.121 (talk) 19:09, 28 September 2009 (UTC)[reply]
See also Opisthokont or [18] for example Nil Einne (talk) 19:20, 28 September 2009 (UTC)[reply]
as is often the case, depends on how you define/classify/draw the lines. fungi are biochemically closer to animals (made of chitin, not cellulose; no chlorophyll) but behave more similar to plants, in general. Gzuckier (talk) 20:14, 28 September 2009 (UTC)[reply]

Plants are a subclass of algae, which split off from the other eukaryotes before a lot of the other kingdoms did. You really should look into plant and fungi alternation of generations (which give interesting evolutionary clues). Fungi are scavenging heterotrophs and spread much more quickly than do plants (which tend to stay rooted in one location). Remember, sessile sponges are animals too -- happened one day that one line of spongeish organisms decided to have the ability to use their vascular current channels for flotation and propulsion. John Riemann Soong (talk) 23:31, 28 September 2009 (UTC)[reply]

supersonic speed

If you have a hypersonic aircraft (say mach 10), would you be able to hear the noise from it's engines while you are flying along? I know on the Concorde jet you could because the speed of sound through metal is like 4x faster then air, but would you still get the noise even if you were surpassing THAT speed? Googlemeister (talk) 19:29, 28 September 2009 (UTC)[reply]

Yes you'd definitely hear the sound conducted through the material of the aircraft, though not through the air outside unless you're at the back and the engines are near the body of the aircraft.. The speed the plane goes at makes no difference to what happens inside, you'll still be able to talk to the person next to you. We're all moving far faster than the speed of sound round the sun and you can't even tell it without astronomical measurements. Dmcq (talk) 19:39, 28 September 2009 (UTC)[reply]
(ec) I would guess "yes". Speed is relative. You are stationary relative to the engines, so the sounds should have no problem reaching you. --Tango (talk) 19:40, 28 September 2009 (UTC)[reply]
True I forgot about the conduction through the aircraft body. What if there was a second such craft flying in formation with me 300ft to my right, could I hear his engines (assuming I could differentiate between his engine noise and my own craft's? Googlemeister (talk) 20:19, 28 September 2009 (UTC)[reply]
No, the sound wouldn't be able to reach you. If you have a supersonic plane going overhead you won't hear anything until the plane has gone past and the sonic boom reaches you. Dmcq (talk) 20:41, 28 September 2009 (UTC)[reply]
No, the sounds would be carried backwards by the air faster than it propagates, so it would never reach the other plane. --Tango (talk) 20:46, 28 September 2009 (UTC)[reply]
If the supersonic aircraft are close enough and long enough for their shock waves to overlap, there would be turbulence and I suspect that sound of the other aircraft would be heard through the airframe.Cuddlyable3 (talk) 21:36, 28 September 2009 (UTC)[reply]
Let me expand on the point "speed is relative" made above. What is important is the speed of the transmission medium relative to you and the source of the sound. You will always hear the sound conducted through the body of the aircraft because the speed of the body relative to you and the engines is zero. You will not hear the sound conducted through the air (unless the engines are almost directly in front of you, as Dmcq says) because the speed of the air relative to you and the engines is faster than sound. --Anonymous, 04:28 UTC, September 29, 2009.

In today's technology, building a zero Enertia vehicles (auto or airplane)

Is it possible in today's technology to create a zero-enertia vehicles? --Reticuli88 (talk) 19:46, 28 September 2009 (UTC)[reply]

Is zero Inertia what you meant? if so the answer is no - since the mass of the vehicle would have to be 0. Still impossible with todays technology.83.100.251.196 (talk) 19:58, 28 September 2009 (UTC)[reply]
I assume that you mean "inertia" rather than "enertia". In that case, no. Inertia is a fundamental property of all mass. Star Trek's "inertial dampening field" remains pure fiction with no basis in reality. — Lomn 19:59, 28 September 2009 (UTC)[reply]
The Zero Inertia powertrain is an innovative powertrain concept in which kinetic energy is exchanged between an additional flywheel and a car engine’s inertia. Cuddlyable3 (talk) 21:25, 28 September 2009 (UTC)[reply]
That's just a type of regenerative braking isn't it? You would still feel the acceleration, it's not like Star Trek's inertial dampeners. --Tango (talk) 21:53, 28 September 2009 (UTC)[reply]
Sorry Inertia is a fallacious concept. Only mass exists and that obeys Newtons laws which did not mention inertia AFAIK--Tappet (talk) 21:32, 29 September 2009 (UTC)[reply]

Effects of the acidity of juice

What kind of effects would a very acidic juice have on the human body? Has any research been done about this?--Dlo2012 (talk) 20:36, 28 September 2009 (UTC)[reply]

For stuff like lime juice the effect would the same as pickling. For strong acids see John George Haigh who performed the experiment. Dmcq (talk) 20:47, 28 September 2009 (UTC)[reply]

I'm sorry, but what I meant was juice that is consumed by a person. Orange juice would be an example.

If you mean naturally occurring strongly acidic juices such as lime juice, then its pH of around 2 is about the same as gastric acid, though the stomach can produce much stronger acid up to 0.8 pH. The stomach is designed to cope with this, but the throat and gullet aren't. Dbfirs 21:28, 28 September 2009 (UTC)[reply]
Well I've used lime juice in marinades but to really break meat down you'd want to use enzymes as well like the stomach does. Dmcq (talk) 22:47, 28 September 2009 (UTC)[reply]
(original research) When I eat lemons, the juice seems to etch the teeth a bit. Edison (talk) 23:53, 28 September 2009 (UTC)[reply]

Not that original! ;-) Dentists warn against drinking strongly acidic juices too often! I use to say that my teeth get a "woollen sweater"... ;-)--Janke | Talk 14:10, 29 September 2009 (UTC)[reply]

[A web site ] says the ph of lemon juice is 2.3, lime juice is 1.8-2, orange juice is 3-4, vinegar is 2.4-3.4, stomach acid is 1-3. Those who binge and purge often eat the enamel off their teeth, from the stomach acid. Acid erosion says fruit juice and soft drinks can demineralize teeth and remove enamel. That article says wine (pH 3.0-3.8) can erode teeth. [19] says Coca Cola Classic has a pH of 2.5. Edison (talk) 21:46, 29 September 2009 (UTC)[reply]

September 29

carbon monoxide as a nucleophile

Can I use carbon monoxide as a nucleophile for carbon-carbon conjugate addition or a direct attack on carbonyls? What happens to the triple bond? If some water is around, could I make a carboxylic acid with it? John Riemann Soong (talk) 00:01, 29 September 2009 (UTC)[reply]

Half the answers can be found at Carbon_monoxide#Organic_and_main_group_chemistry. CO is a nucleophile (lewis base), but:
I've never heard of CO reacting with carbonyl - it's isoelectronic with CN- and N2 . CN- does react with carbonyls [20] , but CO is much more like N2 in it's reactions (mostly due to its neutral charge) - it's unlikely to react. If it did you'd get: (with an acyl chloride)
        O-
        |
      R-C-C+=O
        |
        Cl
This could do many things - form an alpha lactone, and then eliminate CO2 giving a carbene, OR dimerise to give a 'di-lactone'..
Compare with a diazomethan compund - and how unstable they are - it's unlikely to form at all.
The Koch-Haaf reaction for formation of a carboxylic acid is pretty straightforward [21] [22] but requires strong mineral acid, and CO pressure around 100atm. The very reactive electrophile carbocation is made in situ to react with the CO.83.100.251.196 (talk) 02:23, 29 September 2009 (UTC)[reply]

tertiary amines and acyl chlorides

Tertiary amines actually react with acyl chlorides, right, just that the K isn't very favourable? (The amine is a better leaving group than Cl-?) If I found a way to remove the product, could I drive the reaction to completion and get a quaternary amine salt? John Riemann Soong (talk) 04:06, 29 September 2009 (UTC)[reply]

I do see literature examples of this reaction. Distilling the product from the reaction probably isn't useful: RCOX with X=Cl is more volatile than than with X=NR'3, so you'd wind up distilling out the starting material and shifting the equilibrium away from RCON+R'3. You need to remove the Cl...think about adding something MX' that replaces Cl with a non-nucleophilic anion X' so that MCl precipitates? On the other hand, if you pick a solvent in which RCOX and NR'3 are soluble but the RCON+R'3Cl salt is not, it precipitates out as it forms. DMacks (talk) 04:46, 29 September 2009 (UTC)[reply]
With cloride silver is always a joker! Silver halides are insoluable and precipitate and shift the equilibrium. If yo use a silver salt with a non-nucleophilic anion like nitrate than it should work.--Stone (talk) 21:36, 29 September 2009 (UTC)[reply]
Silver tetrafluoroborate (or Silver hexafluorophosphate) is a better choice - when working with organic compounds nitrate is one to avoid.83.100.251.196 (talk) 23:05, 29 September 2009 (UTC)[reply]

Male body area name?

What is the name of the area immediately above the penis, but far below the belly button? About the area a speedo swim suit covers up? --68.103.141.28 (talk) 04:14, 29 September 2009 (UTC)[reply]

It looks to me as though it's the lower part of the hypogastrium, or hypogastric region, or pubic region. (However, note that the diagram on the Wikipedia page seems to show that region extending farther down than the diagram on the University of Michigan page linked to from it. Maybe there is some disagreement.) --Anonymous, 04:50 UTC, September 29, 2009.
I'd say more like the pelvic/pubic region. bibliomaniac15 04:55, 29 September 2009 (UTC)[reply]
Pubis as noun, pubic as adjective, is a good approximation though common usage often includes the entire region including the inner thigh, as in pubic hair. However, the medical term is fairly specific, with the mons pubis (in men and women) overlying the symphysis pubis. -- Scray (talk) 09:37, 29 September 2009 (UTC)[reply]
It's worth pointing out that the above is the straight answer to the question, and everything after that has gone south. →Baseball Bugs What's up, Doc? carrots 05:04, 30 September 2009 (UTC)[reply]
Here is an image I found. The area in question is cropped and highlighted. Does this fatty patch in obese males have a name? It's well defined under the abdomen. I hear the word "pannus" referring to the belly overhang. But this area does not seem to have a name. How does it form? Why does an obese belly stop at the natural "seam" above this area and not grow into that area? --68.103.141.28 (talk) 03:52, 30 September 2009 (UTC)[reply]
This section is getting more educational by the minute. I'm fairly certain that rectangular area could safely be called the flabbis dunlappis. →Baseball Bugs What's up, Doc? carrots 04:38, 30 September 2009 (UTC)[reply]
Caused by Dunlaps Disease (aka The Spare Tire. Aka "His stomach done laps over his belt"). --Jayron32 04:48, 30 September 2009 (UTC)[reply]
Yes, I was presenting the formal Faux Latin name. The disease has obviously reached a point of severity, i.e. you can't tell if it's male or female. →Baseball Bugs What's up, Doc? carrots 04:50, 30 September 2009 (UTC)[reply]
According to this source it's a "flap." Bus stop (talk) 05:00, 30 September 2009 (UTC)[reply]
There's a video of this? TMI. →Baseball Bugs What's up, Doc? carrots 05:03, 30 September 2009 (UTC)[reply]
See Muffin top. Cuddlyable3 (talk) 23:39, 30 September 2009 (UTC)[reply]

Aboriginal North americans as stewards of nature

Growing up in north central Ontario, I was always told that Native Peoples were good stewards of nature and that as a culture they lived "in harmony" with nature. From an historical/archeological/anthropological perspective, how accurate is this? Wer Native peoples just as eco-unfriendly as we "white" folk are? —Preceding unsigned comment added by 209.161.230.65 (talk) 05:00, 29 September 2009 (UTC)[reply]

Native people were largely "eco-unfriendly". They did whatever best suited their interested with little regard for the long-term health of the environment. But then, after a few thousand years and a relatively small population an equilibrium can be reached. In much of North American native peoples used wildfire to thin forests and promote a "park land" type landscape that provided many benefits--such as drawing wildlife like deer, growth of berries, and the ability to see longer distances for defensive purposes. After European colonization took hold and native populations crashed the "open landscapes" rapidly became dense and dark. The "wilderness" that people like Thoreau wrote about where not ancient but the result of the extirpation of natives. There are numerous examples of native peoples overtaxing the environment to the point of ecological collapse. Finally, in the case of Plains Indians, once firearms and horses were acquired in significant numbers the toll on bison herds was severe. While the ultimate collapse of the bison must be blamed mostly on non-natives, there was a significant native contribution to the process (this is explored in some detail in the book The Comanche Empire). The case today may be different, as many native communities work hard to preserve the environment of their reservations (though to varying degrees). In some cases this contributes to the continued poverty in native reservations because the environmental regulations make it less attractive for natural resource extraction companies to do business there. I'm coming at this from the perspective of the Pacific Northwest--it may be different elsewhere. In Ontario, once Europeans came and began offering valuable trade goods in exchange for beaver pelts the natives very rapidly depleted beaver stocks over an extremely wide region. This would be one example of not living "in harmony with nature". Pfly (talk) 06:01, 29 September 2009 (UTC)[reply]
I think the idea that native peoples were careful stewards of nature until corrupted by colonial invaders is now recognised as a romantic over-simplification. The deforestation of Easter Island is thought to be largely due to overpopulation by native islanders, and the extinction of Australian megafauna such as Genyornis is thought to be due to hunting by early human settlers during the Late Pleistocene. Gandalf61 (talk) 11:04, 29 September 2009 (UTC)[reply]
There are also some good books, though I can't recall their titles offhand, about the massive agricultural projects of the natives of South America as well, which were fairly eco-unfriendly by modern standards. In all cases, though, compared to Europeans, the natives do pretty well, mostly they didn't have the technical ability to ratchet up their environmental exploitation exponentially. --Mr.98 (talk) 16:31, 29 September 2009 (UTC)[reply]
In that vein, I would recommend Jared Diamond's Collapse: How Societies Choose to Fail or Succeed as an excellent and accessible book on how environmental factors – and our responses to them – shape the success or failure of human societies. Wikipedia's summary of the book gives a taste of the contents. TenOfAllTrades(talk) 20:17, 29 September 2009 (UTC)[reply]
The introduction to that book in particular addresses this specific question. Diamond concludes that both the image of native peoples as "ecological stewards" and, on the other hand, the less-common idea that they were callous and wasteful of natural resources are both based on the false premise that they're fundamentally different from modern "civilized" people. He recommends considering them in the same terms one would consider any modern society - as a people faced with certain challenges, trying to overcome them as best they can (sometimes this would mean taking steps to protect the environment, other times it would mean doing thing we would consider environmentally harmful). Even if you're not up for taking on the entire book, checking out the few pages of the introduction that address this at a library or something might be worthwhile. -Elmer Clark (talk) 04:26, 2 October 2009 (UTC)[reply]
In some sense, they were "in harmony with nature": they'd been around long enough, doing the same things long enough, for nature to evolve and incorporate them into the ecosystem. --Carnildo (talk) 01:13, 30 September 2009 (UTC)[reply]
I thought that the Natives only came to North America in the last 10-15,000 years. That does not seem like sufficient time for anything to evolve. Googlemeister (talk) 13:17, 1 October 2009 (UTC)[reply]
That number is very much in flux at the moment. It's hard to gauge, but I think most archaeologists are coming around to accepting substantially older dates. However, we're still talking about a few tens of thousands of years, not millions or anything. "Evolution" in that context might not be the correct term, perhaps "adaptation" would be more correct. Matt Deres (talk) 03:49, 2 October 2009 (UTC)[reply]

i have a pancreas working with a capacity of 80%!!!

I am a diabetic since April, 1986. Test Result for H.Pylori IgG was 25 positive then after treatment became 6 Negative. Test Result for C-Peptide was 2.8 (80%)which means that my pancreas is functioning all the time. I want just an explanation as I still take insulin injection 50 American Units. My Doctor is giving me Galvus Tablets. thanks —Preceding unsigned comment added by Luckystar100 (talkcontribs) 09:32, 29 September 2009 (UTC)[reply]

This is not a place to give medical advise, as given in the guidelines at the top of this page. Please consult your doctor. Rkr1991 (Wanna chat?) 10:05, 29 September 2009 (UTC)[reply]
To start with, you can read the diabetes mellitus article, which may help you understand diabetes. However, for anyone to be able to give you a complete explanation would require them to have intimate knowledge of your medical history, such as your doctor. If you don't understand your medical condition, be persistent about asking questions at your next visit. Write them down ahead of time. Don't leave until you understand. If the doctor can't or won't take the time to explain it, ask the nurse. If nobody at the office is willing to educate you about your condition, then switch doctors. Your health is too important to accept mediocrity. There are plenty of bad doctors out there, but there are also plenty of good ones who will MAKE SURE that you understand what is going on in your body, so that you are fully invested in the treatment they offer you. --- Medical geneticist (talk) 13:48, 29 September 2009 (UTC)[reply]

0mph - Warp Drive in (?) seconds

Without in inertial dampener how many Gs would an astronaut feel when travelling at warp speed (or the speed of light, even), and wouldn't the rapid (instantaneous) acceleration kill said astronaut immediately? Getting into science fact and rather than science fiction, what is the top speed that a human could safely travel in space without experiencing the effects of high speed usually felt here on earth? --KageTora - SPQW - (影虎) (talk) 10:00, 29 September 2009 (UTC)[reply]

As far as we know, nothing material can travel at "Warp speed" or even the speed of light. Abstracting from indirect effects, astronauts can travel safely at any speed less than the speed of light, though. Speed is not acceleration. To reach a given speed, an astronaut has to accelerate, but he can accelerate as slowly as desired - at the cost of reaching his or her target speed later. You don't usually feel any effect of high speed on Earth either, but high speed often goes hand in hand with significant acceleration (e.g. when turning). --Stephan Schulz (talk) 10:07, 29 September 2009 (UTC)[reply]

Speed does not kill you, ever. Not even here on earth. It is acceleration that kills you, if it is sufficiently large. So no, an astronaut travelling arbitrarily close to c would not feel any acceleration (Gs). And, if concepts such as the Alcubierre drive ( http://en.wikipedia.org/wiki/Alcubierre_drive ) turn out to be possible, even people traveling at "warp speed" would not feel acceleration. —Preceding unsigned comment added by 157.193.173.101 (talk) 10:10, 29 September 2009 (UTC)[reply]

:That reminds me of what my father in law used to say, who had once been in the RAF: "It's not the speed that kills - it's the sudden stop!" --TammyMoet (talk) 10:24, 29 September 2009 (UTC)[reply]

Similarly, falling off a tall building doesn't kill you. Cuddlyable3 (talk) 13:56, 29 September 2009 (UTC)[reply]
(EC)Thanks, so this is getting close to what is confusing me. What is it that makes a person in a centrifuge black out (and eventually die)? They are travelling at extremely high speed (of course, they have accelerated to that speed in order to get there, but after a while the speed is (or can be) kept constant). Is it because they are continually 'turning', so to speak? I can't understand that, because if the turning itself is constant, would it not be the same as going in a straight line? --KageTora - SPQW - (影虎) (talk) 10:25, 29 September 2009 (UTC)[reply]
No. A body left undisturbed will travel a straight line. In a centrifuge, bodies travel in a circle. They are constantly accelerated towards the center of that circle by a centripetal force. This is perceived by people in a centrifuge as a centrifugal force. When the centrifuge starts, there is an additional force that accelerates it sideways (and that can be felt or measured by the passengers), but that is not the major force that centrifuges are built for. --Stephan Schulz (talk) 10:46, 29 September 2009 (UTC)[reply]
We have an article, circular motion, that explains this important fact - motion in a circle is constantly accelerating - and in a different instantaneous direction, always tangential to the velocity. Nimur (talk) 11:59, 29 September 2009 (UTC)[reply]
If someone had a spaceship capable (don't ask me how) of sustained 1 G acceleration (like normal Earth gravity), starting in orbit, then the speed (neglecting relativistic effects) would be the acceleration times time. How fast would he be going in one month (neglecting relativity)? (9.8 meters/sec2)*(1kilometer/1000 meters)*(3600 seconds/hour)*(24 hours/day)*(30days/month)=25401 kilometers/second after one month, or about 1/12 the speed of light. (Fitzgerald contraction should be less than a 1% effect so far). Keep up 1 G of acceleration for a year and amazing relativistic effects would kick in. Thus neither high accelerations nor "inertia dampers" would not be needed to gain high speeds. (feel free to throw in relativistic refinements which keep our travellor from reaching c after a year.) Edison (talk) 13:38, 29 September 2009 (UTC)[reply]
I won't ask how someone's spaceship works but all of its 1G thrust capability would first be directed to escape from orbit, so the speed after a month would be less than you calculate. Cuddlyable3 (talk) 13:56, 29 September 2009 (UTC)[reply]
For this sort of back-of-the-envelope exercise, the rounding errors are going to be larger than the effect of Earth's gravity. The escape velocity from the surface of the earth is only about 11 km/s; one hits that after less than 20 minutes of straight-line one-gee acceleration. (And yes, I do realize that that number contains some slightly inaccurate approximations as well.) After a month, the contribution of relativistic effects will be a bigger concern than Earth's gravity well. TenOfAllTrades(talk) 14:11, 29 September 2009 (UTC)[reply]
With relativistic corrections, how far could the 1G spacetraveller go in 1 year and in 10 years (ship time) (allowing 1/2 the time to accelerate and 1/2 the time to decelerate, all the while experiencing effectively Earth gravity (except for "turnaround" at the halfway point). Nonrelativistically, it seems he would go about .26 light year in a year, reaching a maximum speed of about .5 c. But in the 10 year case relativity would have a strong effect, since otherwise he would have reached a speed of about 5c after 5 years, if he could have kept adding 1/12 c per month to the speed as he did initially.(This is emphatically not a homework question, but my calculus is rusty). Bonus: In one year or ten years as experienced by the Girl He Left Behind on Earth, how much time would have passed for the spaceman? Edison (talk) 15:53, 29 September 2009 (UTC)[reply]
A website with a relativistic calculator says that at 1 G constant acceleration, and turnaround and deceleration from the midpoint, the spaceman would in 10 years ship time, travel 167 lightyears, reaching a maximum velocity halfway of .0.99993 c, while the person left back on Earth would have experienced 169 years. The calculator says that for 1 year (ship time) at 1G, the distance would be .264 lightyear, at a max speed halfway of .475 c and that the Earth observer would have experienced only 1.045 years. A 20 year trip (shiptime) at 1G would take him 30000 lightyears, while 30001 years passed on Earth. Impressive, but boring compared with "Engage warp drive!" and being lightyears away in a few hours. Long-sustained (over months or years) 1G acceleration seems impossible with onboard energy driving it. Edison (talk) 16:37, 29 September 2009 (UTC)[reply]
What is impressive is that at 1G you could reach the each of the observable universe (at least, where it now) in less than 50 years (if you didn't want to stop - a little under 100 years if you want to stop at the end). That is basically the largest distance that has any significant meaning and you can get there in a human lifetime at the acceleration that humans have evolved under. I think that is really impressive. --Tango (talk) 17:01, 29 September 2009 (UTC)[reply]
The interesting consequences of that sort of travel are explored in Poul Anderson's science fiction novel Tau Zero. I recommend it for fans of hard science fiction. TenOfAllTrades(talk) 20:07, 29 September 2009 (UTC)[reply]
Tau Zero had a fascinating concept, but the author neglected general relativistic calculations; as the ship continued to travel, it would inevitably enter a region of space in which the velocities of nearby particles were less than rediculous (the same reason the oldest neutrinos in the universe, produced shortly after the big bang, are predicted to be moving somewhat slowly). And since the ship isn't going fast enough through the entire crunch/bang cycle, the process happens fast enough that quantum effects won't save it (or at least that's what I surmised/recalled as saving the ship, please correct me if I'm wrong). Someguy1221 (talk) 05:09, 30 September 2009 (UTC)[reply]
Are you saying the ship will be (kind of) redshifted by the universe expanding, meaning it will be moving slower relative to the objects it is passing than it is going relative to Earth? What kind of redshift do you get over those distances? Would it make much difference? --Tango (talk) 21:44, 30 September 2009 (UTC)[reply]
Something like that. You may know that in Special Relativity the Velocity-addition formula is a bit more complicated than it is in classical mechanics. In general relativity, the equations get insanely beyond my understanding [23]. The best I can give you is that it was once explained to me, in the discussion of that very book, how the formula works out for a near light-speed ship going far enough. Someguy1221 (talk) 06:23, 1 October 2009 (UTC)[reply]

moment of inertia

why second moment of inertia is needed —Preceding unsigned comment added by Blossomragav (talkcontribs) 12:14, 29 September 2009 (UTC)[reply]

The second moment of inertia is, apparently, the same thing as the second moment of area: a measure of the resistance of a shape to bending and deflection. Our article goes into a great deal of mathematical detail. TenOfAllTrades(talk) 12:22, 29 September 2009 (UTC)[reply]
...and it is needed in the Euler–Bernoulli beam equation. Gandalf61 (talk) 12:32, 29 September 2009 (UTC)[reply]

Abdominal pain

I think it's safe to say that most people, if not all, have on occasion experienced mild to moderate abdominal pain. If the pain is severe, people would see the doctor, but often the pain is not severe enough and the symptom disappears fast enough that the cause is not investigated. Are there good statistics on the leading causes of non-recurring mild to moderate abdominal pain? (I.e. data on what % of undiagnosed cases are actually caused by some condition X. Some candidates for X that come to mind are food poisoning, constipation, and menstrual cramp.) Physiologically what are the mechanisms for the underlying conditions to manifest as abdominal pain? —Preceding unsigned comment added by 173.49.10.73 (talk) 12:27, 29 September 2009 (UTC)[reply]

Appendicitis is another cause of intermittent abdominal pain. Edison (talk) 13:22, 29 September 2009 (UTC)[reply]
Rather than a trying to list causes here, I'd recommend looking at abdominal pain. Those are mostly disorders that will probably end up getting diagnosed at some point because the symptoms will get severe enough to come to medical attention. I doubt there are any good data on "what % of undiagnosed cases are actually caused by some condition X." If we knew the cause, they wouldn't be undiagnosed, and if the cause of abdominal pain is never investigated, there's no way to know. You might as well just pick numbers out of thin air. --- Medical geneticist (talk) 13:41, 29 September 2009 (UTC)[reply]
(ec)So is pregnancy. But it's safe to say that there are no good statistics about causes of pains that are often not reported or investigated. Cuddlyable3 (talk) 13:43, 29 September 2009 (UTC)[reply]
Is that safe to say, though? I would actually be quite surprised if there hadn't been at least some study of 'everyday aches and pains', including studies targeting mild or idiopathic abdominal pains. It's the sort of thing that manufacturers of over-the-counter analgesics, anti-inflammatories, constipation, and indigestion remedies are apt to look at quite closely. Those manufacturers would want to know about
  • the incidence of such aches and pains;
  • the frequency and duration of pains that would prompt a person to self-medicate (pains severe enough to attract attention, but not severe enough to lead to a doctor's visit and prescription);
  • the types of pains, by body part, demographic group, underlying medical condition, etc. (to target the advertising campaigns); and
  • the root cause (if any) of the symptom (to know what warnings need to go on the package insert, and to scare away lawsuits).
While it's out of my area of expertise (and I don't have time to do extensive digging at the moment), I would be shocked if there weren't studies where, for example, participants were required to keep a log of all their aches and pains for a period of time, and where clinicians made efforts to fully diagnose every little twinge. If the sample group were properly selected, there's no reason not to generalize the results out to the general population. TenOfAllTrades(talk) 14:28, 29 September 2009 (UTC)[reply]
Those would be pains that are investigated. Cuddlyable3 (talk) 18:43, 29 September 2009 (UTC)[reply]
But they are pains that are "often not reported or investigated" — and I've offered at least a partial response to the original poster's question. TenOfAllTrades(talk) 19:58, 29 September 2009 (UTC)[reply]
Well, by far the most common cause of abdominal pain is indigestion. That's why Rolaids and Tums are such big products. Looie496 (talk) 23:51, 1 October 2009 (UTC)[reply]

Force Practice

I have no clue where to begin! "Two horizontal forces, 225 N and 165 N, are exerted on a canoe. If these forces are applied in the same direction, find the net horizontal force on the canoe." Accdude92 (talk) (sign) 13:19, 29 September 2009 (UTC)[reply]

Addition. Cuddlyable3 (talk) 13:36, 29 September 2009 (UTC)[reply]
What you want to do here is add the force vectors. Since we're only concerned with horizontal forces there's only one axis to be concerned with. If they're both in the same direction, for simplicity you can call that direction the "positive" direction. So you just need to add +225N and +165N. 13:40, 29 September 2009 (UTC)
Ok what about "If the same two forces as in the previous problem are exerted on the canoe in opposite directions, what is the net horizontal force on the canore? Be sure to indicate the direction of the net force."
Also, i am not asking for the answer, I just really dont understand this.Accdude92 (talk) (sign) 14:03, 29 September 2009 (UTC)[reply]
You should ask your teacher if you're incapable of doing your own homework. Not trying to be unhelpful - they're there to help you! They also have access to diagrams, which I don't. Vimescarrot (talk) 14:17, 29 September 2009 (UTC)[reply]

Try thinking of it less abstractly. Let's say Person A and person B push things. If they both push the same direction, what happens? If they push in opposite directions, what happens? ~ Amory (usertalkcontribs) 14:33, 29 September 2009 (UTC)[reply]

It might be unintuitive, even to somebody who is thoroughly trained in physics - but forces add linearly. For lack of a better reason, we define forces to be "that conceptual thing which adds linearly to equal the acceleration, scaled by the inertial mass." So, if two workers were pulling at 225 and 165 newtons, the net force would be the sum. Maintaining a constant force, while another worker pulls with a separate force, is a technical or engineering challenge; but for simple physics problems, just assume that everything is attached by non-elastic ropes that are tensioned by perfect 165.000 newtons of force. Nimur (talk) 14:59, 29 September 2009 (UTC)[reply]
Except one of the ropes has 225 N tension.

Forget the canoe. Forget the horizontal. The heart of the question says: If two forces of strengths A and B are applied in the same direction, what is the net force?" The answer to that is "Force A+B in the aforementioned direction." The solution to your problem is to begin by reintroducing what I asked you to forget, and to include the units of force N = Newton after the values of A and B. Cuddlyable3 (talk) 18:38, 29 September 2009 (UTC)[reply]

Oral collagen

Why is collagen not hydrolysed when taken orally? I assumed all proteins had an oral bioavailability of 0. --Mark PEA (talk) 15:49, 29 September 2009 (UTC)[reply]

As per the collagen article: "Collagen is also sold commercially as a joint mobility supplement. Because proteins are broken down into amino acids before absorption, there is no reason for orally ingested collagen to affect connective tissue in the body, except through the effect of individual amino acid supplementation.". In other words, it doesn't really do what it's being sold to do. The fraud is kept within legal bounds by careful wording on the package. Red Act (talk) 17:00, 29 September 2009 (UTC)[reply]

Collagen is indeed hydrolyzed and digested (albeit this study was in rats and snakes). This study: AKG Jones, 1986. Fish bone survival in the digestive systems of pig, dog and man in D. Brinkhuizen & A Clason (eds.) Fish and Archaeology. Oxford, British Archaeological Reports International Series 294: 53–61, unfortunately not available online demonstrates this in humans. By the way, colostrum contains immunoglobulins that are able to cross the gut barrier in infants (mammals). Axl ¤ [Talk] 18:25, 29 September 2009 (UTC)[reply]

Electricity in a house

Why are there two wires for supply of household electricity?Since only one of them is required the other being the earth itself? —Preceding unsigned comment added by 113.199.176.92 (talk) 16:41, 29 September 2009 (UTC)[reply]

There are live, neutral and earth. Live and neutral are the ones that enter the house from outside - you need two wires in order to have a complete circuit. Earth is only there for emergencies and is usually earthed at the house itself, so there won't be a wire entering the house for it. --Tango (talk) 16:44, 29 September 2009 (UTC)[reply]
Our article on mains electricity explains this quite well.--Shantavira|feed me 17:11, 29 September 2009 (UTC)[reply]
Earth has some resistance as a conductor and if it were used as a return path for household electricity there would be problems with voltage loss at any distance from the generator, ground heating by high current close to the generator and disturbance to equipment that relies on a signal ground. Cuddlyable3 (talk) 18:16, 29 September 2009 (UTC)[reply]
If earth were used as a return path for household electricity, equipment that uses half-wave rectification of the supply would pass a DC into an earthing rod which would corrode it. Cuddlyable3 (talk) 18:22, 29 September 2009 (UTC)[reply]
Interesting point about half wave rectification. I wondoer how many equipments now still use it?--Tappet (talk) 21:22, 29 September 2009 (UTC)[reply]
See Earthing systems for the ground rules--Tappet (talk) 21:27, 29 September 2009 (UTC)[reply]
Single-wire earth return does exist - but although it cuts distribution wire construction costs, it brings significant extra costs in terms of efficiency loss, safety hazards (and costs to mitigate those hazards), and power quality (e.g. poor power factor correction capability). Nimur (talk) 21:01, 29 September 2009 (UTC)[reply]
Single-wire earth return is possible for high voltage (= low current) distribution using isolating transformers but the OP asks about household electricity which I take to be 240 or 120 VAC. Many houses on Earth are not on earth, meaning that a house built on a rock outcrop has no earth return. Cuddlyable3 (talk) 23:35, 30 September 2009 (UTC)[reply]

Detox, heavy metals in food

Although I despise the whole New Age nonsense of detox, I do believe that certain foods - like fish - could be contaminated with heavy metals, like mercury; and if a person eats fish regularly, these heavy metals will accumulate in his body. Is this true? If yes, is there a way of detoxing our body? --Quest09 (talk) 17:15, 29 September 2009 (UTC)[reply]

Yes, eating too much fish is the most common way to wind up with mercury poisoning. My niece got murcury poisoning after eating large amounts of tuna fish over a half-year period. See the mercury poisoning article for how mercury poisoning is treated. Also see mercury in fish. Red Act (talk) 17:24, 29 September 2009 (UTC)[reply]
The process is called bioaccumulation. That said, there's no real way to detox yourself. You can treat acute poisonings with chelating agents and similar, but these new-agey things are all bogus. Actually 'new age' is perhaps anachronistic, since I think most of those ideas go back to the late 19th century (but even before that you had 'purging' of various sorts, leeches, etc.) Way back before bioaccumulation was known as scientific fact. --Pykk (talk) 18:24, 29 September 2009 (UTC)[reply]
According to bio accumulation, that works because the mercury is stored in fat, so are there any publications suggesting liposuction as a treatment? Googlemeister (talk) 19:57, 29 September 2009 (UTC)[reply]
Never heard of that, but the worst way to get rid of all the fat-loving toxines, like mercury, DDT and dioxines is breast feeding. Not very nice for the baby but, if you loose nearly all your bodyfat during the breast feeding time most of the toxines will end up with the baby. I read the warning that mothers were warned not to go onto a strict fat free diet during breast feeding. So --Stone (talk) 21:30, 29 September 2009 (UTC)[reply]
Well, how can I get rid of my toxines if I have no baby and I am not even female? I suppose that when I get fat and make a diet afterwards, I will be loosing mercury along the fat, wouldn't I? And which mercury is the real danger for us? The mercury stored in fat or the mercury that reaches other organs - like the brain? Or will the mercury stored in fat someday be released and reach other organ where it can cause real damage? Quest09 (talk) 15:30, 30 September 2009 (UTC)[reply]
The problem not being female is not solvable. If you go on a diet you do not loose the fat loving toxins, as there is no natural way that your body will give away the highly caloric fats. The evolution made it the way that you use them for long term storage. The diet will mobilize the toxines, and they will go wandering around in your body, and it is possible too get really ill by the toxines, especially if they are reach other organs like the brain or the liver. The water soluable toxines are easier to deal with they normaly go with the urin.--Stone (talk) 18:16, 30 September 2009 (UTC)[reply]
The best way to get rid of the toxines is not to eat them.--Stone (talk) 18:17, 30 September 2009 (UTC)[reply]
Just FYI, the usual spelling in English (whether American or British) is toxin, with no e. Wiktionary lists toxine in Dutch and French. --Trovatore (talk) 19:15, 30 September 2009 (UTC)[reply]
Is it just me or did someone else expect Stone to mention male lactation? --antilivedT | C | G 07:24, 1 October 2009 (UTC)[reply]

Cast Iron History

The unseasonable drought has revealed some cast iron structures in some of what is normally moat in my garden. I am not sure if they are certainly modern or possibly historic (in the context of my garden by historic I mean pre Black Death). I obviously know that Iron dates to the Iron age but can anyone give a guide to e.g. when they might have had blacksmiths making squared iron rod, and how much was used in practice, and how much survives today? There is a lot of iron in old buildings but I am not sure if it is a later add on. The fact this has survived (ish) underwater may or may not say something about the quality but it looks like it might be a grid for separating part of the moat into a kitchen pond (for carp to be eaten soon) or similar. Does much medieval iron work still exist? --BozMo talk 20:53, 29 September 2009 (UTC)[reply]

Sure, for instance.. I'm not sure what the proper English architectural term is, but Germans call them (beam-)anchors. I'm talking about the iron brackets which anchored the wooden floor-beams to the load-bearing wall in medieval buildings. They're usually still in place, they're even used to date buildings sometimes. I don't think metal quality would affect how long it'd last underwater as much as the oxygen conditions. If there's little oxygen, it could probably last quite some time. (the iron bits of the Vasa were intact after over 400 years underwater). --Pykk (talk) 23:00, 29 September 2009 (UTC)[reply]
Anchor plate (and other names [24] ) common on medieval (and newer) buildings - not sure of medieval ones - but they must have existed.83.100.251.196 (talk) 00:15, 30 September 2009 (UTC)[reply]
Pattress plate is the UK term [25]
History_of_ferrous_metallurgy#Medieval_to_modern_Europe All the blacksmith needs to do is hammer it out. Consider medieval portcullises. Also consider medieval iron hinges [26] etc - well within the abilities from the 10th Century and beyond, and probably before.
There are examples of using iron bar in construction pre black death eg [27] p255 footnote 2.
Also consider the production of swords - which requires iron bars. There's still a lot of iron work from the medieval period surviving in churches (eg [28] 11th or 12th century ironwork for doors), as for more everyday stuff - I don't know.83.100.251.196 (talk) 23:44, 29 September 2009 (UTC)[reply]
What is the thing you found? Is it some sort of grill - letting little fish out?83.100.251.196 (talk) 23:45, 29 September 2009 (UTC)[reply]
Yes, some sort of grill I think but it is mainly buried in sludge now. that area has been abandoned for a very long time --BozMo talk 05:36, 30 September 2009 (UTC)[reply]
As an aside, I think it's probably not a medieval fish grill, simply because of the expense of iron in those times meant it was mostly used where other materials would not suffice - some sort of wicker (or water reed) woven fish trap would seem like a cheaper alternative.
Additionally your local (county) archaelogy department would be interested, and I think investigate these things if you tell them - they will almost certainly have a look for free, and should be able to tell of it's origins. Suggest your local county council website an look for archaelogy. 83.100.251.196 (talk) 23:29, 30 September 2009 (UTC)[reply]
It might be a wrought iron gridiron or grating and have been thrown in the ditch when it was no longer needed or useful. Edison (talk) 04:54, 2 October 2009 (UTC)[reply]

Sunburn

Can dark skinned people suffer from sunburn? How do they knew they have it. what effects does it have . Are they more susceptible to skin cancer/--Tappet (talk) 21:42, 29 September 2009 (UTC)[reply]

They definitely can, and it hurts just like for white folks. I can't tell you anything about their probability of getting skin cancer compared with whites. →Baseball Bugs What's up, Doc? carrots 21:45, 29 September 2009 (UTC)[reply]
YES, I'm amongst them!!!--Leon (talk) 17:04, 30 September 2009 (UTC)[reply]
How about an actual reference here. From our article melanin:
Dark-skinned people, who produce more skin-protecting eumelanin, have a greater protection against sunburn and the development of melanoma, a potentially deadly form of skin cancer, as well as other health problems related to exposure to strong solar radiation, including the photodegradation of certain vitamins such as riboflavins, carotenoids, tocopherol, and folate.
Comet Tuttle (talk) 22:37, 29 September 2009 (UTC)[reply]
Greater protection, yes, but not absolute protection. →Baseball Bugs What's up, Doc? carrots 23:18, 29 September 2009 (UTC)[reply]

You can tell the exact same way you would for a light-skinned person - their skin will be darker. It may be a less noticeable contrast, but the skin will most definitely be darker (tan) or redder (burned). If you have any friends or acquaintances you are close with, it should be obvious. ~ Amory (usertalkcontribs) 00:09, 30 September 2009 (UTC)[reply]

To be entirely accurate it must depend on exactly how dark skinned they are. Where I used to live in West Africa the range of skin tone went through deep browns to totally coal black. The only ones who got burned (pretty much on the equator) were Albinos or ones who were brown enough to mean they were teased for being "Metisse". --BozMo talk 08:32, 30 September 2009 (UTC)[reply]

who coined the term anti matter ?

lately , ive been arguing with a friend of mine who thinks anti matter is an esoteric concept and i was trying to find the name of the scientist who coined the term but im not sure wich of the scientist mentioned in the article on the subject actualy coined it

thanks for the help —Preceding unsigned comment added by 72.0.206.235 (talk) 23:39, 29 September 2009 (UTC)[reply]

I suspect it was Paul Dirac in 1928 because it was his discovery of the Dirac equation that predicted the positron - which is antimatter. However, he first called the positron a 'hole' - so it's not clear whether the term antimatter originates with him. SteveBaker (talk) 00:55, 30 September 2009 (UTC)[reply]
The word "positron" was coined by Carl D. Anderson, who was the person to experimentally discover the predicted positrons (the first known form of antimatter), in 1932. However, I don't know if Anderson used the term "antimatter", either. Red Act (talk) 01:14, 30 September 2009 (UTC)[reply]
According to the OED, Dirac was using the term "anti-electron" by 1931, before Anderson's discovery. The OED has no quotes for "anti-matter" before 1953. Algebraist 01:21, 30 September 2009 (UTC)[reply]

The OED Supplement has a combined entry for all uses of anti- in reference to antimatter. This item would have been compiled in the 1960s; the OED Online might have a newer version. In the Supplement, the earliest cited usage is indeed by Dirac, in a 1931 paper: "We may call such a particle an anti-electron". The first cite shown for "antiparticle" and "antineutrino" is from 1934, for "antiproton and "antineutron" from 1942, for "anti-nucleon" from 1946, and for "anti-matter" from 1953. The writers' names are not given for any of these cites except the 1931 one.

However, because it is a combined entry, all the cites are just illustrating various uses of "anti-"; they can't be assumed to be saying that "antimatter" was not used before 1953, only that it was not used (that they know about) before 1931.

--Anonymous, 01:25 UTC, September 30, 2009.

That's one of those tricky ones though - having coined the 'anti-' prefix for the electron - then Dirac had set the terminology for those who came later - nobody was going to call the anti-proton the "unproton" or the "opposite-proton" or (god forbid) the "notorp". So the words anti-proton and so forth were effectively decided by Dirac's choice of 'anti-' for the positron (although it's kinda ironic that few people talk about 'anti-electrons' anymore). So when you're talking about opposite matter in general, the term 'anti-matter' was a forgone conclusion. If Dirac himself didn't actually use the term - he did (in a sense) coin it by forcing the future choice of vocabulary. SteveBaker (talk) 02:03, 30 September 2009 (UTC)[reply]
Your IP address indicates you might be in Montreal. If your friend is there, too, why not invite him/her to go to the Ville Marie Nuclear Imaging and PET/CT Center, 2345 rue Guy downtown and take a look at their positron emission tomography scanner, which shoots antimatter into people's brains! It can't be too esoteric when most large metropolitan areas in the industrialized world have such facilities. --Sean 16:06, 30 September 2009 (UTC)[reply]
Actually, in PET, the antimatter comes out of people's brains. What goes in is a radioactive substance that emits positrons (hence positron emission tomography). --Trovatore (talk) 21:30, 30 September 2009 (UTC)[reply]


I looked in Google Book Search for the first usage, but they are useless, between incorrect dates on scanned works (they typically use the first year a journal was published as the year of publication for an article many decades later) and everything being "no preview." You have to check the actual copyright page. Google Scholar has a source stating that the term "antimatter" was coined by Sir Arthur Schuster in 1898, and as support offers a website [29] as support. This in turn cites "Quantum Generations: A History of Physics in the Twentieth Century" by Helge Kragh, Princeton University Press, 1999, and links to the publisher's website, which is the source for the Schuster 1898 credit. I could not find at Google Book search an 1898 publication by Schuster. Schuster was the predecessor of Rutherford as Langworthy Professor, supported Einstein's Special Relativity in lectures in 1908. He was vice president of the Royal Society 1919-1920). He lived until 1934 and published a fair amount, so his coining of the term, even in a period before Dirac's theories, might have been known to the next generation of physicists.Edison (talk) 17:39, 30 September 2009 (UTC)[reply]
Schuster is credited with publishing about negative matter in Nature in 1898 in Gordon Fraser's "Antimatter-the ultimate mirror (2000). Fraser says (p3)that "almost everyone had forgotten" Schuster's 1898 letter when Dirac predicted duality in 1927, but that the duality was what Schuster "had suggested." So perhaps someone could find the 1898 letter and cite it in the Schuster article, which does not credit him with anything related to antimatter. "Cosmology: the science of the universe" by Edward Robert Harrison says (p433) that the 1898 writing was the first speculation about antimatter. Schuster is quoted by Harrison as concluding his 1898 Nature letter: "Astronomy, the oldest and most juvenile of the sciences, may still have some surprises in store. May antimatter be commended to its care!" Another source says he published two letters on antimatter in Nature in 1898. There is no mention of Schuster in the Antimatter article, but various books and articles credit him with coining the term as well as the concept, although his hypothetical antimatter would have repelled normal matter. Edison (talk) 18:13, 30 September 2009 (UTC)[reply]
Interesting find. I fixed your Schuster link, btw. Sounds like it would be worthwhile adding something about this to his article. --Anon, 18:21 UTC, September 30, 2009.
I added Schuster's hypothesis to Antimatter and to Arthur Schuster. Those skilled in wordsmithing and/or theoretical physics might take a look and see if it needs any tweaking. We still need an exact citation for the Nature letters of 1898. Edison (talk) 18:36, 30 September 2009 (UTC)[reply]
Found one of his 1898 letters free at Google Books: [30] Schuster, Arthur, "Potential matter: a holiday dream." Nature, 58, 367 (18 August 1898) doi:10.1038/058367a0. Pretty different from 20th century predictions of the properties of antimatter. Here is the other letter, in which he credits Karl Pearson with prior work which implies "negative matter." Edison (talk) 18:53, 30 September 2009 (UTC)[reply]
The OED online has not picked up the Schuster usage. They still have Dirac's 1931 usage of anti-electron as first cite, with 1953 being their earliest usages of anti-matter (in the NY Times on October 23rd, and slightly earlier in "Sci. News Let." on March 14th". The OED last updated this section in 1989. Have they not heard of Schuster, or did he never use the term antimatter? Dbfirs 09:26, 1 October 2009 (UTC)[reply]
Click on the link I provided, and you will see that Schuster definitely used the term "antimatter" in the 1898 letter to Nature. OED should correct their error. Edison (talk) 04:52, 2 October 2009 (UTC)[reply]
I clicked on both your letter links but no preview was available. The search was for "potential matter", not "antimatter". The Google scholar link leads to a dead source link. Only the claim at antimatterenergy.com supports use of the word "antimatter" and I suspect that this is a later interpretation. Dbfirs 11:57, 2 October 2009 (UTC)[reply]
Schuster wasn't talking about the same thing, and neither his theory or terminology ever got any widespread support. So it's your error, not the OED's error. Isolated cases of usage of a word don't make it into dictionaries, which are supposed to reflect common usage. Unless there's some evidence that the later usage of 'anti-matter' borrowed the term from Schuster, then it's irrelevant trivia. --Pykk (talk) 06:04, 2 October 2009 (UTC)[reply]

thank you so much for all the effort , this is very helpfull —Preceding unsigned comment added by 72.0.206.235 (talk) 22:58, 2 October 2009 (UTC)[reply]

Hunger Mechanism

There was a question several days ago which mentioned being IV-Fed all the nutrients a human would need. I'm wondering, if someone was hooked up to this, would they still feel hunger?

That is, is hunger a sensation of needing nutrients or does the stomach need a physical digestive process to satiate the hunger feeling?216.15.125.41 (talk) 23:50, 29 September 2009 (UTC)[reply]

Hunger is a sensation due to a specific neurotransmitter that sends messages from the stomach - I'm assuming - to the brain that tells it "enough." I'm not exactly sure what the "hunger-stopping neurotransmitter" is, but I know for sure that hunger is caused and stopped by a transmitter. Some obese people, or people that cannot stop eating, lack this, which is the reason for their uncontrollable consumption of food. About the IV-Fed, theoretically, if this IV triggered a specific element to send these messages to the brain, then yes; likewise, much like endorphins and pain, it could be our own sensation, caused my the taste of a specific food that release these "hunger-stopping neurotransmitters" to the brain. Hope this somewhat helped. 74.184.100.154 (talk) 01:01, 30 September 2009 (UTC)[reply]
See Hunger#Biological_mechanisms, Ghrelin and Leptin. Fences&Windows 02:45, 30 September 2009 (UTC)[reply]
OR: I experienced a saline drip that made me feel "over watered" i.e. the opposite of thirsty. Cuddlyable3 (talk) 08:21, 30 September 2009 (UTC)[reply]
Tube feeding is more common that IV feeding. Experiments have been done with volunteers who swallowed a balloon attached to a feeding tube, so that the stomach could be filled to varying degrees with food which the body did not get to digest, to see if the subject felt less hungry with a full stomach. Satiety is a function of several factors, including the experience of seeing, smelling, chewing, tasting and swallowing the food. Unfortunately I can't find the book which summarized the results, but I recall that nondigestible stomach filling did not eliminate hunger. For some reason they do IV feeding for premature babies, but not for grownups who are dying and wasting away from something like COPD. They just give IV water and electrolytes, but not enough calories to maintain weight along with vitamins, protein and minerals like they do the premies. Edison (talk) 17:24, 30 September 2009 (UTC)[reply]

September 30

Molarity - Cd2+ Ions, Chemistry Help?

Hey, I've looked everywhere, can't find anything to help me. Does anyone know how to solve the following problem?

What mass in grams of NaOH is needed to precipitate the Cd2+ ions from 64.0 mL of 0.500 M Cd(NO3)2 solution?

Please help, Thanks 74.184.100.154 (talk) 00:31, 30 September 2009 (UTC)[reply]

Cadmium hydroxide has two OH per Cd . So you need twice as many moles of NaOH as you have of Cadmium. Can you workout how much cadmium you have?83.100.251.196 (talk) 00:38, 30 September 2009 (UTC)[reply]
I got .032 moles for Cd, so that means .064 moles for OH, times the molar mass of NaOH - 39.9971 - gives you 2.18; what did I do wrong? 74.184.100.154 (talk) 00:42, 30 September 2009 (UTC)[reply]
Seems ok - why do you think it's wrong?Shortfatlad (talk) 01:01, 30 September 2009 (UTC)[reply]
Because the online "checker" says it is! That is 2.18 grams to clarify, any suggestions? 74.184.100.154 (talk) 01:03, 30 September 2009 (UTC)[reply]
What online checker?83.100.251.196 (talk) 01:40, 30 September 2009 (UTC)[reply]
Try multiplying 0.064 x 39.9971 again - it's ~2.5

83.100.251.196 (talk) 01:42, 30 September 2009 (UTC)[reply]

Thanks, that's correct, multiplication failure. 74.184.100.154 (talk) 02:02, 30 September 2009 (UTC)[reply]

Has Mendel Sachs' theory on unifying gravity and quantum theory been well accepted?

The Wikipedia article on Mendel Sachs indicates that he has "...by all accounts, completed Albert Einstein's unified field theory...' If this is the case, we should have heard much more about this physicist. Has his theory been challenged?67.250.125.166 (talk) 00:47, 30 September 2009 (UTC)[reply]

As noted at unified field theory, "There is no accepted unified field theory yet, and this remains an open line of research." I suspect that the Mendel Sachs article simply has inflated claims. Unfortunately, the lack of online citations makes it difficult to verify what's supposedly been cited. Regardless, it seems safe to say that his claims are not validated "by all accounts". — Lomn 01:08, 30 September 2009 (UTC)[reply]
Never 'eard of him. Nobody else is talking about him. He's published some articles and books, but if anyone really thought he had come up with a unified field theory that worked, there would be discussion of it. Sounds a wee bit crankish to me. He's certainly not recognized as having unified physics. --Mr.98 (talk) 17:39, 30 September 2009 (UTC)[reply]

Boiler insulation

We have a 120 tph high pressure CFBC boiler.It is a common practice to fix insulating mattresses at the outer surface of the boiler casing with the help of supporting lugs. The question is these lugs are welded on boiler tube and not on fins, why? Isnt it should be on the fins instead of pressure parts? —Preceding unsigned comment added by NTR1234 (talkcontribs) 02:38, 30 September 2009 (UTC)[reply]

Boilers are prone to blowing up, killing people, and destroying things, if mismanaged in any detail, so nothing presented here can be taken as advice on boiler management. Please consult the manufacturer, a boiler engineer or reference books on boilers. Some books which cover CFBC insulation can be seen at Google Book search, here. Edison (talk) 17:11, 30 September 2009 (UTC)[reply]

Herds of rabbits

This question was posted at Talk:Rabbit:

I have personally witnessed rabbits congregating in numbers exceeding 100. Has anyone any information on rabbits "herdiing" behavior? - Rog —Preceding unsigned comment added by 71.165.78.38 (talk) 22:30, 29 September 2009 (UTC)

Fences&Windows 02:41, 30 September 2009 (UTC)[reply]

There's a considerable body of research on Rabbit behavior [31] [32], but I can't find anything that specifically mentions "herding" in the abstracts or at least the snipets that google gives. I'm probably just not searching the right keywords, although you can use the word "communal" and you get some papers that might answer the question. I don't have access to the journals rabbit ecologists publish in, however, so I'm not certain. Someguy1221 (talk) 05:48, 30 September 2009 (UTC)[reply]
Since rabbits are timid, sexy and hungry (in that order?) they don't stray far from a safe hole, other rabbits and a place to eat. Cuddlyable3 (talk) 08:18, 30 September 2009 (UTC)[reply]
The right search terms are "gregarious", "social" and "group size". Grouping seems to be a predator defence:[33], as it is in many species, but group living vs grazing doesn't seem to be an advantage in rabbits:[34]. Here's a good summary of rabbit sociality:[35][36]. It seems they are territorial and don't interact much above ground, so a rabbit 'herd' isn't usual. There is probably a large warren system there, but the rabbits won't be 'herding' as such. Fences&Windows 20:22, 30 September 2009 (UTC)[reply]
There are many species of rabbits throughout the world. The cottontail rabbit that I am most familiar with in the US does not congregate in such herds. At most you might have a family group hanging out together. Perhaps other species congregate in such a fashion. Googlemeister (talk) 13:14, 1 October 2009 (UTC)[reply]

Thermal death points of different species (and genera and subfamilies) of bedbugs

I'm searching for the thermal death points (upper and lower) of bedbugs. The thermal death points of the common bedbug (Cimex lectularius) are usually given as 45C and anywhere from -10C to -32C, depending on duration of exposure. Less extreme temperatures also affect the lifespan and life cycle of bedbugs. However, in every source I find mentioning the effects of climate on bedbugs, the focus is invariably on the common bedbug, with little or no mention of any other subfamily, genus, or species (and there are more than 100 species). Presumably, other species (such as the tropical bedbug, C. hemipterus) have different temperature requirements for reproduction and higher or lower thermal death points. The question is, what are the numbers?

(This issue has serious implications for pest control.) —Preceding unsigned comment added by 125.26.218.54 (talk) 05:24, 30 September 2009 (UTC)[reply]

I don't think there is such a thing as a thermal death point. There is a Thermal death time for microorganisms, but no single thermal death point for a living organism. There are two reasons for that. First, surrounding air temperature does not equal body temperature for a macroscopic organism. For example, humans can survive a heat of 80-100 C in a sauna with little or no adverse effect, but raise a human core temperature to 50 C or more and he or she will promptly die. Insects, too, can have core temperature much below the surrounding air temperature. I have seen Adesmia abbreviata beetles running around in a blistering heat, in full sun, with no ill effect, simply because there is an insulating gap between their wing-covers and their bodies. The second reason is that death from overheating is not instant. The stronger is overheating, the faster the death occurs. Hope this helps. --Dr Dima (talk) 21:04, 30 September 2009 (UTC)[reply]

Body Heat and Energy Generation

How does the body generate heat? It has been suggested by some that heat is generated by the burning of food, by the individual cells and so on. For food to burn, heat to burn the food must already exist. For cells to generate heat there must be a heat source, an energy source. For the life in the body to continue there must be an energy supply within the body. Nothing can come from nothing so please what generates the heat in the body. It must be the body itself that does it but how? The body does enormous amount of work, moving, lifting, walking, digestion, talking, thinking, all the emotions etc. The heat just to keep the body at the same temperature continuously must be considerable. When we sleep we recover energy from sleep, we do nothing, do not eat, drink etc,where does this energy come from Koulouriotis (talk) 06:14, 30 September 2009 (UTC)[reply]

Beginning to produce heat does not actually require any initial input of extreme heat (the common experience if lighting a fire or a candle, for instance). But consider that when you strike a match, it bursts into flame, even though neither the match nor the matchbox are any hotter than room temperature. The flame erupts because the friction between match and matchbox triggers a chemical reaction that itself releases heat. And chemical reactions are where the body's heat comes from. All of the food that you eat is made up of large, complicated molecules (sugars, fats, and proteins). These are broken down by your body into carbon dioxide through chemical reactions that produce heat. Not only do the reactions produce heat, they are also the indirect energy source for everything your body does. So you're right that an energy source is required to produce heat, but the key concept you missed was that heat is not the only form of energy. Try looking at chemical reaction, energy and cellular respirationfor starters if you're more interested. Someguy1221 (talk) 06:28, 30 September 2009 (UTC)[reply]
So many misconceptions here! Let's try to break it down a little.
  1. When people talk about "burning" food - they are talking metaphorically. We do not literally set light to the stuff and get heat from tiny little flames. Our cells combine the foods with oxygen and produce carbon-dioxide (which we breath out). That chemical reaction is kinda similar to what happens when you set light to something - but it's all done without flames. The energy produced by that chemical reaction is what provides the heat. Other reactions produce chemicals such as Adenosine triphosphate (ATP) which is what drives our muscles to contract. ATP is therefore an energy source made from food.
  2. We do not "recover energy" as we sleep. That's a common misconception. Our bodies continue to process whatever food is in our system - which continues to generate energy - but that's the same basic activity as is going on while we are awake. There is no special energy recovery thing going on while we sleep. Mostly, sleep is to allow our brains to reorganize themselves. Animals without fairly large brains don't sleep. Some animals (dolphins, some birds) sleep with only one half of their brain at a time(!) - so, for exampe, dolphins can keep swimming all day and all night without "resting their bodies to recover energy"...if they ever fell totally asleep - they'd probably drown! But they can't avoid the need for their brains to take a "time-out" - so they sleep with half their brains at a time so that the other half can keep them swimming and coming up to the surface for air.
  3. There are enough fats and other stored energy supplies in our bodies to enable us to continue to generate heat, produce muscular activity and run our brains - for many days after we run out of food in our digestive system. People have been known live for maybe 70 to 80 days without eating any solid foods whatever - continuing to maintain bodily function on stored fat supplies and almost nothing else.
SteveBaker (talk) 12:00, 30 September 2009 (UTC)[reply]
Here are some other references the OP might find useful: basal metabolic rate, electron transport chain, exothermic reaction, thermogenesis, temperature regulation. The simple answer is that your body's utilization of food to make "energy" (see Steve's post above) generates heat as a by-product of all those enzymatic reactions. The basal metabolic rate is a main determining factor of body temperature and for the most part provides enough heat to maintain normal body temperature. The body has the capability to release excess heat (typically through sweating), and to prevent heat loss in the cold (through constriction of the blood vessels on the surface of the skin). If the body gets too cold, shivering is a way to increase muscle activity to generate heat. In certain situations (newborn babies and hibernating animals), special types of fat called brown adipose tissue actually uncouples the process of oxidative phosphorylation in order to generate heat (see thermogenin). --- Medical geneticist (talk) 14:56, 30 September 2009 (UTC)[reply]
The questioner is right in there with scientists of the 1840's and the question of "animal heat." German physiologists wondered if there was a "vital force" in living things, besides them operating like little locomotive burning fuel to move around and keep warm. Helmholtz was a leading experimentalist, and determined (with only a little handwaving) that the heat ingested by an animal in the form of food, equalled the heat, work and combustion products produced. At that time the mechanical equivalent of heat was not well understood, and his experiments and theoretical writing argued that work was neither created nor destroyed, which was still an open question and which led others to posit vital forces or perpetual motion. Concepts which are quickly passed over as obvious in introductory science books were the battleground of the greatest scientific minds a few generations ago. Edison (talk) 17:00, 30 September 2009 (UTC)[reply]
also the concepts of catalysis and enzymes in reducing the activation energy of reactions.Gzuckier (talk) 19:59, 30 September 2009 (UTC)[reply]

What is a "broken" nose?

I read that the nose is actually bartillage; it's sure not bone. So, why do we say someone has a "broken" one? Is it just weird word usage (meaning this should be for the Language desk)? Is it the sinus cavity that really gets broken? And, if the nose is cartillage, is it posslbe to get a small rip in it, like one reads of athletes having cartillage problems in knnes and shoulders? Thanks. (Edit: That last could explain why one's nose gets red after blowing it 8,123 times during a bad cold :-); perhaps it does get irritated like an athlete's knee, etc.)4.68.248.130 (talk) 12:15, 30 September 2009 (UTC)[reply]

Is there anything that Broken nose doesn't answer? Nil Einne (talk) 12:24, 30 September 2009 (UTC)[reply]
Broken nose doesn't really have much detail. Nasal bone has a little more detail on the bone(s) that might be affected. --LarryMac | Talk 12:35, 30 September 2009 (UTC)[reply]
Nasal septum deviation is more common (from what I've seen in life) than a "broken" nose. This usually refers to damage to the cartilage, not the bone. As for it is referred to as "broken" and not "torn" as in other uses of damaged cartilage, I would say it is because it is not cartilage in a muscular system. It is cartilage in a bone system. You tear muscles. You break bones. -- kainaw 12:48, 30 September 2009 (UTC)[reply]
I see; that's where I was confused. I hadn't known there were two types of cartilage. (And didn't realize the nose had bones.)4.68.248.130 (talk) 13:04, 30 September 2009 (UTC)[reply]
There's actually 3 types of cartilage: hyaline, elastic and fibrocartilage. DRosenbach (Talk | Contribs) 14:46, 30 September 2009 (UTC)[reply]
I do like the concept of "bartilage", though. Gzuckier (talk) 20:01, 30 September 2009 (UTC)[reply]

Question in inorganic chem,help me please?

i have read in a book that the electopositivety of group 3 increases from boron to aluminium and then decreases from gallium to indium and thallium.although i suppose it should be lesser in case of aluminium and boron because of its small size and low ionization potential? 1992rajab (talk) 12:59, 30 September 2009 (UTC)[reply]

In general, Electropositivity increases down the group, because of, as you said, increase in size. If the electrons are farther away from the nucleus, they will have less force of attraction and hence are easier to remove. Hence, Al is more electropositive than Boron. However, Gallium and Indium are less electropositive. This is due to the presence of d and f electrons which are not able to screen the valence electrons from the nucleus. This is called Screening effect. Since the d and f orbitals are more diffused, they are unable to protect the valence electrons. I understand that our articles need some work on this topic... Rkr1991 (Wanna chat?) 15:48, 30 September 2009 (UTC)[reply]
That doesn't make sense -- Ga still has less-energetic valence shell (additional p and s orbitals). Al is still quite electronegative, what with its empty p orbital and all, to the extent that it has many analogous reactions with boron. John Riemann Soong (talk) 18:36, 30 September 2009 (UTC)[reply]
It does make sense, and is true. See d-block contraction.
Ben (talk) 21:01, 30 September 2009 (UTC)[reply]

Calories and weight loss

Do we - independent of our body type - need to consume the same amount of calories to burn the same amount of fat? Since fat is a fuel, the difference is that a fat person will consume more fat moving around than a slim person, but if both consume X calories will burn Y fat.--Quest09 (talk) 16:05, 30 September 2009 (UTC)[reply]

See Weight loss and Dieting. The terminology of "consuming calories" to "burn fat" is confusing because it implies the more calories we consume, the more fat we burn. If I consume 2000 calories, and burn 2350 due to the calories burned from basal metabolism plus the calories from increased activity, then there is a 350 calorie deficit per day. A pound of body weight equals about 3500 calories, so I should lose a pound every ten days. If I give up some exercise which burns 350 calories per day, then I should neither gain nor lose weight. People differ in their basal metabolism. Dietitians calculate calories per kilogram to estimate our caloric need. It would take more calories for me to maintain 230 pounds (16 stone 6 lb for Brits, 104 kg for others) than to maintain 180 pounds (12 stone 12 lb, or 82 kg) with the same activity level. So the weight, the activity level, and individual metabolic factors dictate how many calories we need to just maintain our weight. Fewer calories taken in, all things being equal, and we lose. More calories, and we gain. Edison (talk) 16:34, 30 September 2009 (UTC)[reply]
Yes, but imagine if two persons - who weigh 200 and 150 pounds - were eating enough to maintain their weight and they keep the same amount of food intake. Then, both start to do some exercise. The first walks x miles until he burns n calories, the second has to walk a little bit longer, since he has to move less weight consumes less energy to do that, but at the end, he also burns n calories. Will these n calories spent mean the same weight loss? Quest09 (talk) 16:49, 30 September 2009 (UTC)[reply]
Yes, if both have the same energy deficit, that should correspond to the same amount of fat burned. Fat stores about 9 kcal/gram of energy, regardless of how much you weigh (the Adipocyte article mentions that brown fat cells contain a lot of water too, but that brown fat is mostly prevalent in babies). The problem is that the amount of energy used by a person in a day depends on a lot of factors besides just how much exercise they do. Rckrone (talk) 18:34, 30 September 2009 (UTC)[reply]
Theoretically, that is true. However, what constitutes a "calorie deficit" is about impossible to calculate, even for the same person at different times. If I consume 2000 calories/day and start losing weight for a month or two, I may find that a may suddenly start to plateau or even gain weight, with no change in diet or exercise. We can speak in general trends, but to nail down exactly how many calories you need each day to maintain, or lose, or gain weight, and how much calories you "burn" for a given activity, are impossible to work out to any reliability. Eat less, exercise more, and you'll lose weight. How much less you need to eat, and how much more you need to exercise, will need to be worked out for you personally by trial and error and tweaked all the time. --Jayron32 19:18, 30 September 2009 (UTC)[reply]
The question assumes that the thin guy and the heavy guy both burn the same number of calories, by walking different distances, and otherwise eat the amount to exactly maintain weight without the exercise. I would expect them to lose at the same rate. But weight loss in practice, following a medically approved program, often has a rapid initial loss, which dieters call "water weight." It may actually represent the stopping of eating salty treats. The rapid first week loss may be followed by a plateau of no loss or little loss for a couple of weeks, at which point the dieter is likely to quit the program. This always seemed like the body's way of getting the dieter to drop out of the weight loss program and stay fat. Eventually, the weight loss kicks back in, so that the initial rapid loss and plateau are just a blip on the graph. Of course the metabolism could vary over time. Edison (talk) 19:46, 30 September 2009 (UTC)[reply]
Also, different people metabolize their food differently, and the same person may do so over time, too. Some people can eat all they want and won't gain. --Janke | Talk 07:44, 1 October 2009 (UTC)[reply]

Efficiency of a system

The problem in my book is: If an engine does 200J of net work and exhausts 600J of heat per cycle, what is the thermal efficiency?

What I want to do is divide the work done by the heat and get 33.3%. What my book has for an equation in this section is E = 1 - (QC/QH). I don't really understand where all that comes from or what it means. I've tried juggling the numbers around and cannot figure out a way to get the book's answer of 25% efficiency. Any help? Dismas|(talk) 16:30, 30 September 2009 (UTC)[reply]

If 800 units of fuel go into the engine, and 800 Joules of energy go out in the form of work (200J) or waste heat (600J), what proportion of the energy leaving the engine is useful work? The erroneous method you want to use would imply that if it did 400 J or work and had 400J of waste heat, it would be 100% efficient. Edison (talk) 16:38, 30 September 2009 (UTC)[reply]
Ah! Yeah, it was right in front of me. Thanks! Dismas|(talk) 16:42, 30 September 2009 (UTC)[reply]

The engine transform 800J of chemical energy ( I assumed this is a car engine). This consists of 200J of desireable outcome (aka work) and 600J of undesirable outcome (aka waste heat). The efficiency of the engine is "desirable outcome"/"total energy transformed" * 100 %

So the answer is 25% or 200/800 202.147.44.83 (talk) 22:34, 30 September 2009 (UTC)[reply]

Mechanical engineering/automobile design

Im a mechanical engineering student. I want to become an automobile designer, what should do after completing my course? —Preceding unsigned comment added by Salam1166 (talkcontribs) 16:51, 30 September 2009 (UTC)[reply]

I'm a mechanical engineering student as well, and I think you'd be best off first going to the careers service at your college/university, presuming there is one. But, I warn you, engineering is a massive field, and automobile design is massive in itself, which makes it difficult to give more specific advice than this.--Leon (talk) 17:02, 30 September 2009 (UTC)[reply]
I think that the best thing you can do is to get a co-op or internship at an automobile company BEFORE graduating. ike9898 (talk) 17:13, 30 September 2009 (UTC)[reply]
It's worth noting that what Wikipedia considers "automotive design" has only limited applications to mechanical engineering. If you want to design cars, a background that includes art and design will be useful. If you mean that you want to develop engines or safety systems or the like, then ask about courses specific to automotive engineering -- those should overlap with ME nicely in many cases. — Lomn 17:59, 30 September 2009 (UTC)[reply]
You might seriously want to read the news about the automotive industry in your region or country. While there are plenty of jobs still related to the automotive industry, you should become very aware of the major strategic changes which are facing the industry at the present time. For example, in the United States, Detroit (which was the hub of the automotive design and manufacture for nearly a century) is ... (to put it lightly)... in a "transitional phase" at the moment. Our article calls it the Automotive industry crisis of 2008–2009. This will definitely have serious ramifications to your career as an engineer. In the worst case, you may not find work at a major firm. In the best case, you might find that there is a lot of emergence of new automobile corporations - (e.g. Tesla Motors - but even these new companies are struggling). The kinds of engineering skills and technical knowledge which you will need for a future in the automotive industry is very different from what it might have been 25 or 50 years ago. Nimur (talk) 18:44, 30 September 2009 (UTC)[reply]

Any lubricant experts here?

I've got a "lubicator" (a device that injects oil into a compressed air stream) which is supposed to use class 1 turbine oil (ISO VG32). It is connected to an air motor which is supposed to use SAE 10W oil. My question is, how different are these two oils? Considering that I have to choose only one, can you think of a good basis for preferring one over the other? ike9898 (talk) 17:10, 30 September 2009 (UTC)[reply]

Just an aside - injecting hydrocarbon lubricant into a compressed air stream is completing the fire triangle. You have oxygenated air; you have heat energy supplied by the compression heating; and now you have a fuel. Be sure you understand the safety implications of whatever you are doing. It is possible that the manufacturer-lubricant has been safety tested under those conditions. Nimur (talk) 18:55, 30 September 2009 (UTC)[reply]
Agree with the above. I work with industrial compression equipment frequently, and every instance of injecting oil into the process stream I have seen involved very low levels of oxygen <1% in the process gas. Air has a bit more oxygen then that. Safety first and all that. Googlemeister (talk) 19:30, 30 September 2009 (UTC)[reply]

Nuclear fallout discovered theoretically or empirically?

After seeing a nuclear bomb discussion elsewhere, I saw in our Trinity (nuclear test) article the following:

For the actual test, the plutonium-core nuclear device, nicknamed the gadget, was hoisted to the top of a 30-meter (100 ft) tall steel tower for detonation — the height would give a better indication of how the weapon would behave when dropped from an airplane, as detonation in the air would maximize the amount of energy applied directly to the target (as it expanded in a spherical shape), and would generate less nuclear fallout.

I wonder about the fallout thing, though -- did the scientists developing the bomb really consider those effects? Purely military rationales for air bursts were already understood, and what little I've read about Operation Crossroads (the first post-war nuclear tests) suggests that massive fallout from ground bursts came as a surprise. — Lomn 18:31, 30 September 2009 (UTC)[reply]

JUst because there was more fallout than expected from ground bursts does not mean that such ground-based detonations were not expected to have more fallout than air bursts, just that they had even more fallout than expected. --Jayron32 18:34, 30 September 2009 (UTC)[reply]
Sure, I get that distinction. I just note that radioactivity has an illustrious history of unintended consequences. I think there's also a parallel question that would be good for article accuracy: even if the potential for fallout was understood, was that a meaningful factor in deciding on an air burst? Our article implies that it was, though I don't recall ever seeing it discussed either way in other media. — Lomn 19:06, 30 September 2009 (UTC)[reply]
They originally had a poor understanding of the fallout that would come from an air burst, and it was one of the factors involved in planning for an air burst (air bursts are more efficient, anyway, but they were definitely concerned with minimizing fallout). This is one of the reasons that they initially denied Japanese reports of significant numbers of people suffering radiation sickness—they thought that with an air burst, you wouldn't have much ground material sucked in to produce fallout, and anyone who suffered from the radiation emissions of the bomb would have simply been killed by the blast and heat waves (which extended further than the radiation effects). Alas, they were wrong, and it turns out that significant fallout was created in the Japanese blasts—the air burst sucked up significant amounts of debris after all.
They took it into consideration, but didn't really know what the effects would be. At that stage, they were really just guessing, having a sample size of 1. One of the major reasons for testing in the late 1940s and early 1950s was to better understand fallout effects. It is a very empirical sort of effect, and their understanding of the precise effects of fission explosions was very primitive until the late 1940s. --Mr.98 (talk) 01:07, 1 October 2009 (UTC)[reply]

Amines as a leaving group, under acidic conditions

Say an amine is attached to a tertiary carbon (R3C-), and I react it with acid. Shouldn't it be easy to lose the amine and form a carbocation?

Also, are aminals more or less reactive than hemiaminals? Why do they seem to be products whereas hemiaminals are intermediates? And why is it the oxygen and not the nitrogen that is lost?

John Riemann Soong (talk) 19:40, 30 September 2009 (UTC)[reply]

No. But the reverse reaction is a great way of catching carbocations ie
R3N + R'3C+ >>> R'3C-N+ R3
NR3 or NR2H is a good leaving group (as good as Cl- in a simple comparison) - but in this type of reaction the stability of the carbocation is going to be the major factor.
It does have potential though, as any NR3 released will be inactivated as a nucleophile by further protonation.. However activating the NR3+ as a leaving group is difficult since the molecule would need to invert to form a bond with further H+ in a concerted reaction (plus of course problems with getting two positively charge things to interact.
Maybe this is why a common way of getting carbocations is alkyl halide + strong lewis acid trap - eg R-Cl + BCl3 >>> R+ + BCl4- - note the Cl can be activated by interaction with the BCl3 before being lost - NR3 can't do this.
Aminals/hemiaminals - can you say more about the reactions you're thinking of - ie which reaction has O not N lost.
As for reactivity - what sort? for N as a nucleophile it should be fairly straightforward that an aminal is more reactive.83.100.251.196 (talk) 22:58, 30 September 2009 (UTC)[reply]
Do you mean H2O elimination - I suppose since the NH2 is more nucleophilic than OH that it would tend to be the one forming a new bond - leaving the OH as the one to be eliminated.83.100.251.196 (talk) 23:43, 30 September 2009 (UTC)[reply]
What about an allylic or benzylic carbocation? I also don't get your point about structure inversion -- the amine doesn't need to be tertiary, just the potential carbocation. I can't seem to think of any names that would give me examples (prolly since I just have bad IUPAC-naming skills) -- maybe you can help me. John Riemann Soong (talk) 19:51, 1 October 2009 (UTC)[reply]
Ammonium is more stable than oxonium, so it's a worse leaving group. The usual cases of deamination are concerted reactions rather than via carbocation intermediates--there is an additional energetic "push" rather than relying solely on the N+ "pull" to initiate the reaction. You can make carbocation via N+ loss, but you need it as a better leaving group than NH3 or NH3; diazonium (leaves as N2, and additionally made irreversible as it bubbles out of solution) is a good one. DMacks (talk) 04:41, 2 October 2009 (UTC)[reply]
Correction: there is an additional energeticmechanistic "push" and more stable organic product. DMacks (talk) 13:30, 2 October 2009 (UTC)[reply]
If I distilled away the ammonia, couldn't I drive carbocation formation to completion? John Riemann Soong (talk) 22:31, 2 October 2009 (UTC)[reply]

inlands/ocean coast temperaturs

In summer valleys like Inland Empir-San Bernadino gets very hot at summer and very cold at winter, but Coastal cities in orange county-Santa Monica, Long Beach, Newport Beach, San Juan Capistrano stays mild all year. But what about spring and fall, will spring and fall be warmer at orange county coastal, or inland vallies?--209.129.85.4 (talk) 19:41, 30 September 2009 (UTC)[reply]

It can be hard to generalize, due to other considerations such as currents and local terrain, in general coastal locations have less extreme weather due to the presence of water. Water has a high specific heat, so it takes a long time to heat up in the summer and a long time to cool down in the winter. Spring and fall will essentially be somewhere in between whatever winter and summer end up being. In other words, spring and fall temperatures are likely to be further from summer and winter temperatures in inland areas compared to coastal areas. ~ Amory (usertalkcontribs) 21:00, 30 September 2009 (UTC)[reply]

Rottweiler pit bull mix

What kind of temperment would this dog have? --Reticuli88 (talk) 21:21, 30 September 2009 (UTC)[reply]

A lot of it's temperament would be governed by how it is raised and trained. "Show me your dog and I'll tell you who you are" I think Bob Dylan said that. 86.4.186.107 (talk) 21:42, 30 September 2009 (UTC)[reply]
You can't judge a dog by its breed or that breed's reputation. I have seen some full blood pits and many more pit mixes that were very sweet dogs. Same goes for Rotties. The opposite is also true though. I've known some that were raised poorly and were not socialized well enough to meet any strangers. Dismas|(talk) 22:48, 30 September 2009 (UTC)[reply]
I would say that's true that if you do everything right. If you bring up the dog just right - you could end up with an impressive and well-tempered dog. But your room for error is a LOT smaller with such an obviously short-fused animal than with a naturally calm dog like a labrador. If you've raised a bunch of dogs before - and had nothing but calm, relaxed animals - then maybe you could take on this challenge. But if this is your first dog - or if you have kids - or if you don't have the space for them to exercise - or you've had unreliable dogs before (...which would undoubtedly be your fault!)...then I would strongly recommend looking for something with a better reputation. If you have a Shih Tzu go nuts on you - it'll be annoying, but no more. If a Cocker Spaniel goes nuts on you - they have soft mouths - you can definitely handle it. If a labrador goes nuts on you - you can probably handle it - if a German Shepherd goes nuts...it's not going to end well - but if a gigantic Rottweiler-sized Pit-bull goes nuts...your next-door neighbors' bratty kid could get it's throat ripped out. But - if you get everything right, almost any animal can be a good pet. SteveBaker (talk) 01:14, 1 October 2009 (UTC)[reply]
But make sure the dog is legal in whichever jurisdiction you live in. --Tango (talk) 03:24, 1 October 2009 (UTC)[reply]
Mixing breeds generally produces highly variable results. Since Rottweilers and pit bulls have somewhat similar baseline personalities, though, it's most likely that a cross would also be similar. It seems like the question is based on an idea that the aggressiveness might be the sum of the two breeds, but that isn't what happens: a cross on average produces a result about midway between the parents, but often with a lot of unpredictable variability. Looie496 (talk) 22:53, 1 October 2009 (UTC)[reply]

October 1

Pauli Exclusion Principle, Electron Spin, and Magnetism

I am teaching a lot of atomic theory (including quite a bit of quantum mechanics) to my 5th grade gifted and talented students. Even though my BA is in English, I take science very seriously (in a fun way) and really strive to get them excited about science. People seem to assume that kids are largely incapable of learning the really cool, deep science.

That being said. Is magnetism caused by the, "uniform motion of electrons," or is it a result of a large number of electrons spinning in the same direction along axis that are parallel? Is either statement close to the truth? Am I right to assume that Pauli's Exclusion Principle is not violated as long as none of the electrons are occupying the same space, even if they are all spiining the same way?

I apologize for the ignorant oversimplifications, but I'm a self-taught, math-challenged, lover of physics. —Preceding unsigned comment added by 98.169.171.112 (talk) 00:54, 1 October 2009 (UTC)[reply]

Both can cause magnetism. a large number of electrons spinning in the same direction will give you a permanent magnet. ANd a massed flow of electrons will give you an electromagnet. Even in a magnet many of the electrons are spinning in opposite directions, the but there is a surplus in one direction. For most substances the electrons will pari up, with one spinning one way and one spinning another way. It is only in special elements which have unpaired electrons not involved in a bond that you will have a spare electron to have one spin without being counter balanced. These are likely to be the transition elements or rare earth elements. They have a d-shell or f-shell that is "inside" the atom. Someone else can probably explain this better. Graeme Bartlett (talk) 04:46, 1 October 2009 (UTC)[reply]


(ec)Our magnetism article has a section explaining the sources of magnetism in different types of materials (and there are actually several different types of magnetism, each with different electronic origins; even protons and atomic nuclei have magnetic properties, which your students may have experienced in an MRI scan). Lots of complicated stuff in there! But many of the ideas should make some sense (and of course ask specific questions about it if not!).
The Pauli Exclusion Principle says two electrons cannot be the same in all respects. So if you have two different places you are already "different", and the spin can be "same" and not be a problem. This is the origin of the idea of "2 electrons in each atomic orbital": the pair electrons are the same except for spin; two different spin possibilities, so 2 electrons per orbital. DMacks (talk) 04:51, 1 October 2009 (UTC)[reply]
We need to distinguish three different types of magnetism: ferromagnetism, diamagnetism, and paramagnetism. All of these are essentially modeled by a quantum interaction between an electron's intrinsic magnetic moment and an externally applied magnetic field; in the case of ferromagnetism, the field is due to local interactions with other electrons in a magnetic domain (resulitng in a strong feedback and mutual alignment). In general, it is not a clean description to say that the magnetic moment is caused by "uniform motion" of electrons - this implies bulk motion (net measurable current) which contradicts experimental observation. In general though, there is a quantum-mechanical interaction between an electron's orbit description and its magnetic moment, if a magnetic field is present. If the magnetic field is uniform, the effect on all electrons' bulk motions in the material will be a similar alignment - but this is not the cause of the magnetic moment. Even if it were possible to freeze an electron motionless - (which is not actually possible) - that electron would still have an intrinsic magnetic moment. This is a very non-intuitive, non-classical effect - but it has been repeatedly observed by numerous experimental methods. A "stationary" electron is always "spinning" - it can never be stopped from spinning - it has this property built into the definition of an electron and it cannot be removed. Nimur (talk) 06:11, 1 October 2009 (UTC)[reply]
'Spin' is not actually a rotation of the particles. Electron's don't spin about their axis. (They don't even have an axis, what with lacking any kind of volume or internal structure as far as we know.) Two electrons with the same spin can occupy the same space. Two electrons with the same spin cannot occupy the same orbital - which is not a point in space but an entire pattern of motion (although 'motion' is a somewhat ill-defined concept at that level as well), different orbitals do overlap spatially. --Pykk (talk) 05:58, 2 October 2009 (UTC)[reply]
It may be helpful to think of electrons in their orbitals like a three-dimensional standing wave, though the actual term is more properly called a wave function. Bonding theories like Molecular orbital theory basically work on treating bonding like constructive and destructive interference of these waves. I commend you for taking this on with 5th graders; most adults cannot wrap their heads around quantum theory. You need to completely abandon your intuitive understanding of how physics works to really "get" it... --Jayron32 05:26, 3 October 2009 (UTC)[reply]

Garbage

the most hygienic way of handling and disposing garbage —Preceding unsigned comment added by TAMIKA LINDSAY (talkcontribs) 01:16, 1 October 2009 (UTC)[reply]

It depends on the nature of the garbage. See our articles Waste disposal . Intelligentsium 01:20, 1 October 2009 (UTC)[reply]
Annihilation with antimatter. Sagittarian Milky Way (talk) 02:29, 1 October 2009 (UTC)[reply]
I wouldn't advise it - the explosion would scatter the bits that didn't get annihilated all over the place. Not very hygienic. --Tango (talk) 03:22, 1 October 2009 (UTC)[reply]
It also depends on your definition of "hygienic." You can incinerate all traces of garbage "out of existence" if you're willing to release the vapors it produces into the atmosphere. List of solid waste treatment technologies is relevant. Sanitary landfill and incineration are both listed there. See also hazardous waste - final disposal. Nimur (talk) 03:30, 1 October 2009 (UTC)[reply]

The most hygienic manner in which to handle garbage would undoubtedly be with out direct body contact -- you can use impermeable gloves or chopsticks. The most hygienic fashion in which to dispose of garbage would follow suit, making sure whatever you encase the garbage in does no disintegrate until after the garbage disintegrates -- or you could decontaminate the garbage in an autoclave. DRosenbach (Talk | Contribs) 03:42, 1 October 2009 (UTC)[reply]

The most hygienic fashion in which to dispose of garbage (including nuclear waste) would be to load it into a reliable rocket and Set the controls for the heart of the sun. Dbfirs 09:05, 1 October 2009 (UTC)[reply]
While not a bad idea on the face, what happens if you have your rocket malfunction and crash, or worse, explode in midair? Then instead of having your nuclear waste contained in a nice small barrel, you scattered it across 500 square miles. Googlemeister (talk) 13:09, 1 October 2009 (UTC)[reply]
I did say a reliable rocket. Also, nuclear waste is regularly enclosed in strong containers that will withstand explosions and falls from great heights, but I agree that there is a small risk. Perhaps it would be safer to transport it in very small quantities up to an orbiting space station just outside the atmosphere, then send the rocket from there. Of course, once we get nuclear fusion going properly, there will be no nuclear waste, and we can use the fusion reactor to get rid on non-nuclear toxic waste. Dbfirs 16:25, 1 October 2009 (UTC)[reply]
The Earth is moving sideways at 30 km/s relative to the Sun — it'd need to drop like a stone to fall into the Sun. If ever feasible you'd probably want to do gravity assist(s) to help. Considering that we have what, 70,000 tons of nuclear waste and a thousand tons of the highest grade waste alone, at the current prices (hundreds of millions of dollars per ton) this would be a very expensive method of disposal. Sagittarian Milky Way (talk) 18:59, 1 October 2009 (UTC)[reply]
Solar sails can do the job, I believe (in a manner equivalent to a sailing boat tacking). --Tango (talk) 22:17, 1 October 2009 (UTC)[reply]

Quantum paradox

Let's say you're doing a single particle double-slit experiment with detectors. You shoot the electron at the screen and measure which slit it goes through. Your detector has a unique design. It stores the information somewhere and prevents it from decohering. If you ask it for which slit the particle went through, it tells you. However, you can also tell it that you don't want to know. Then, it makes a conjugate measurement of it's information storage, erasing the original observation. Obviously, whether you see interference patterns depends on what question you ask your detector. The paradox is - what if you ask your detector after you have run all the trials? What if you pick your question to be the one that would cause the set of probabilities that is not the one you observed (ie. see interference + ask which slit or don't see interference + and ask to forget)? 76.67.76.19 (talk) 04:23, 1 October 2009 (UTC)[reply]

Read Wheeler's delayed choice experiment and Delayed choice quantum eraser Graeme Bartlett (talk) 04:39, 1 October 2009 (UTC)[reply]
The short answer is that the pattern on the screen will be the same (no interference) regardless of what you do to your stored information and when. Your choice of measurement basis affects what you learn from the measurement, but the pattern on the screen is only affected by the initial act of copying. -- BenRG (talk) 12:58, 1 October 2009 (UTC)[reply]
But how is this a measurement? The detector prevents the information from decohering, so it isn't really a classical measurement. 76.67.73.124 (talk) 04:47, 3 October 2009 (UTC)[reply]

temp. & pressure depdendence of heat of fusion/vaporisation

The heat of vaporization article tells me that enthalpy and entropy of a phase change does not change significantly with T, but the graph on that page shows different! Which is it? John Riemann Soong (talk) 04:54, 1 October 2009 (UTC)[reply]

Clearly, it depends on the material. The graph illustrates several volatile fluids which have a dramatic enthalpy-of-vaporization dependence on temperature. Most materials do not have such a large variation. Nimur (talk) 05:56, 1 October 2009 (UTC)[reply]
So for most metals (for phase changes like changing from alpha-manganese to beta-manganese, or from solid to liquid lead) I can assume that the enthalpy of phase change is constant? John Riemann Soong (talk) 07:58, 1 October 2009 (UTC)[reply]
Yes, as an approximation assuming the volume change on phase change is relatively insignificant.83.100.251.196 (talk) 12:18, 1 October 2009 (UTC)[reply]

Homework check

Per the "no homework" rule, I'm posting on here to check that my thinking is correct, not just fishing for answers. Anyway, among others, I was given these two problems for a Physics class:

"You are sitting in a boat in the middle of a lake. Also in the boat with you is an anvil. If you throw the anvil into the lake, what will happen to the level of the water in the lake?"

My thought here is that the level will go down. When the anvil is inside the boat, it's displacing water equal to its weight. When it's thrown out, it's displacing water equal to its volume. Since the anvil is presumably less dense than the lake (barring a wooden anvil on a mercury lake), the water level will go down.

"You are attempting to lift a bar with a man grasping either end. Once off the ground, the men each perform a chin-up. Does this increase the amount of work needed to lift the bar? What if the men were jumping up and down on the bar, instead?"

Newton's third law would mean that as the men perform their chin-ups, the force they're exerting downwards on the bar would equal the force upwards on themselves. As I'm lifting the whole lot, the work required to lift the system wouldn't change. If they're jumping up and down on the bar, the initial jump upwards wouldn't change the work needed for the same reasons, but when they land, they're adding an additional downward force due to gravity. So the work in that case would increase, provided they complete at least one full jump and landing.

Is my thinking correct? 84.66.41.175 (talk) 07:01, 1 October 2009 (UTC)[reply]

I think you meant "more dense" instead of less dense in your answer for the first question. For your second question do they start performing their exercises during or after you have lifted the bar? --antilivedT | C | G 07:18, 1 October 2009 (UTC)[reply]
Your first answer looks good, although you made an error here: "the anvil is presumably less dense than the lake". In your second answer, for the bar to remain stationary, you must oppose the downward force on the bar with an equal, opposite force. As the man begins to pull up, he exerts another force on the bar due to his upward acceleration, in addition to his body weight. As he decelerates at the top of his pull up, he exerts less force on the bar than his body weight, so it becomes slightly easier for you to keep the bar still. Axl ¤ [Talk] 07:30, 1 October 2009 (UTC)[reply]
Yes, it's all a matter of timing. In the first example, as you throw the anvil slightly upwards to clear the side of the boat the average water level may rise marginally, then it will fall as the anvil is in the air, then rise again as the anvil enters the water, but the overall effect is a slight fall (too small to measure in a normal-size lake with a liftable anvil). In your second example, it is theoretically possible to reduce the amount of work done in lifting the bar if the chin-up men complete their upward acceleration before you start lifting, and decelerate (slightly pushing the bar upwards) as you lift. They would have to perform very athletic chin-ups to achieve this. With the opposite timing, the work needed would increase. Exactly the same applies to those jumping on the bar except that no upwards force is possible (unless they hook a toe under the bar), so work will nearly always increase. Remember that there is an increased force as they begin their jump, as well as on landing, so the work needed would always increase (over the empty bar) unless you lifted in mid-jump. Such split-second timings are unlikely to be achieved in practice. If the number of chin-ups or jumps is an integer during your lift, then the total work done is the same as it would be if the men were just hanging from or standing on the bar. Remember that if you lift whilst the jumpers are in the air, you are lifting only the bar, but if you lift whilst they are starting the jump, you are lifting the bar, plus their weight, plus an additional force needed to accelerate them upwards. Very hard work! You must be super-human! Dbfirs 08:46, 1 October 2009 (UTC)[reply]
The work needed to lift the bar and the gymnasts is equal to their combined weight times the height through which you raise their combined centre of mass. As long as the gymnasts end up in the same position relative to the bar at the end of each lift, the height through which the centre of mass is raised is the same, so the work done is the same. It does not matter whether they do chin-ups, jump up and down, or do a whole Olympic high bar routine while the bar is being lifted - as long as they always end up in the same position relative to the bar at the end of the lift, the work done on the bar and gymnasts is always the same. Gandalf61 (talk) 10:12, 1 October 2009 (UTC)[reply]
True, but only if they end up at rest in that position. Extra work needs to be done if they are still moving at the end of the lift. Also, the two jumpers or chin-uppers might have done some of the work. Dbfirs 16:18, 1 October 2009 (UTC)[reply]
If the gymnasts are still moving at the end of the lift, extra or less lifting work will be needed depending on the direction of their combined kinetic energies. Slightly extra work is needed in any case due to the energy they dissipate in causing air turbulence. Cuddlyable3 (talk) 23:38, 1 October 2009 (UTC)[reply]
Good point - I'd forgotten about air turbulance, though the amount of extra work is minimal at the speed most people do chin-ups! Kinetic energy is a scalar, so how can less work be needed if kinetic energy is given to the gymnasts? I'm still thinking about this ... Dbfirs 11:49, 2 October 2009 (UTC)[reply]

Followup (Sam)

Somewhat related, perhaps someone could comment on the physics of the event portrayed at about 4:30 of this video: [37]Baseball Bugs What's up, Doc? carrots 10:30, 1 October 2009 (UTC)[reply]
And to whoever it was that tried to delete the above, he needs to back off. I would, in fact, like to hear the explanation (if any) as to why the item at 4:30 is realistic or fanciful, as I've always wondered about it, and it's within the general realm of the original question here. More importantly, why it's realistic or fanciful. →Baseball Bugs What's up, Doc? carrots 11:04, 1 October 2009 (UTC)[reply]
Well, it pains me to break the news to you - but cartoons aren't real. I understand how this might come as a shock...would this be a good time to break the news about Santa Clause and the Tooth Fairy? SteveBaker (talk) 11:14, 1 October 2009 (UTC)[reply]
I didn't say it was "real", I asked if it was "realistic". You've got a heavy object (again, an anvil) falling from a significant height, and Sam catches it. Assuming all that's possible, does the boat drop in the water somewhat? It sounds to me like a cousin to OP's question. The boat sinking is obviously a gag. But would the boat's waterline drop at all? →Baseball Bugs What's up, Doc? carrots 11:23, 1 October 2009 (UTC)[reply]
Yes - of course it drops...the ship has to displace water equal to it's weight...including that of the anvil. In cartoon-physics, anvils have near-infinite mass - so the ship sinks. SteveBaker (talk) 11:35, 1 October 2009 (UTC)[reply]
If the anvil were heavier than the water displaced by the boat itself, presumably it could sink the ship, right? So if it were dropped from an airplane and had sufficient mass, combined with a sturdy deck and a Sam of super-human strength, it might sink the ship, right? But instead it's dropped from the crows nest - making the situation fairly similar to the OP's question. When the anvil is dropped from the crows nest, would the ship rise slightly? And would it then fall back to its original waterline once Sam catches it, but also drop a little farther due to the added force of the anvil's fall? →Baseball Bugs What's up, Doc? carrots 11:42, 1 October 2009 (UTC)[reply]
But the weight of the anvil is not invariant under situational transformation in comic physics. In this case, this is even true in real physics - the anvil up on the mast is further from the middle of the Earth than down on the deck. I'd guess that the mast is roughly 1.5 times the radius of the Earth, so on the deck it will have near infinite massIn comic physics, mass and weight are interchangable*blush* (per Steven), while in the crows nest it will only have near (infinite/5.25) mass, obviously much less. --Stephan Schulz (talk) 11:51, 1 October 2009 (UTC)[reply]
The mast doesn't look that long, but it could be trick photography. The point being, if the ship were made of balsa, i.e. lighter than the anvil, then the anvil dropping probably wouldn't sink the ship by weighing it down, it would probably sink it by breaking all the way through it. But assuming the deck is strong enough to withstand the anvil hitting it (or to withstand it hitting Sam), would the anvil cause the waterline to at least momentarily drop somewhat lower than it was when the anvil was at rest in the crows nest? I say momentarily because I'm guessing the boat would have to eventually rise back up to its original waterline, with the anvil again being at rest. Right? →Baseball Bugs What's up, Doc? carrots 12:01, 1 October 2009 (UTC)[reply]
You can only gauge the relative mast height from long distance shots showing the curvature of the Earth. When the anvil is released, the ship will start an upward movement, and perform a dampened oscillation around the new waterline. When the anvil hits Sam, a downwards impulse is transmitted to the ship, which leads to a second oscillation. I think those two will simply add up until the ship settles again. --Stephan Schulz (talk) 13:44, 1 October 2009 (UTC)[reply]
That makes sense. And the more I think about it, the more similar this question is to the original, especially as Sam throws the anvil off the ship once it's underwater. It's really not too different from OP's hypothesis, except it's taken to a comical extreme. →Baseball Bugs What's up, Doc? carrots 13:52, 1 October 2009 (UTC)[reply]
The ship would rise when the anvil was released from the top of the crow's nest. Time would elapse before the anvil reconnected with the ship. Upon reconnecting with the ship there would result a lowering of the ship to the level it had been at prior the the anvil being released from the point of the crow's nest. There would be bobbing motion through all these steps, as the ship is free to move to beyond its point of equilibrium, in the process of finding its point of equilibrium. The only factor that I am aware that I am leaving out of this description involves the distance from the center of the Earth as that may be negligible in this instance. I think if the distance from the center of the Earth were factored in the result would be a lower level of the ship in the water with the anvil on the deck than with the anvil in the crow's nest. Bus stop (talk) 14:05, 1 October 2009 (UTC)[reply]

(I'm getting rid of my comment, (And it's followups) and making a similar comment on the talk page. ) ) APL (talk) 14:44, 1 October 2009 (UTC)[reply]

WHAAOE - see cartoon physics. hydnjo (talk) 15:35, 1 October 2009 (UTC)[reply]
Yeh, except I'm not talking cartoon physics, that's just an illustration. →Baseball Bugs What's up, Doc? carrots 16:57, 1 October 2009 (UTC)[reply]
Yeh, I know - I was just sayin'... hydnjo (talk) 20:58, 1 October 2009 (UTC)[reply]

calculating the entropy of a substance

So my problem set isn't making any sense for me. Say that below some phase change temperature T_c (this particular case is 993 K), an alpha phase is favoured over the beta phase. But apparently if I calculate the free energy of each phase (from enthalpy and entropy of each phase), I predict a much lower phase transition temperature. On the other hand, I'm given the enthalpy of the phase transition and the temperature it occurs at (so I could well, predict the entropy change of the phase transition, and I have this value).

I am however given their differential heat capacities ... which happen to be constant (not dependent on temperature). But don't the free energy lines (with dependence on temperature) on the phases cross ultimately because of temperature-differential changes in heat capacity? John Riemann Soong (talk) 11:46, 1 October 2009 (UTC)[reply]

"But apparently if I calculate the free energy of each phase (from enthalpy and entropy of each phase), I predict a much lower phase transition temperature." - as a guess - are you assuming that the entropy enthalpy is constant with T ?
Second part - the free energy lines cross because of the dependence of free energy with temperature ie don't you need to calculate
GT= Gstart/standard - ʃSTdT
Where ST is the entropy as a function of temperature - the relationship includes the heat capacities..
Is this what you've been trying, orr something else (there's usually different approaches to the same answer...)83.100.251.196 (talk) 12:28, 1 October 2009 (UTC)[reply]
ok I'm a bit confused - have you tried calculating G using the entropy derived from the heat capacities? ie using dS = (C/T) dT as described at Specific heat capacity - does that give a different or closer answer? I think that's wrong.83.100.251.196 (talk) 18:48, 1 October 2009 (UTC)83.100.251.196 (talk) 13:43, 1 October 2009 (UTC)[reply]

How do electromagnetic waves propagate through a vacuum?

I'm looking for the simplest, most layman-friendly answer to be able to understand this. Thanks.20.137.18.50 (talk) 12:05, 1 October 2009 (UTC)[reply]

Electromagnetic waves are a form of energy, so unlike sound waves, they don't need a "medium" in order to propogate. All they need is space. That's what I recall from 9th-grade science, anyway. →Baseball Bugs What's up, Doc? carrots 12:36, 1 October 2009 (UTC)[reply]
You can visualize them as Photons, traveling through space at the speed of light. If you want to think of it as a wave, then consider it as electric and magnetic fields oscillating perpendicular to each other traveling at the speed of light. Look up Light. Rkr1991 (Wanna chat?) 12:44, 1 October 2009 (UTC)[reply]
And if you want to think of light as a wave - the varying magnetic field produc es and electric field (at right angles), and the varying electric field produces a magnetic field. These fields exist OK in a vacuum. Graeme Bartlett (talk) 14:03, 1 October 2009 (UTC)[reply]
What is a field in a vacuum if a vacuum is nothingness?20.137.18.50 (talk) 14:07, 1 October 2009 (UTC)[reply]
A vacuum isn't really nothingness. It's just the quantum state with the lowest possible energy. See vacuum state. Red Act (talk) 14:18, 1 October 2009 (UTC)[reply]
The electromagnetic field progates by medium of photons, the electromagnetic fields gauge boson. Photons are the field, and the field is photons, the two 'classical style' parts of the wave-prticle duality as described by QED. The field as such is not smooth as is taught in classical dynamics, but lumpy and made up of phtons as in QED. However for most purposes the smooth classical model suffices, and is useful for being drastically simpler in most cases. Elocute (talk) 17:37, 1 October 2009 (UTC)[reply]
Magnets work fine in a vacuum; they attract and repel just as in air. Electrostatic attraction and repulsion works fine in a vacuum. Thus there is no reason that electromagnetic waves would not propagate in a vacuum. Edison (talk) 04:47, 2 October 2009 (UTC)[reply]

Followup (Photon)

OK, here's a spinoff question, which I'm not so sure has an answer. Supposedly an object's mass increases exponentially as it accelerates toward the speed of light, right? So that, in theory, a massive object can never get very close to the speed of light, and certainly not equal to speed of light, because its mass would become infinite, assuming the math about light-speed is correct. Or that's what I recall from 9th grade science. However, an extremely small particle can be accelerated nearly to the speed of light. The built-in assumption is that photons are traveling at the speed of light, i.e. at this assumed upper-bound of speed. But are they really? Do photons have literally 0 mass, or do they merely have a mass that's too small to measure? In the latter case, are they really traveling at that upper bound of speed? Or are they just short of it? →Baseball Bugs What's up, Doc? carrots 12:52, 1 October 2009 (UTC)[reply]

Well, I'll give the annoying answer. Yes - the photons are moving at the speed of light, because their speed is what we know, call, and have measured to be the speed of light since, by definition, light moves at the speed of light (well, usually). ~ Amory (usertalkcontribs) 13:00, 1 October 2009 (UTC)[reply]
As far as we know, photons have a rest mass of zero (or, more preciely, the concept of rest mass does not apply to them since they can never be at rest). If they had a non-zero rest mass then the speed of light would depend on its energy, and hence on its frequency - but observations of gamma-ray bursts in distant galaxies show to a high degree of accuracy that the speed of light is independent of its frequency [38]. Usenet Physics FAQ lists some other theoretical and experimental evidence for zero rest mass. Gandalf61 (talk) 13:10, 1 October 2009 (UTC)[reply]
By definition, the speed of light is the speed that photons travel. That follows. The speed of light also varies depending on what it's traveling through. A partial or spinoff answer to the OP's question, ironically, is that the presence of a "medium", e.g. clear glass, actually slows down the speed of the photons traveling through it, if I recall correctly. When "the speed of light" is said by itself, it usually implies "in a vacuum". But I wonder if the speed of light itself, in a vaccuum, is not the actual upper bound of speed, or whether light-speed is just short of the actual upper bound - maybe so slightly short of it that it's unmeasurable. →Baseball Bugs What's up, Doc? carrots 13:18, 1 October 2009 (UTC)[reply]
Gandalf's above bit about photons having zero rest mass is how we assert that c is in fact the upper bound. We have no evidence that speeds above that exist. Of course, once you get to "so slightly short that it's unmeasurable"... well, that's why 0.999...=1Lomn 13:25, 1 October 2009 (UTC)[reply]
OK, I think that covers it. Danke. →Baseball Bugs What's up, Doc? carrots 13:33, 1 October 2009 (UTC)[reply]
Photons always (under every conceivable circumstance) travel at c, this is in all meaningful sense a tautology, as when we measure c, we measure the speed of photons. However the speed of light is a different issue for reasons relating to the difference between physical and effective (optical) path lengths. That is, a photon going through a medium doesnt go slower, it just travels further at the same speed. It is correctly noted that it cannot be taken as given that all photons (and thus all light) travels at the same speed, no purely logical reasoning will deduce this, but that as Gandalf said, the evidence to support it is strong. A side note that the mass increas is not exponential, it has a pole at c.Elocute (talk) 17:33, 1 October 2009 (UTC)[reply]
How fast is a photon going at the moment that it hits a mirror and is about to be reflected? →Baseball Bugs What's up, Doc? carrots 23:46, 1 October 2009 (UTC)[reply]
There's not really such a moment since there's an uncertainty relation between the energy of the photon and the point in time it hits the mirror. That aside, there's no reason the velocity of a photon has to be continuous in time (unlike for a massive particle). In other words, it can go from c to -c without ever being in between and in fact it's required to. You could call the value at that point of discontinuity zero, but what would that mean? Rckrone (talk) 00:07, 2 October 2009 (UTC)[reply]
OK, so it's always either headed toward the mirror or headed away from the mirror. The instant it touches the mirror is essentially 0 time, hence it doesn't count, i.e. it doesn't "compress" and bounce off like a spaldeen or something. So let's say the room is full of mirrors. The photons would bounce all over the place, forever, wouldn't they? Or if not, then where would they go? →Baseball Bugs What's up, Doc? carrots 02:16, 2 October 2009 (UTC)[reply]
Yeah, in the ideal case with perfect mirrors they would bounce around forever. What happens in practice is that mirrors absorb some of the photons that hit them and the photons' energy becomes heat (or gives some electrons enough energy to make a break for it). The other issue is that photons have momentum, so when they bounce of a stationary object (like an electron of some atom), they have to transfer some energy to the object for momentum to be conserved. The effect is pretty minimal for visible light but a comes into play with high energy photons like X-rays (see Compton scattering). Rckrone (talk) 02:57, 2 October 2009 (UTC)[reply]
OK, that was the missing piece. The photon travels at light speed while it's a photon, but it can be transformed into another state, yes? I think this exhausts my questions on this topic. I probably knew all this stuff in 9th grade, but that was some time back. I appreciate your help and your patience. →Baseball Bugs What's up, Doc? carrots 03:17, 2 October 2009 (UTC)[reply]
Here's a paper that goes through a lot of experiments that have been done to show that photons have zero rest mass [39] to high accuracy. As others have mentioned, this isn't something scientists can take for granted, despite the fact that the universal constant c that crops up in relativity is also called "the speed of light." They're only the same under the assumption that photons have zero rest mass, although that seems to be a pretty good assumption. A story that worked out differently was neutrinos which were first assumed to be massless but experiments showed this to be wrong. Rckrone (talk) 19:25, 1 October 2009 (UTC)[reply]
The em wave creates its own virtual particle pairs as it propagates? just an idea. —Preceding unsigned comment added by 79.75.110.116 (talk) 23:41, 1 October 2009 (UTC)[reply]

cooling question

If I take an airstream which is at 500K and vaporize 1 gallon of water to cool it to 400K, will evaporating the same amount of water in the same volume of airstream which is at 600K cool it to 500K, or would it be more like 480K? Googlemeister (talk) 15:14, 1 October 2009 (UTC)[reply]

It depends how exact you want to be. See specific heat capacity. Evaporating the same amount of water requires the same amount of energy - but that assumes that the only relevant energy transfer is into the latent heat of vaporization. It also assumes a constant heat-capacity of the air stream with respect to temperature - again, this is probably a fairly invalid assumption, because at higher temperatures, the density may be lower due to thermal expansion - so there may be less mass present. Heat capacity is usually quantified in terms of mass, so a moving airstream needs some way to parameterize its mass flux. The intricacies are going to depend on what approximations you want to make - but since you've only specified so few parameters, it might be safe to use a very simple model. Nimur (talk) 16:35, 1 October 2009 (UTC)[reply]

Name of integrated foreign DNA?

Is there a technical name for foreign DNA (from some kind of pathogen, for example) that has been integrated into the DNA of the host? I'm not asking about integrons or retroviral proviruses, but rather some non-directed integration. Thanks. —Preceding unsigned comment added by 129.49.7.150 (talk) 15:31, 1 October 2009 (UTC)[reply]

Exogenous DNA. If it was artificially done, the organism is a transgenic organism and the "chimeric" DNA is called recombinant DNA. 152.16.15.144 (talk) 19:24, 1 October 2009 (UTC)[reply]

Moving with a cat

Can't you move to a new home with a cat, or is it just a legend? Quest09 (talk) 17:36, 1 October 2009 (UTC)[reply]

Sure you can. It might take them a little while to get adjusted to their new territory, but they will. It depends on their disposition. Some will hide for awhile, others will adjust quickly. Just be as good to them as you can, and over time they'll get used to their new surroundings. →Baseball Bugs What's up, Doc? carrots 17:40, 1 October 2009 (UTC)[reply]
(EC)Google it? This is the first result I clicked on. Stressful but not impossible. Vimescarrot (talk) 17:41, 1 October 2009 (UTC)[reply]
I've personally done it. Twice. With the same cat. I had no problems with it, and neither did the cat ;-) J.delanoygabsadds 17:43, 1 October 2009 (UTC)[reply]
Just make sure you know if the property permits pets if you are renting. Googlemeister (talk) 18:11, 1 October 2009 (UTC)[reply]
That's a must. →Baseball Bugs What's up, Doc? carrots 21:05, 1 October 2009 (UTC)[reply]
I've moved house with a cat. It is important to keep them indoors for a week or two so they learn that this new place is home, otherwise they may try to go home to the old house. We shut the cat in the bathroom for the first day or two so as not to expose her to the chaos of moving. It all went very smoothly. --Tango (talk) 19:00, 1 October 2009 (UTC)[reply]
Indoors vs. indoors/outdoors vs. outdoors cat is a key issue. And "sealing them off" in a room with everything they need will likely ease their discomfort (along with lessening the likelihood of them running away). →Baseball Bugs What's up, Doc? carrots 21:05, 1 October 2009 (UTC)[reply]
We had a cat that used to jump up on the back shelf of the car to come with us when we went off on weekends. Didn't half surprise a person who was admiring our nice soft toy at the back when he stretched and yawned. Dmcq (talk) 22:49, 1 October 2009 (UTC)[reply]
Turns out that we had heavy rain the day after moving day and our outdoor cat sat like a soldier waiting to be let in, Our old house had "cat doors" but I hadn't yet installed them in our new house. Anecdotally, they seem to tolerate the move better than →hydnjo (talk) 22:59, 1 October 2009 (UTC)[reply]
If your cat doesn't seem to cooperate, there are other remedies -- penubag  (talk) 02:55, 2 October 2009 (UTC)[reply]
I strongly advise you to keep the cat locked up at the new place for a week or so to make it clear that this is now "home". Otherwise, there's a reasonable chance it will attempt an "incredible journey" to its old place, as our cat did after a move. --Sean 14:54, 2 October 2009 (UTC)[reply]

Why are clouds so "lumpy"?

I've been thinking a lot about clouds recently (as one does when trying to write software to make pretty pictures of them) - and it occurs to me that it's a bit odd that clouds form such dense, compact 'lumps'. The basic idea is that water evaporates off the ocean or something - the water vapor moves up to some altitude where it condenses into droplets...but why isn't there just a more or less uniform sheet of grey 'stuff'? Why does it form clumps that look like bunny rabbits? (Except that one over there that looks like a cartoon fish). Some of them are uniform-ish sheets of grey - but some of them are really dense - with relatively hard edges. Mutual gravitation of the water droplets? Nah...something like surface tension? Van-der-waal's forces? None of these seems plausible. SteveBaker (talk) 21:50, 1 October 2009 (UTC)[reply]

My daughter asked me this a little while ago. I think it is because (1) when warm and cold fronts of air mix you get edges between air of differing temp and humidity (2) when one side of the interface starts to form water droplets it rapidly changes its absorption and radiation of radiation which extenuates the difference between it and the other side of the boundary. I don't know though it is a guess, but some feedback seems needed to give the clean edge versus natural mixing, and the upper side of clouds are often sharper which fits with radiative heat loss once droplet form. I have also noticed that sometimes jet trails of planes disappear but sometimes they just spread out and out thicken. Again, my assumption is that in the right marginal conditions the heat loss from radiating droplets can be a positive feedback cooling air and increasing droplets but generally the mixing of the water and heat just reverts the trail to the surrounding conditions. --BozMo talk 22:07, 1 October 2009 (UTC)[reply]
"compact lump" sounds to me like some mechanism is minimising surface area, that would tend to result in a sphere (like surface tension, but it isn't a liquid so I guess it isn't actually surface tension, I'm not sure what it would be [certainly not gravity, it will be something based on electromagnetism]). I imagine it is flattened by atmospheric effects - the atmosphere is full of layers with different winds, temperatures, humidities, etc., the cloud is restricted to one layer (if it crossed the boundary between two layers with different winds it would get ripped apart). That flattening restricts your degrees of freedom, which increases the chance of spheres meeting and becoming bunny rabbits. I'm not sure what stops those bunny rabbits becoming a larger sphere... --Tango (talk) 22:10, 1 October 2009 (UTC)[reply]
While you are looking at the topic Steve the WP article on Double diffusive convection could do with some work. There was a whole graduate course on this when I was a lad, including atmospherics...I don't think there is any attraction involved, only feedback but I could be wrong. --BozMo talk 22:14, 1 October 2009 (UTC)[reply]
There are essentially two types of cloud (well there are in climate models, and probably in reality): convective and "large scale". Large scale clouds are when the entire atmospheric layer becomes saturated (typically because the whole lot cools, either in place or because of forced ascent) and clouds form - these are the grey slabs. Convective ones are the bunny rabbits. In this case, the entire layer isn't supersaturated, but the column is unstable with respect to overturning, once you take into account heat of condensation. But in that case the ascent is organised, and this is where the clouds form. So ascending air cools, hence vapour condenses, hence the air warms and ascends, and so on (i.e. since the air is being heated as it ascends the moist adiabat is different to the dry, so the plume stays buoyant). And the plume is what makes the clouds lumpy (it can be a tall plume if its a thundercloud or a rather squat one if only a little puffy cloud) William M. Connolley (talk) 22:46, 1 October 2009 (UTC)[reply]
Here's a not-too-incorrect explanation of why daytime clouds are lumpy.[40] The lumpiness is fundamentally caused by atmospheric turbulence. There's a lot going on in clouds -- rumor has it there are people who make their living trying to predict them. Short Brigade Harvester Boris (talk) 00:16, 2 October 2009 (UTC)[reply]
I think there's really another question here. Besides being "lumpy", some clouds, including the standard cumulus humilis cloud have an oddly discrete outline to them, which I think adds to the impression of lumpiness. "Non-lumpy" clouds like nimbostratus, or even fog obviously also have all kinds of structural variation due to slight differences in temperature and pressure, we just don't think of them as lumpy because they're indistinct. So maybe a better question would be to ask why some clouds, like cumulus, are so sharply delineated, thus letting us see their lumpiness. I have a couple of hypotheses of my own on that, but no evidence to back them up and so will keep them to myself in hopes somebody else can come forward with something more concrete. I would also like to record for posterity that this is easily the most times I've typed the word "lump" in a single paragraph! Matt Deres (talk) 03:39, 2 October 2009 (UTC)[reply]
The pedant in me wished to point out that you didn't use the word "lump" even once in the preceding paragraph. "lumpy"=3, "lumpiness"=2. It's OK though - I have my 'inner pedant' under control again now. SteveBaker (talk) 20:34, 2 October 2009 (UTC)[reply]
Cumuliform clouds are buoyant parcels that are distinct from the air around them. They are the same as "thermals" in the lower atmosphere that as birds and glider pilots use. If the thermal ascends so high that it cools enough for the water vapor in it to condense, it becomes visible as a cloud. (This height is called the "lifting condensation level" and corresponds to the flat bottoms of these clouds.) The distinct boundary of the cloud is essentially the interface between the rising parcel and the outside environment. Stratiform (layered) clouds have less distinct boundaries because they are large masses of air that get lifted rather than distinct blobs, as William explains above. The high-level wispy clouds are made of ice and have fuzzy boundaries because ice crystals doesn't evaporate as readily as water droplets. Short Brigade Harvester Boris (talk) 13:10, 2 October 2009 (UTC)[reply]

October 2

cancerous lump?

Hi, im 15 years old, and i recently found a lump on my left arm, near the tricep i guess, and i think i have a similar one on my right arm, my dad said its just muscle, but i dont think so, is it natural or cancerous? —Preceding unsigned comment added by Joejoe94 (talkcontribs) 01:53, 2 October 2009 (UTC)[reply]

If you are concerned, you and your parents should consult a doctor. Wikipedia does not provide medical advice. — Lomn 02:09, 2 October 2009 (UTC)[reply]

The rain after earthquake

Did anyone notice this? Or is it just me, I remembered or notice it's almost always raining after the quake. Or maybe it's just a matter of geographical location or coincident or false observation. roscoe_x (talk) 02:10, 2 October 2009 (UTC)[reply]

It's not quite what you're looking for, but we have articles on earthquake cloud and earthquake weather (and see also here. Those are phenomena said to presage the earthquake, though, not come after it. You'll note that our article asserts that "Geologists maintain that there is no connection between weather and earthquakes. " but it is uncited and, again, is talking about pre-earthquake effects. Matt Deres (talk) 03:44, 2 October 2009 (UTC)[reply]
It rains every day there so there would be strange event if it was dry. Graeme Bartlett (talk) 11:29, 2 October 2009 (UTC)[reply]

Would it be possible to shatter a toilet by alighting on it?

In Guinness a team of kung fu masters demolished a real house without tools so it's very probably humanly possible. Whether without injury I dunno, they could've just pulled it out and thrown it off the second floor (when the floor still existed). Could an amount of jumping-on force reasonable for a person merely wanting to get on (say, to change the lightbulb or close the upper window) break it and cause grave injury? Corner case of course, (i.e. temperature unfavorable, the dude is the heaviest person that has the athletic capability to want to do such a thing, he does this all the time for decades, and the workers who brought it up the stairs ages ago banged it pretty good) Sagittarian Milky Way (talk) 05:54, 2 October 2009 (UTC)[reply]

Your question reminded me of the Great John Toilet Company, but sadly, their site appears to be gone. Their products still seem to be available, and a google search will turn up pictures and brochures. One reason cited for making these XXL johns was just what you are talking about: safety. Presumably, a standard toilet might break under a very heavy person, if there was some kind of defect. And it's easy to imagine very nasty damage: split femoral arteries and the like.--Rallette (talk) 10:24, 2 October 2009 (UTC)[reply]
Well, it'd be entirely reasonable for a person jumping up on a toilet to break cheap plastic hinges holding the seat together. It's then entirely reasonable that serious injury could result when the now-detached seat slides out from under someone. It's not as dramatic as the porcelain suddenly shattering, but it's a lot more likely to be called "plausible" by the Mythbusters. — Lomn 13:14, 2 October 2009 (UTC)[reply]
Corner case is a concept usually applied to more complex systems than a toilet but I understand what the OP is getting at. It would take a kung fu master to answer how best to shatter a toilet deliberately but the OP has described a feasible way of doing it without martial art. In the obviously unlikely event of a toilet shattering accidentally merely by someone standing on it, and that causing a serious injury, the cause of the breakage would become a legal question - who, if anyone, can be held negligent? We can't predict a legal outcome. Cuddlyable3 (talk) 14:34, 2 October 2009 (UTC)[reply]
Our Dawn Eden article describes an incident where a toilet "collapsed", causing injury, but I can't find the original reference, so it's not clear if the shitter shattered. --Sean 15:05, 2 October 2009 (UTC)[reply]

i demolished a toilet with a firecracker Talk Shugoːː 17:23, 2 October 2009 (UTC)[reply]

Ethical guidelines for biologists?

I'm currently doing research on the Tanager Expedition and I'm trying to get past the physical sickness and vomiting reflex I experience each time I read about the "sampling" of the species they encounter. Granted, this is 1923, but I'm curious about the ethical guidelines of scientific expeditions today in comparison to the ones conducted in the early 20th century. My question is this: If and when a biologist encounters a new species, or even a previously known species that is undergoing current study, what are the guidelines about killing that species in the wild to bring back to the laboratory? In other words, how does science approach the study of living organisms, while at the same time, acknowledging their right to exist in spite of our efforts to study them? Reading the notes of the primary biologist for Tanager, it seems like there were no ethical guidelines, and anything they encountered that was of any interest was immediately killed on the spot. Viriditas (talk) 07:44, 2 October 2009 (UTC)[reply]

There's clearly an awareness of the issue of ethics in field biology, and WP has content regarding ethics in conservation biology. There are efforts to raise awareness of this issue in the scientific literature and textbooks. Here's a very nice essay on the subject. It's worth noting that "Science" is not a single organization, and scientists come from diverse cultural backgrounds, so it would be naive to suggest that they have a single, shared set of values or rules. -- Scray (talk) 10:08, 2 October 2009 (UTC)[reply]
I don't think science is practiced differently around the world. My understanding is that scientific methodology is a shared value, so we should find a common procedure for studying captured organisms. So, what is it? Catch and release? Kill and dissect? Study from afar and don't interfere? It sounds like it depends on the situation rather than our values. I doubt there are that many different scenarios so it should be easy to describe. I'm just curious what they are and how they differ from those used in the past, or even if they do. Viriditas (talk) 11:08, 2 October 2009 (UTC)[reply]
see Whaling_in_Japan "Scientific research"--Digrpat (talk) 13:06, 2 October 2009 (UTC)[reply]
Conservation is recognized as an important, actually "vital" is the appropriate adjective, issue today. The links Scray provided demonstrate this. The situation was different in 1923 in many ways: the diversity of species was less well known and the rate at which they were being lost due to human interaction was little known (and do we really know today?). It is only recently that the ethic of conserving biodiversity has been recognized internationally following advances in understanding of genetics, atmospheric pollution and the potential for discovering medicines in nature. At the time of the Tanager Expedition to Hawaii, not yet a US state but already the site of American political and military involvement for which the US would later apologise, Hawaii was with its many islands an ecological terra incognita. That gave the motivation for the expedition. It had some ethically laudable goals that included halting the extinction of local species by feral rabbits and investigating hitherto unknown species. Then as now, one has to discover a species before one can take specific steps to conserve it. That research usually requires obtaining "samples" in some way, dead or alive. We should not protest against this or this or doing warning: image that may upset this because these are the only ways to understand the natural world. The OP is clearly upset by this aspect of Biology and is curious about how the ethical issues are debated today. However the Ref. Desk is not meant to be a debating forum so if you need advice or opinions, it's better to ask elsewhere. Cuddlyable3 (talk) 14:14, 2 October 2009 (UTC)[reply]
I'm not upset by biology in and of itself nor am I trying to debate, but you seem to have reinforced my original point by saying "these are the only ways to understand the natural world". Obviously, such an absolutist statement can only be wrong. While a tricorder is still science fiction, there may come a time when biologists of the 19th and 20th century are looked at in the same way we look at primitive societies who practiced human sacrifice to appease their Gods. Surely for them, it was the "only" way. Viriditas (talk) 19:28, 2 October 2009 (UTC)[reply]
It's of note, of course, that Gray didn't kill anyone to make his drawings. It makes him somewhat different than the frog bits. One can, of course, object to frog dissection on the grounds that when done by elementary school students, its educational import is rather low and could be easily simulated. It seems to be more a rite of passage than anything else. Which, depending on how you feel about killing frogs, may or may not be justified. (And just speaking historically, the Conservation movement was actually quite prominent in America in 1923 and had existed for quite some time by that point.) --Mr.98 (talk) 14:59, 2 October 2009 (UTC)[reply]
This is a wee bit tangential, but you might take a look at Gregg Mitman's Reel Nature: America's Romance with Wildlife on Film. One of the main narrative arcs, if I remember correctly, is how wildlife photography was initially developed as a way of documenting the hunting of big game (for science!) but over time, the actual killing of the game became frowned upon, and less invasive forms of observation were adapted and valued (hence the difference between Jane Goodall and, say, a primatologist of the 1920s). --Mr.98 (talk) 14:59, 2 October 2009 (UTC)[reply]
Thanks, that sounds exactly like what I was looking for here. Viriditas (talk) 19:28, 2 October 2009 (UTC)[reply]

Particles from space time traveling thanks to special relativity...

I can not think of this to save my life, hopefully someone could help me out... What is the name of the particles that enter the earth's atmosphere due to solar flares and make it to the earths surface alive, even though they do not live long enough to make the journy from space to the surface of the earth. But what keeps them alive is that their timeline has slowed down due to special relativity.

Can anyone help me on this?

Thanks! Goatofmendes (talk) 13:14, 2 October 2009 (UTC)[reply]

Are you thinking of muons, created by cosmic rays hitting atoms in the upper atmosphere? AlmostReadytoFly (talk) 13:28, 2 October 2009 (UTC)[reply]
Muon Dmcq (talk) 13:29, 2 October 2009 (UTC)[reply]

YES! Thank You! Goatofmendes (talk) 13:59, 2 October 2009 (UTC)[reply]

acidity... how it changes

great scientists at the other end ... please help me on a small ... question.. ..can we have a plot of change of acidic character of an acid when we keep on adding water to it. if not .. then i would love to know the trend of acidity in above case. thankyou SCI-hunter (talk) —Preceding undated comment added 13:17, 2 October 2009 (UTC).[reply]

What is "the acidic character"? Do you mean the pKa of the acid itself, the pH of the solution, or...? DMacks (talk) 13:26, 2 October 2009 (UTC)[reply]
You yourself can intuitively guess that "acidity" will decrease when you keep adding water to an "acid",ie, the substance will become less acidic. The rest depends on how you scale acidity. Rkr1991 (Wanna chat?) 13:36, 2 October 2009 (UTC)[reply]

thanks a lot for your quick attention ... what i actually want to know is :((pH)) of solution as we go on adding water h3o+concentration in acid goes on increasing.so pH of acid must decrease.but it is diluted.kindly elaborate with plot. thanks SCI-hunter (talk) —Preceding undated comment added 13:43, 2 October 2009 (UTC).[reply]

Adding more H2O decreases the concentration of H3O+ in it. As you said, it's diluting the same amount of acid in a larger amount of solution. DMacks (talk) 16:26, 2 October 2009 (UTC)[reply]
pH gives some general rules but in particular cases something surprising may happen see Activity (chemistry) Dmcq (talk) 16:51, 2 October 2009 (UTC)[reply]
A favorite exam question deals with the pH of a water solution with an H+ concentration of 10–7 M. What happens if it's diluted? DMacks (talk) 19:28, 2 October 2009 (UTC)[reply]

Human intelligence

Will prolonged ice age cause human intelligence to devolve? If a community could not operate any 'schools' say starting today for say the next 10 / 100 / 1000 / 10,000 .... years and most energies of adults are focused on survival, can we truly expect the children to be more intelligent or less? To me it seems like we will become less intelligent until the environmental conditions change and allow us back to have educational focus and less survival focus.

-- Srini Kasturi <email redacted> 2009/10/02 —Preceding unsigned comment added by 192.86.100.35 (talk) 15:16, 2 October 2009 (UTC)[reply]

Intelligence increased in the past when there was no schools and there were ice ages. What makes you think that another ice age and no schools would reverse that? Dmcq (talk) 16:20, 2 October 2009 (UTC)[reply]
We're in an ice age, by the way. Do you mean in a more general sense, would intelligence devolve in some sort of generalised apocalypse scenario? Vimescarrot (talk) 16:30, 2 October 2009 (UTC)[reply]
I think he mean Glacial period, which we're not in at the moment. TastyCakes (talk) 17:06, 2 October 2009 (UTC)[reply]
"Intelligence" is such a poorly define concept that it is impossible to say. Knowledge would certainly decrease without education. --Tango (talk) 16:59, 2 October 2009 (UTC)[reply]

Intelligence as measured by IQ tests would likely decrease, just as it has increased with modern education systems. People tend to view "IQ" as a innate, static property, but it really isn't. Evidence suggests that education and environment have an enormous effect on it. TastyCakes (talk) 17:10, 2 October 2009 (UTC)[reply]

It wouldn't negatively affect human intelligence in an evolutionary sense, or at least there's no obvious mechanism to me that would cause that. Even if their knowledge would suffer, I don't think "devolve" is the right word. Rckrone (talk) 18:16, 2 October 2009 (UTC)[reply]

Indeed, I can't see the genetics of intelligence changing. If civilisation completely fell and we lost (almost) all technology, then evolution might move us towards a genetic makeup better suited to the environment, but even that seems unlikely (there wasn't a great deal of evolutionary change during the last glacial period). If we maintain a decent level of technology, then I don't expect much evolution at all - we'll handle everything technologically, like we do now, so almost all selection pressure will be removed. So, nothing interesting for genetics - memetics on the other hand... --Tango (talk) 21:29, 2 October 2009 (UTC)[reply]

The reference desk does not answer requests for opinions or predictions about future events. Cuddlyable3 (talk) 18:42, 2 October 2009 (UTC)[reply]

Do you think they might sometime soon? 72.58.74.34 (talk) 23:59, 2 October 2009 (UTC)[reply]
They might. Wait and see. In the meantime, don't do anything. →Baseball Bugs What's up, Doc? carrots 09:24, 3 October 2009 (UTC)[reply]

Architecture, High Ceilings (United States)

I wasn't sure if Science was the correct category or not...my question is: Why do rooms in buildings built prior to the 1920s/30s in the United States, particularly residential apartments, have higher ceilings than many modern ones? It would've made sense to prefer lower ceilings in that era, when heating technology was less effective. . .Thanks --68.175.44.30 (talk) 16:46, 2 October 2009 (UTC)[reply]

I don't know why but a few ideas occur to me:
Your observation may be selection in that only the better apartments have survived.
High ceilings allowed better circulation of air and that's a good idea with a bad fire particularly if one can afford the fuel.
The kerosene lamps needed air and produced fumes and some soot which was better left on the high ceiling. Dmcq (talk) 17:12, 2 October 2009 (UTC)[reply]
I don't have a definite answear, but I do know that the advent of electric lighting changed architecture. When the sun is high in the sky, you need a tall window for the light to reach deep into the room. Dmcq may also have a point in that upper-class buildings may have had higher ceilings and these are the ones that survive. I think I see this in the old upper and lower-class buildings in my city. The reason may be architectural style, light or simply more room for the chandeliers. EverGreg (talk) 18:11, 2 October 2009 (UTC)[reply]
This reference[41] bemoans the "disregard for...aesthetic dimension of a ceiling" compared with 100 years ago and concludes "Ceiling[s] in houses and apartments usually range from about eight feet to an expansive twelve feet high. Many New York apartments are prewar buildings with ceiling averaging nine feet six inches high; most newer buildings have lower ceilings. Ideally, the ceiling height should be in proportion to the size of the room. The larger the room, the higher the ceiling can be and still please the eye." Cuddlyable3 (talk) 19:03, 2 October 2009 (UTC)[reply]
What makes you think the heating technology was less effective? fire places work pretty well. Dauto (talk) 19:21, 2 October 2009 (UTC)[reply]
Many more people had domestics then too. Having a high ceiling would show your status difference. Dmcq (talk) 20:06, 2 October 2009 (UTC)[reply]
Perhaps it had more to do with staying cool in summer? Hot air rises - so a nice high ceiling allows heat to stay out of your way? That's a guess though. SteveBaker (talk) 20:28, 2 October 2009 (UTC)[reply]
Thanks, everyone! By the way I like your moving fractal images on your user page, Cuddlyable. --68.175.44.30 (talk) 21:21, 2 October 2009 (UTC)[reply]
Not USA, but just to comment that the older part of the house where I was born and brought up had ceilings just over six feet high, and one doorway was only five foot seven high. It dated back to the early 1600s, but many houses in the area where I live have similar low ceilings. When an extension was built in 1835, they made the new doorways and ceilings much higher. Dbfirs 01:53, 3 October 2009 (UTC)[reply]

I think that the rise both in standards of living (thus the price of labor) and in safety standards since the 1920s/30s has meant that building construction is more expensive in real terms than it was then. So builders may be looking more for ways to cut costs now, particularly in apartment buildings where any extra cost is replicated for each unit. But this is just my theory, I don't have a cite and could be quite wrong. --Anonymous, 04:12 UTC, October 3, 2009.

Spanking a baby when it's born

Is it true that they spank a baby when it's born? If so, what is the purpose of this? ScienceApe (talk) 17:43, 2 October 2009 (UTC)[reply]

Whom do you mean by "they"? I'm sure there are many cultural and regional factors in this. Intelligentsium 17:49, 2 October 2009 (UTC)[reply]
The doctor or the nurse. ScienceApe (talk) 17:57, 2 October 2009 (UTC)[reply]
No it's not true. The misconception is derived from the fact that a small number of infants seem to be slow to take their first air breaths, and it is difficult for the attendant adults to restrain themselves from administering some form of tactile stimulation. However, actual spanking is not what is done. alteripse (talk) 17:52, 2 October 2009 (UTC)[reply]
I'm pretty sure this is an old fashioned thing that has survived in movies and television because it makes for good story-telling short-hand. (The entire birth can be presented to the audience as a light slap and a loud cry.) APL (talk) 18:25, 2 October 2009 (UTC)[reply]
The article Midwife notes that in ancient Rome a midwife "inspected the newborn for congenital deformities and testing[sic] its cry to hear whether or not it was robust and hearty." Her method of testing its cry is not specified. Those who promote Unassisted childbirth have produced videos[42][43] of their deliveries where spanking the infant is usually unnecessary. (The boy in the second video typically takes his time to arrive but is vociferous enough once he does.) Cuddlyable3 (talk) 18:30, 2 October 2009 (UTC)[reply]
What they actually do in most parts of the world is some variation on the Apgar test. There is a whole procedure associated with this which assesses how the baby is coping and whether medical intervention is needed. They do test for reflexes (typically by stimulating the babies feet) - and they expect to hear some sort of a cry when they do that - or the baby doesn't score a point for that. Read the article - it explains it pretty well. SteveBaker (talk) 20:16, 2 October 2009 (UTC)[reply]
APL is correct, it's a fiction used by screenwriters. Comet Tuttle (talk) 21:05, 2 October 2009 (UTC)[reply]

Glycerol Surface Tension

What is the surface tension of the mixture between glycerol and water at a ratio of 1:1 (by volume) at room temperature? Thank you, 128.59.151.224 (talk) 19:20, 2 October 2009 (UTC)[reply]

This [44] has the value for $34
or, [45] p81 gives 69mN/m for 68% (w/w) glycerol water which is ~54% by vol.
There's a table here [46] p139 table 6.2.7 from which you could extrapolate a value.83.100.251.196 (talk) 20:27, 2 October 2009 (UTC)[reply]

The speed of light vs. special relativity

Quick silly question hopfully some one can answer for me... If your in a car, traveling at the speed of light, then you turn on the headlights...what happens? Does the light still project due to the velocity already being at the speed of light, then the light particles just travel at double the speed? 74.218.50.226 (talk) 19:44, 2 October 2009 (UTC)[reply]

So someone will point out that it is not possible for objects with nonzero rest mass (presumably this applies to your car) to travel at the speed of light, because it would take an infinite amount of energy to accelerate them to that speed.
That's true, but shouldn't stop us from examining your question; Einstein thought about just such questions, and they helped lead him to the special theory.
One answer is that, for an object (however that word be interpreted) traveling at the speed of light, the Lorentz contraction of time goes all the way to zero. So you don't have time to turn on the switch. This is how it was originally determined that neutrinos have nonzero rest mass. If they had zero rest mass, they would have to travel at the speed of light. But then there would be no way for them to switch between electron, muon, and tau neutrinos, because there would be no time for that to happen in. But we know this switching happens (because there are too few observed electron neutrinos from the Sun). --Trovatore (talk) 20:05, 2 October 2009 (UTC)[reply]
I don't think that's true about neutrino oscillation; I think it's only necessary that the masses not be all the same. The observed oscillation implies that there are three different masses, but one of the three could still be zero, in principle. -- BenRG (talk) 22:53, 2 October 2009 (UTC)[reply]
No, it's correct. If they were massless they would get from the sun to our detectors in zero proper time (to the extent proper time is even defined in that case), so there would be no time for them to oscillate. I guess it's possible that one or both or the flavours the sun doesn't emit could be massless, but then I would expect we would detect almost all them since once a neutrino became that flavour it would be stuck there, and that isn't what we see. --Tango (talk) 23:48, 2 October 2009 (UTC)[reply]
Well, you can't travel AT the speed of light - only a little below it. But let's suppose you're travelling at 99.9999999999999999999999999999999999999999999999999999999999999999999999999% of the speed of light - is that OK? Well, the weird thing about light - and the entire reason we have their weird relativity stuff - is because when you're travelling at that ungodly speed and you turn on your headlights - the light shoots off away from you at EXACTLY the speed of light. In fact - you can't really tell that you're moving at all. Now - there is a problem with that - which is if someone is watching you (flagrently going WAY over the speed limit!) you seem to be moving at ALMOST the speed of light - and the beams from your headlights are travelling only just a fraction faster. That's odd because you seem to think the light is racing away very quickly. What's really going on is that time has compressed for you. SteveBaker (talk) 20:09, 2 October 2009 (UTC)[reply]
(EC) Your car can't go that fast, so let's say instead that it's going 1% of the speed of light. You shoot a bullet out the front window at 2000 feet per second. From your perspective, the bullet is moving away from you at 2000fps. From someone "standing still" as you whiz by at 1%c, the bullet is going 1%c + 2000fps -- faster! Now, you turn on the headlights. From your perspective, the light shoots away from you ... at the speed of light, of course. From the stationary observer's perspective, the light from your headlights is going ... the speed of light! Sorry, I know it doesn't make sense, and it seems like it should be going 1.01c, but the speed of light is constant in all reference frames. --Sean 20:15, 2 October 2009 (UTC)[reply]

These all do make sense to me and thanks for the answers...one last quick question for all of you physicsheads 8') Sorry again if these questions seem silly, but I love physics...but I think I'm missing half of my brain... Why is it that light can only travel at the speed of light? —Preceding unsigned comment added by 74.218.50.226 (talk) 20:55, 2 October 2009 (UTC)[reply]

Please see our Speed of light article, which probably explains in more detail than anyone at the Refdesk will enter into. Comet Tuttle (talk) 21:05, 2 October 2009 (UTC)[reply]
I don't think there really is a "why", it's just the way it is. It falls out of the mathematics without too much effort. --Tango (talk) 21:32, 2 October 2009 (UTC)[reply]
Wouldn't it be fairer to say that it is the mathematics that has been built up to conform with that experimentally observed fact ? Abecedare (talk) 21:42, 2 October 2009 (UTC)[reply]
An "enlightening" discussion that answers a question taken straight from a Steven Wright joke, and which I also wondered about when I first heard it years ago.[47] The second question, about why the speed of light is what it is, I once heard posed differently: "When God created the universe, and when it came to setting the speed of light, did He have any choice?" To put it another way, what is it about the universe that compels light to travel at the speed it does? Maybe if that could be determined, it could answer a lot of other questions about the nature of the universe. →Baseball Bugs What's up, Doc? carrots 05:33, 3 October 2009 (UTC)[reply]

Can't remember a term... (Biology)

Does anyone possibly know what the term is for the effects an environment has on an organism? My mind has gone blank. --Glaesisvellir (talk) 21:02, 2 October 2009 (UTC)[reply]

Many different factors can occur depending on the situation. For example, when I am at my Mother-in-Law's house, the major effect on me is stress. 68.245.48.172 (talk) 21:26, 2 October 2009 (UTC)[reply]
I meant in general. The general term for how an organism's environment effects it's behavior. --Glaesisvellir (talk) 21:41, 2 October 2009 (UTC)[reply]
Tropism? --Sean 23:02, 2 October 2009 (UTC)[reply]
No, not tropism. I mean the general effects that an environment would have on an organism. i.e. why twins can grow up with different personalities. --Glaesisvellir (talk) 03:11, 3 October 2009 (UTC)[reply]
The twin example makes me wonder if you mean "nurture", though that's more pseudophilosophic than biological. - Nunh-huh 03:36, 3 October 2009 (UTC)[reply]

October 3

Is there really meaning to the speed of light?

This shoots off from the most recent comment from the last speed of light question a couple posts above: When God chose the speed of light, did he have a choice? Anyway, given that essentially every measurable quantity is dependent to an extent on the speed of light, is there meaning to its value? For example, all elementary forces are transmitted at the speed of light; the speed of light is the maximum speed of transfer of information; a change in the speed of light would influence the energies of atomic orbitals. So my question really is, if the speed of light were to be changed, universe-wide, how would all of these things it influences scale to eachother? Basically, would everything get smaller at the same proportion, or would the size of things like different atoms, or different bond lengths, scale disproportionately, as I suspect? Someguy1221 (talk) 05:56, 3 October 2009 (UTC)[reply]

Unfortunately there's no well defined answer to this question; it depends on arbitrary notational choices. For example, the energy of a photon is E = hν = hc/λ where ν is the frequency and λ is the wavelength, so changing c won't change the energy-frequency relationship but it will change the energy-wavelength relationship. But Planck could just as well have chosen a constant k with the property that E = k/λ = kν/c, and in that case it would be the other way around, even though the physics is the same. As another example, particle physicists quote masses in kg but also in MeV/c². If you change c then those no longer agree, so the theory isn't even internally consistent without further changes. It is possible to choose new values of the constants and get a new theory that works, but it's not well defined whether you've changed "just one" or "several". It's also possible to change the numeric value of c without changing anything of physical importance. For example, you can just redefine the meter such that c = 123456789 m/s and express all other constants in terms of the new meter. -- BenRG (talk) 09:34, 3 October 2009 (UTC)[reply]
For an interesting take on the potential effects of varying c (and other physical constants) you might like to read the "Mr Tomkins" books by George Gamow. --Phil Holmes (talk) 10:57, 3 October 2009 (UTC)[reply]

Magnetic saturation

I would like to know what will happen if we try to magnetise a magnetic material beyond its saturation,and what behavioral change we get if the materaial back to below saturation.Roon93 (talk) 06:45, 3 October 2009 (UTC) Thanks[reply]

Your first question is answered in Saturation (magnetic). Your second question is answered in Magnetic hysteresis. Red Act (talk) 09:16, 3 October 2009 (UTC)[reply]

solid mechanics

I just need to know what is stress and how can we realize a stress in a body? what is the physical interpretation of it?210.212.244.133 (talk) 09:32, 3 October 2009 (UTC)[reply]

In the simplest, where a body is stretched (in tension) or squeezed (in compression) by a single force and an equal and opposite reaction, the (average) stress is simply the force divided by the cross-sectional area perpendicular to the direction of the force. In more complex cases, where shear stresses are involved, stress has to be treated as a tensor. See stress (mechanics) for more derails. Gandalf61 (talk) 10:23, 3 October 2009 (UTC)[reply]

Speed of light between Casimir plates?

When light moves through a dense medium such as glass or water it slows down. By the same logic, since the gap between Casimir plates is less "dense" in terms of energy than the normal space outside, would for example, a pulse of laser light shone between such plates arrive slightly faster than it would in "normal" space, and (assuming this was the case) could such a phenomenon be exploited for intelligible superluminal signaling?Trevor Loughlin (talk) 10:47, 3 October 2009 (UTC)[reply]